IGGY med surg

अब Quizwiz के साथ अपने होमवर्क और परीक्षाओं को एस करें!

15. A nurse cares for a client who is prescribed a serum catecholamine test. Which action should the nurse take when obtaining the sample? a. Discard the first sample and then begin the collection. b. Draw the blood sample after the client eats breakfast. c. Place the sample on ice and send to the laboratory immediately. d. Add preservatives before sending the sample to the laboratory.

ANS: C A blood sample for catecholamine must be placed on ice and taken to the laboratory immediately. This sample is not urine, and therefore the first sample should not be discarded nor should preservatives be added to the sample. The nurse should use the appropriate tube and obtain the sample based on which drugs are administered, not dietary schedules.

15. A nurse plans care for a client who has hypothyroidism and is admitted for pneumonia. Which priority intervention should the nurse include in this clients plan of care? a. Monitor the clients intravenous site every shift. b. Administer acetaminophen (Tylenol) for fever. c. Ensure that working suction equipment is in the room. d. Assess the clients vital signs every 4 hours.

ANS: C A client with hypothyroidism who develops another illness is at risk for myxedema coma. In this emergency situation, maintaining an airway is a priority. The nurse should ensure that suction equipment is available in the clients room because it may be needed if myxedema coma develops. The other interventions are necessary for any client with pneumonia, but having suction available is a safety feature for this client.

5. A client has external otitis. On what comfort measure does the nurse instruct the client? a. Applying ice four times a day b. Instilling vinegar-and-water drops c. Use of a heating pad to the ear d. Using a home humidifier

ANS: C A heating pad on low or a warm moist pack can provide comfort to the client with otitis externa. The other options are not warranted.

14. A nurse cares for a client who is recovering from a hypophysectomy. Which action should the nurse take first? a. Keep the head of the bed flat and the client supine. b. Instruct the client to cough, turn, and deep breathe. c. Report clear or light yellow drainage from the nose. d. Apply petroleum jelly to lips to avoid dryness.

ANS: C A light yellow drainage or a halo effect on the dressing is indicative of a cerebrospinal fluid leak. The client should have the head of the bed elevated after surgery. Although deep breathing is important postoperatively, coughing should be avoided to prevent cerebrospinal fluid leakage. Although application of petroleum jelly to the lips will help with dryness, this instruction is not as important as reporting the yellowish drainage.

15. A nurse assesses a client with bladder cancer who is recovering from a complete cystectomy with ileal conduit. Which assessment finding should alert the nurse to urgently contact the health care provider? a. The ileostomy is draining blood-tinged urine. b. There is serous sanguineous drainage present on the surgical dressing. c. The ileostomy stoma is pale and cyanotic in appearance. d. Oxygen saturations are 92% on room air.

ANS: C A pale or cyanotic stoma indicates impaired circulation to the stoma and must be treated to prevent necrosis. Blood-tinged urine and serous sanguineous drainage are expected after this type of surgery. Oxygen saturation of 92% on room air is at the low limit of normal.

5. A preoperative nurse is reviewing morning laboratory values on four clients waiting for surgery. Which result warrants immediate communication with the surgical team? a. Creatinine: 1.2 mg/dL b. Hemoglobin: 14.8 mg/dL c. Potassium: 2.9 mEq/L d. Sodium: 134 mEq/L

ANS: C A potassium of 2.9 mEq/L is critically low and can affect cardiac and respiratory status. The nurse should communicate this laboratory value immediately. The creatinine is at the high end of normal, the hemoglobin is normal, and the sodium is only slightly low (normal low being 136 mEq/L), so these values do not need to be reported immediately.

34.A nurse assesses a client who is being treated for hyperglycemic-hyperosmolar state (HHS). Which clinical manifestation indicates to the nurse that the therapy needs to be adjusted? a. Serum potassium level has increased. b. Blood osmolarity has decreased. c. Glasgow Coma Scale score is unchanged. d. Urine remains negative for ketone bodies.

ANS: C A slow but steady improvement in central nervous system functioning is the best indicator of therapy effectiveness for HHS. Lack of improvement in the level of consciousness may indicate inadequate rates of fluid replacement. The Glasgow Coma Scale assesses the clients state of consciousness against criteria of a scale including best eye, verbal, and motor responses. An increase in serum potassium, decreased blood osmolality, and urine negative for ketone bodies do not indicate adequacy of treatment.

8. The nurse is teaching assistive personnel (AP) about hormones that are produced by the adrenal glands. Which hormone has the primary responsibility of maintaining fluid volume and electrolyte composition? a. Sodium b. Magnesium c. Aldosterone d. Renin

ANS: C Aldosterone is a hormone secreted by the adrenal cortex that causes water and sodium absorption to maintain body fluid volume. Renin is secreted by the kidney to trigger angiotensinogen converting angiotensin I to angiotensin II to help control blood pressure. Magnesium and sodium are electrolytes and not hormones.

11. A hospitalized client is being treated for Ewings sarcoma. What action by the nurse is most important? a. Assessing and treating the client for pain as needed b. Educating the client on the disease and its treatment c. Handling and disposing of chemotherapeutic agents per policy d. Providing emotional support for the client and family

ANS: C All actions are appropriate for this client. However, for safety, the nurse should place priority on proper handling and disposal of chemotherapeutic agents.

9. The nurse is teaching a client who is planning to have a total hip arthroplasty. What statement by the client indicates a need for further teaching? a. "I will get an IV antibiotic right before surgery to prevent infection." b. "I may request a regional nerve block as part of the surgical anesthesia." c. "I will receive IV heparin before surgery to decrease the risk of clots." d. "I will receive tranexamic acid to help reduce blood loss during surgery."

ANS: C All of the choices are correct except that IV heparin is not given before or after surgery. A different anticoagulant is given after surgery to prevent postoperative venous thromboembolism, such as deep vein thrombosis and pulmonary embolus.

5. After teaching a client with acromegaly who is scheduled for a hypophysectomy, the nurse assesses the clients understanding. Which statement made by the client indicates a need for additional teaching? a. I will no longer need to limit my fluid intake after surgery. b. I am glad no visible incision will result from this surgery. c. I hope I can go back to wearing size 8 shoes instead of size 12. d. I will wear slip-on shoes after surgery to limit bending over.

ANS: C Although removal of the tissue that is oversecreting hormones can relieve many symptoms of hyperpituitarism, skeletal changes and organ enlargement are not reversible. It will be appropriate for the client to drink as needed postoperatively and avoid bending over. The client can be reassured that the incision will not be visible.

18. A client has a left knee arthrocentesis to remove excess joint fluid. What postprocedure health teaching will the nurse include? a. "Take your opioid medication as prescribed by the primary health care provider." b. "Do not bear weight on your left leg for at least a week after you get home." c. "Monitor the site for bleeding or clear fluid leakage when you are home." d. "Tell your employer that you can't come back to work for 2 to 3 weeks."

ANS: C An arthrocentesis is performed as an ambulatory procedure and may require a mild analgesic such as acetaminophen for discomfort. Opioids are not used. The client may bear weight and return to work, but needs to monitor for bleeding or leakage of synovial fluid at the injection site.

15. A nurse cares for a client who is recovering from a closed percutaneous kidney biopsy. The client states, My pain has suddenly increased from a 3 to a 10 on a scale of 0 to 10. Which action should the nurse take first? a. Reposition the client on the operative side. b. Administer the prescribed opioid analgesic. c. Assess the pulse rate and blood pressure. d. Examine the color of the clients urine.

ANS: C An increase in the intensity of pain after a percutaneous kidney biopsy is a symptom of internal hemorrhage. A change in vital signs can indicate that hemorrhage is occurring. Before other actions, the nurse must assess the clients hemodynamic status.

9. A confused client with pneumonia is admitted with an indwelling catheter in place. During interdisciplinary rounds the following day, which question should the nurse ask the primary health care provider? a. Do you want daily weights on this client? b. Will the client be able to return home? c. Can we discontinue the indwelling catheter? d. Should we get another chest x-ray today?

ANS: C An indwelling catheter dramatically increases the risks of urinary tract infection and urosepsis. Nursing staff should ensure that catheters are left in place only as long as they are medically needed. The nurse should inquire about removing the catheter. All other questions might be appropriate, but because of client safety, this question takes priority.

2. The nurse teaches assistive personnel (AP) about care of an older adult diagnosed with osteoporosis. What teaching would the nurse include? a. "Teach the client to eat high-calcium foods in the diet." b. "Assist the client with activities of daily living." c. "Osteoporosis places the client is at risk for fractures." d. "The client should stay in bed to prevent falling."

ANS: C Anyone who has osteoporosis is at risk for fragility fractures even if he or she does not experience trauma like a fall. The client needs to keep active rather than stay in bed where more bone could be lost. High-calcium foods may not be helpful because bone loss is already severe. There is no indication that the client needs assistance with ADLs.

13. A nurse assesses a client with Alzheimers disease who is recently admitted to the hospital. Which psychosocial assessment should the nurse complete? a. Assess religious and spiritual needs while in the hospital. b. Identify the clients ability to perform self-care activities. c. Evaluate the clients reaction to a change of environment. d. Ask the client about relationships with family members.

ANS: C As Alzheimers disease progresses, the client experiences changes in emotional and behavioral affect. The nurse should be alert to the clients reaction to a change in environment, such as being hospitalized, because the client may exhibit an exaggerated response, such as aggression, to the event. The other assessments should be completed but are not as important as assessing the clients reaction to environmental change.

8. The nurse is teaching a client who had a left humeral biopsy about home care. Which statement by the client indicates understanding of the nurse's teaching? a. "I will take my opioids only when I have severe pain." b. "I will keep my left arm elevated for 24 hours." c. "I will watch for tenderness and warmth around the biopsy site." d. "I will report any discomfort to my primary health care provider immediately."

ANS: C Bone biopsy is an ambulatory procedure which can cause some discomfort but not severe pain. The client can use the affected arm soon after the procedure but should watch for tenderness and warmth which could indicate infection.

10. A client has a metastatic bone tumor. What action by the nurse takes priority? a. Administer pain medication as prescribed. b. Elevate the extremity and apply moist heat. c. Handle the affected extremity with caution. d. Place the client on protective precautions.

ANS: C Bones invaded by tumors are very fragile and fracture easily. For client safety, the nurse handles the affected extremity with great care. Pain medication should be given to control pain. Elevation and heat may or may not be helpful. Protective precautions are not needed for this client.

11. A nurse evaluates laboratory results for a male client who reports fluid secretion from his breasts. Which hormone value should the nurse assess first? a. Posterior pituitary hormones b. Adrenal medulla hormones c. Anterior pituitary hormones d. Parathyroid hormone

ANS: C Breast fluid and milk production are induced by the presence of prolactin, secreted from the anterior pituitary gland. The other hormones would not cause fluid secretion from the clients breast.

21. The nurse is teaching a group of college students about the importance of preventing meningitis. Which health promotion activity is the most appropriate for preventing this disease? a. Eating a well-balanced diet that is high in protein b. Having an annual physical examination c. Obtaining the recommended meningitis vaccination and boosters d. Identifying signs and symptoms for early treatment

ANS: C CDC-recommended vaccinations and boosters are available for prevention of a number of diseases including meningococcal meningitis. While the other activities are appropriate for general health promotion, they are not specific to meningitis prevention.

7. The nurse is teaching a client about cataract surgery. Which statement would the nurse include as part of preoperative preparation? a. "You will receive general anesthesia for the surgical procedure." b. "You will be in the hospital for only 1 to 2 days if everything goes as expected." c. "You will need to put several types of eyedrops in your eyes before and after surgery." d. "You will be on bedrest for about a week after the surgical procedure."

ANS: C Cataract surgery is done as an ambulatory care procedure and the client is not hospitalized, does not receive general anesthesia, and does not need to be on bedrest postoperatively.

16. A nurse is teaching a client with cerebellar function impairment. Which statement should the nurse include in this clients discharge teaching? a. Connect a light to flash when your door bell rings. b. Label your faucet knobs with hot and cold signs. c. Ask a friend to drive you to your follow-up appointments. d. Use a natural gas detector with an audible alarm.

ANS: C Cerebellar function enables the client to predict distance or gauge the speed with which one is approaching an object, control voluntary movement, maintain equilibrium, and shift from one skilled movement to another in an orderly sequence. A client who has cerebellar function impairment should not be driving. The client would not have difficulty hearing, distinguishing between hot and cold, or smelling.

14. A nurse cares for a client who has pyelonephritis. The client states, I am embarrassed to talk about my symptoms. How should the nurse respond? a. I am a professional. Your symptoms will be kept in confidence. b. I understand. Elimination is a private topic and shouldnt be discussed. c. Take your time. It is okay to use words that are familiar to you. d. You seem anxious. Would you like a nurse of the same gender to care for you?

ANS: C Clients may be uncomfortable discussing issues related to elimination and the genitourinary area. The nurse should encourage the client to use language that is familiar to the client. The nurse should not make promises that cannot be kept, like keeping the clients symptoms confidential. The nurse must assess the client and cannot take the time to stop the discussion or find another nurse to complete the assessment.

10. A nurse cares for a client who has pyelonephritis. The client states, "I am embarrassed to talk about my symptoms." How would the nurse respond? a. "I am a professional. Your symptoms will be kept in confidence." b. "I understand. Elimination is a private topic and shouldn't be discussed." c. "Take your time. It is okay to use words that are familiar to you." d. "You seem anxious. Would you like a nurse of the same gender to care for you?"

ANS: C Clients may be uncomfortable discussing issues related to elimination and the genitourinary area. The nurse would encourage the client to use language that is familiar to the client. The nurse must assess the client and cannot take the time to stop the discussion or find another nurse to complete the assessment.

17. A nurse prepares to discharge a client with Alzheimers disease. Which statement should the nurse include in the discharge teaching for this clients caregiver? a. Allow the client to rest most of the day. b. Place a padded throw rug at the bedside. c. Install deadbolt locks on all outside doors. d. Provide a high-calorie and high-protein diet.

ANS: C Clients with Alzheimers disease have a tendency to wander, especially at night. If possible, alarms should be installed on all outside doors to alert family members if the client leaves. At a minimum, all outside doors should have deadbolt locks installed to prevent the client from going outdoors unsupervised. The client should be allowed to exercise within his or her limits. Throw rugs are a slip and fall hazard and should be removed. The client should eat a well-balanced diet. There is no need for a high-calorie or high-protein diet.

16. A client with Mnires disease is in the hospital when the client has an attack of this disorder. What action by the nurse takes priority? a. Assess vital signs every 15 minutes. b. Dim or turn off lights in the clients room. c. Place the client in bed with the upper siderails up. d. Provide a cool, wet cloth for the clients face.

ANS: C Clients with Mnires disease can have vertigo so severe that they can fall. The nurse should assist the client into bed and put the siderails up to keep the client from falling out of bed due to the intense whirling feeling. The other actions are not warranted for clients with Mnires disease.

19. A nurse assesses a client with a rotator cuff injury. Which finding should the nurse expect to assess? a. Inability to maintain adduction of the affected arm for more than 30 seconds b. Shoulder pain that is relieved with overhead stretches and at night c. Inability to initiate or maintain abduction of the affected arm at the shoulder d. Referred pain to the shoulder and arm opposite the affected shoulder

ANS: C Clients with a rotator cuff tear are unable to initiate or maintain abduction of the affected arm at the shoulder. This is known as the drop arm test. The client should not have difficulty with adduction of the arm, nor experience referred pain to the opposite shoulder. Pain is usually more intense at night and with overhead activities.

4. The nurse works with clients who have hearing problems. Which action by a client best indicates goals for an important diagnosis have been met? a. Babysitting the grandchildren several times a week b. Having an adaptive hearing device for the television c. Being active in community events and volunteer work d. Responding agreeably to suggestions for adaptive devices

ANS: C Clients with hearing problems can become frustrated and withdrawn. The client who is actively engaged in the community shows the best evidence of psychosocial adjustment to hearing loss. Babysitting the grandchildren is a positive sign but does not indicate involvement outside the home. Having an adaptive device is not the same as using it, and watching TV without evidence of other activities can also indicate social isolation. Responding agreeably does not indicate the client will actually follow through.

4. A nurse cares for a middle-aged female client with diabetes mellitus who is being treated for the third episode of acute pyelonephritis in the past year. The client asks, What can I do to help prevent these infections? How should the nurse respond? a. Test your urine daily for the presence of ketone bodies and proteins. b. Use tampons rather than sanitary napkins during your menstrual period. c. Drink more water and empty your bladder more frequently during the day. d. Keep your hemoglobin A1c under 9% by keeping your blood sugar controlled.

ANS: C Clients with long-standing diabetes mellitus are at risk for pyelonephritis for many reasons. Chronically elevated blood glucose levels spill glucose into the urine, changing the pH and providing a favorable climate for bacterial growth. The neuropathy associated with diabetes reduces bladder tone and reduces the clients sensation of bladder fullness. Thus, even with large amounts of urine, the client voids less frequently, allowing stasis and overgrowth of microorganisms. Increasing fluid intake (specifically water) and voiding frequently prevent stasis and bacterial overgrowth. Testing urine and using tampons will not help prevent pyelonephritis. A hemoglobin A1c of 9% is too high.

7. A client with diabetes mellitus type 2 has been well controlled with metformin. The client is scheduled for magnetic resonance imaging (MRI) scan with contrast. What priority would the nurse take at this time? a. Teach the client about the purpose of the MRI. b. Assess the client's blood urea nitrogen and creatinine. c. Tell the client to withhold metformin for 24 hours before the MRI. d. Ask the client if he or she is taking antibiotics.

ANS: C Contrast media can be nephrotoxic (damaging to the kidneys). Metformin can also be nephrotoxic and the client should not be exposed to two agents. Clients who have diabetes are already at risk for renal damage.

9. A nurse plans care for a client with Cushings disease. Which action should the nurse include in this clients plan of care to prevent injury? a. Pad the siderails of the clients bed. b. Assist the client to change positions slowly. c. Use a lift sheet to change the clients position. d. Keep suctioning equipment at the clients bedside.

ANS: C Cushings syndrome or disease greatly increases the serum levels of cortisol, which contributes to excessive bone demineralization and increases the risk for pathologic bone fracture. Padding the siderails and assisting the client to change position may be effective, but these measures will not protect him or her as much as using a lift sheet. The client should not require suctioning.

13. A nurse plans care for an 83-year-old client who is experiencing age-related sensory perception changes. Which intervention should the nurse include in this clients plan of care? a. Provide a call button that requires only minimal pressure to activate. b. Write the date on the clients white board to promote orientation. c. Ensure that the path to the bathroom is free from equipment. d. Encourage the client to season food to stimulate nutritional intake.

ANS: C Dementia and confusion are not common phenomena in older adults. However, physical impairment related to illness can be expected. Providing opportunities for hazard-free ambulation will maintain strength and mobility (and ensure safety). Providing a call button, providing the date, and seasoning food do not address the clients impaired sensory perception.

12. A nurse is teaching the daughter of a client who has Alzheimers disease. The daughter asks, Will the medication my mother is taking improve her dementia? How should the nurse respond? a. It will allow your mother to live independently for several more years. b. It is used to halt the advancement of Alzheimers disease but will not cure it. c. It will not improve her dementia but can help control emotional responses. d. It is used to improve short-term memory but will not improve problem solving.

ANS: C Drug therapy is not effective for treating dementia or halting the advancement of Alzheimers disease. However, certain drugs may help suppress emotional disturbances and psychiatric manifestations. Medication therapy may not allow the client to safely live independently.

19. The nurse is taking the vital signs of a client after hemodialysis. Blood pressure is 110/58 mm Hg, pulse 66 beats/min, and temperature is 99.8 F (37.6 C). What is the most appropriate action by the nurse? a. Administer fluid to increase blood pressure. b. Check the white blood cell count. c. Monitor the clients temperature. d. Connect the client to an electrocardiographic (ECG) monitor.

ANS: C During hemodialysis, the dialysate is warmed to increase diffusion and prevent hypothermia. The clients temperature could reflect the temperature of the dialysate. There is no indication to check the white blood cell count or connect the client to an ECG monitor. The other vital signs are within normal limits.

14. A nurse teaches a client who is recovering from a urography. Which instruction should the nurse include in this clients discharge teaching? a. Avoid direct contact with your urine for 24 hours until the radioisotope clears. b. You may have some dribbling of urine for several weeks after this procedure. c. Be sure to drink at least 3 liters of fluids today to help eliminate the dye faster. d. Your skin may become slightly yellow from the dye used in this procedure.

ANS: C Dyes used in urography are potentially nephrotoxic. A large fluid intake will help the client eliminate the dye rapidly. Dyes used in urography are not radioactive, the client should not experience any dribbling of urine, and the dye should not change the color of the clients skin.

2. A nurse assesses a client who has a history of migraines. Which clinical manifestation should the nurse identify as an early sign of a migraine with aura? a. Vertigo b. Lethargy c. Visual disturbances d. Numbness of the tongue

ANS: C Early warning of impending migraine with aura usually consists of visual changes, flashing lights, or diplopia. The other manifestations are not associated with an impending migraine with aura.

4. A nurse assesses a client with renal insufficiency and a low red blood cell count. The client asks, Is my anemia related to the renal insufficiency? How should the nurse respond? a. Red blood cells produce erythropoietin, which increases blood flow to the kidneys. b. Your anemia and renal insufficiency are related to inadequate vitamin D and a loss of bone density. c. Erythropoietin is usually released from the kidneys and stimulates red blood cell production in the bone marrow. d. Kidney insufficiency inhibits active transportation of red blood cells throughout the blood.

ANS: C Erythropoietin is produced in the kidney and is released in response to decreased oxygen tension in the renal blood supply. Erythropoietin stimulates red blood cell production in the bone marrow. Anemia and renal insufficiency are not manifestations of vitamin D deficiency. The kidneys do not play a role in the transportation of red blood cells or any other cells in the blood.

15. The nurse is planning teaching for a client who is starting exenatide extended release (ER) for diabetes mellitus type 2. Which statement will the nurse include in the teaching? a. "Be sure to take the drug once a day before breakfast." b. "Take the drug every evening before bedtime." c. "Give your drug injection the same day every week." d. "Take the drug with dinner at the same time each day."

ANS: C Exenatide ER is an incretin mimetic (GLP-1 agonist) that works with insulin to lower blood glucose levels by reducing pancreatic glucagon secretion, reducing liver glucose production, and delaying gastric emptying. As an extended-release drug, it is given only once a week by injection.

1. A nurse assesses clients on the medical-surgical unit. Which client is at greatest risk for the development of bacterial cystitis? a. A 36-year-old female who has never been pregnant b. A 42-year-old male who is prescribed cyclophosphamide c. A 58-year-old female who is not taking estrogen replacement d. A 77-year-old male with mild congestive heart failure

ANS: C Females at any age are more susceptible to cystitis than men because of the shorter urethra in women. Postmenopausal women who are not on hormone replacement therapy are at increased risk for bacterial cystitis because of changes in the cells of the urethra and vagina. The middle-aged woman who has never been pregnant would not have a risk potential as high as the older woman who is not using hormone replacement therapy.

15. A nurse assesses a client with multiple sclerosis after administering prescribed fingolimod (Gilenya). For which adverse effect should the nurse monitor? a. Peripheral edema b. Black tarry stools c. Bradycardia d. Nausea and vomiting

ANS: C Fingolimod (Gilenya) is an antineoplastic agent that can cause bradycardia, especially within the first 6 hours after administration. Peripheral edema, black and tarry stools, and nausea and vomiting are not adverse effects of fingolimod.

1. The nurse is preparing to teach a client recently diagnosed with multiple sclerosis about taking glatiramer acetate. Which statement by the client indicates a need for further teaching? a. "I will rotate injection sites to prevent skin irritation." b. "I need to avoid large crowds and people with infection." c. "I should report any flulike symptoms to my primary health care provider." d. "I will report any signs of infection to my primary health care provider."

ANS: C Glatiramer is given by subcutaneous injection. The first dose is administered under medical supervision, but the nurse teaches the client how to self-administer the medication after the initial dose, reminding the client about the need to rotate injection sites. Like other immunomodulators, this drug can make the client susceptible to infection. However, flulike symptoms occur more commonly with interferons rather than glatiramer.

2. A client with a family history of glaucoma asks the nurse how to prevent glaucoma? What statement by the nurse is appropriate? a. "You should check with your primary health care provider about eye examination." b. "You should have genetic testing to determine your risk for glaucoma." c. "You should have your intraocular pressure measured once or twice a year." d. "You should check with your primary health care provider about preventive drug therapy."

ANS: C Glaucoma tends to occur more often in clients who have a family history but cannot be prevented. Genetic testing is not the best response because the client's family history is already known. Therefore, early detection by having intraocular pressure measured frequently.

8. A nurse cares for a client who has hypothyroidism as a result of Hashimotos thyroiditis. The client asks, How long will I need to take this thyroid medication? How should the nurse respond? a. You will need to take the thyroid medication until the goiter is completely gone. b. Thyroiditis is cured with antibiotics. Then you wont need thyroid medication. c. Youll need thyroid pills for life because your thyroid wont start working again. d. When blood tests indicate normal thyroid function, you can stop the medication.

ANS: C Hashimotos thyroiditis results in a permanent loss of thyroid function. The client will need lifelong thyroid replacement therapy. The client will not be able to stop taking the medication.

2. A nurse plans care for a client with lower back pain from a work-related injury. Which intervention should the nurse include in this clients plan of care? a. Encourage the client to stretch the back by reaching toward the toes. b. Massage the affected area with ice twice a day. c. Apply a heating pad for 20 minutes at least four times daily. d. Advise the client to avoid warm baths or showers.

ANS: C Heat increases blood flow to the affected area and promotes healing of injured nerves. Stretching and ice will not promote healing, and there is no need to avoid warm baths or showers.

6. The nurse knows that hematopoiesis occurs in what part of the musculoskeletal system? a. Cancellous tissue b. Collagen matrix c. Red marrow d. Yellow marrow

ANS: C Hematopoiesis occurs in the red marrow, which is part of the cancellous tissues containing both types of bone marrow.

6. A nurse asks a client to take deep breaths during an electroencephalography. The client asks, Why are you asking me to do this? How should the nurse respond? a. Hyperventilation causes vascular dilation of cerebral arteries, which decreases electoral activity in the brain. b. Deep breathing helps you to relax and allows the electroencephalograph to obtain a better waveform. c. Hyperventilation causes cerebral vasoconstriction and increases the likelihood of seizure activity. d. Deep breathing will help you to blow off carbon dioxide and decreases intracranial pressures.

ANS: C Hyperventilation produces cerebral vasoconstriction and alkalosis, which increases the likelihood of seizure activity. The client is asked to breathe deeply 20 to 30 times for 3 minutes. The other responses are not accurate.

6. A nurse plans care for a client with hypothyroidism. Which priority problem should the nurse plan to address first for this client? a. Heat intolerance b. Body image problems c. Depression and withdrawal d. Obesity and water retention

ANS: C Hypothyroidism causes many problems in psychosocial functioning. Depression is the most common reason for seeking medical attention. Memory and attention span may be impaired. The clients family may have great difficulty accepting and dealing with these changes. The client is often unmotivated to participate in self-care. Lapses in memory and attention require the nurse to ensure that the clients environment is safe. Heat intolerance is seen in hyperthyroidism. Body image problems and weight issues do not take priority over mental status and safety.

9. A nurse is caring for a client with a history of renal insufficiency who is scheduled for a computed tomography scan of the head with contrast medium. Which priority intervention should the nurse implement? a. Educate the client about strict bedrest after the procedure. b. Place an indwelling urinary catheter to closely monitor output. c. Obtain a prescription for intravenous fluids. d. Contact the provider to cancel the procedure.

ANS: C If a contrast medium is used, intravenous fluid may be given to promote excretion of the contrast medium. Contrast medium also may act as a diuretic, resulting in the need for fluid replacement. The client will not require bedrest. Although urinary output should be monitored closely, there is no need for an indwelling urinary catheter. There is no need to cancel the procedure as long as actions are taken to protect the kidneys.

14. A client has severe tinnitus that cannot be treated adequately. What action by the nurse is best? a. Advise the client to take anti-anxiety medication. b. Educate the client on nerve cutting procedures. c. Refer the client to online or local support groups. d. Teach the client side effects of furosemide (Lasix).

ANS: C If the clients tinnitus cannot be treated, he or she will have to learn to cope with it. Referring the client to tinnitus support groups can be helpful. The other options are not warranted.

8. A client experiences impaired swallowing after a stroke and has worked with speech-language pathology on eating. What nursing assessment best indicates that a priority goal for this problem has been met? a. Chooses preferred items from the menu b. Eats 75% to 100% of all meals and snacks c. Has clear lung sounds on auscultation d. Gains 2 pounds after 1 week

ANS: C Impaired swallowing can lead to aspiration, so the priority goal for this problem is no aspiration. Clear lung sounds is the best indicator that aspiration has not occurred. Choosing menu items is not related to this problem. Eating meals does not indicate the client is not still aspirating. A weight gain indicates improved nutrition but still does not show a lack of aspiration.

8. An older client with diabetes is admitted with a heavily draining leg wound. The clients white blood cell count is 38,000/mm3 but the client is afebrile. What action does the nurse take first? a. Administer acetaminophen (Tylenol). b. Educate the client on amputation. c. Place the client on contact isolation. d. Refer the client to the wound care nurse.

ANS: C In the presence of a heavily draining wound, the nurse should place the client on contact isolation. If the client has discomfort, acetaminophen can be used, but this client has not reported pain and is afebrile. The client may or may not need an amputation in the future. The wound care nurse may be consulted, but not as the first action.

25. The nurse is teaching a client how to increase the flow of dialysate into the peritoneal cavity during dialysis. Which statement by the client demonstrates a correct understanding of the teaching? a. I should leave the drainage bag above the level of my abdomen. b. I could flush the tubing with normal saline if the flow stops. c. I should take a stool softener every morning to avoid constipation. d. My diet should have low fiber in it to prevent any irritation.

ANS: C Inflow and outflow problems of the dialysate are best controlled by preventing constipation. A daily stool softener is the best option for the client. The drainage bag should be below the level of the abdomen. Flushing the tubing will not help with the flow. A diet high in fiber will also help with a constipation problem.

16. A nurse obtains a sterile urine specimen from a clients Foley catheter. After applying a clamp to the drainage tubing distal to the injection port, which action should the nurse take next? a. Clamp another section of the tube to create a fixed sample section for retrieval. b. Insert a syringe into the injection port and aspirate the quantity of urine required. c. Clean the injection port cap of the drainage tubing with povidone-iodine solution. d. Withdraw 10 mL of urine and discard it; then withdraw a fresh sample of urine.

ANS: C It is important to clean the injection port cap of the catheter drainage tubing with an appropriate antiseptic, such as povidone-iodine solution or alcohol. This will help prevent surface contamination before injection of the syringe. The urine sample should be collected directly from the catheter; therefore, a second clamp to create a sample section would not be appropriate. Every sample from the catheter is usable; there is the need to discard the first sample.

2. The nurse is teaching new assistive personnel (AP) about caring for older adults. Which statement would the nurse include about hearing ability of this client group? a. "You need to talk very loudly when communicating with these clients." b. "You always need to check each client's ears for excess ear wax." c. "Remember to face the client when talking with him or her." d. "Assess each client's hearing ability using the voice or whisper test."

ANS: C Losing one's hearing is not a normal change of aging although high frequency sounds may be more difficult to hear. AP does not perform assessments and it is not necessary to talk loudly or shout unless a hearing impairment exists. Therefore, facing the client is the best strategy when communicating with most older adults.

19. The primary health care provider prescribes methotrexate (MTX) for a client with a new diagnosis of rheumatoid arthritis. The nurse provides health teaching about the drug. What statement by the nurse is appropriate to include about methotrexate? a. "It will take at least 1 to 2 weeks for the drug to help relieve your symptoms." b. "The drug is very expensive but there are pharmacy plans to help pay for it." c. "The drug can increase your risk for infection, so you should avoid crowds." d. "It's OK for you to drink about 2 to 3 glasses of wine each week while taking the drug."

ANS: C MTX takes up to 4 to 6 weeks to begin to help relieve RA symptoms and is very inexpensive. Clients should avoid alcohol due to the potential for liver toxicity. MTX suppresses the immune system which makes clients susceptible to infection. The nurse teaches clients to avoid crowds and anyone with a known infection.

14. After teaching a client who is scheduled for magnetic resonance imaging (MRI), the nurse assesses the clients understanding. Which client statement indicates a correct understanding of the teaching? a. I must increase my fluids because of the dye used for the MRI. b. My urine will be radioactive so I should not share a bathroom. c. I can return to my usual activities immediately after the MRI. d. My gag reflex will be tested before I can eat or drink anything.

ANS: C No postprocedure restrictions are imposed after MRI. The client can return to normal activities after the test is complete. There are no dyes or radioactive materials used for the MRI; therefore, increased fluids are not needed and the clients urine would not be radioactive. The procedure does not impact the clients gag reflex.

6. A nurse assesses a client who is recovering from a transsphenoidal hypophysectomy. The nurse notes nuchal rigidity. Which action should the nurse take first? a. Encourage range-of-motion exercises. b. Document the finding and monitor the client. c. Take vital signs, including temperature. d. Assess pain and administer pain medication.

ANS: C Nuchal rigidity is a major manifestation of meningitis, a potential postoperative complication associated with this surgery. Meningitis is an infection; usually the client will also have a fever and tachycardia. Range-of- motion exercises are inappropriate because meningitis is a possibility. Documentation should be done after all assessments are completed and should not be the only action. Although pain medication may be a palliative measure, it is not the most appropriate initial action.

3. A nurse assesses a client who is recovering from a subtotal thyroidectomy. On the second postoperative day the client states, I feel numbness and tingling around my mouth. What action should the nurse take? a. Offer mouth care. b. Loosen the dressing. c. Assess for Chvosteks sign. d. Ask the client orientation questions.

ANS: C Numbness and tingling around the mouth or in the fingers and toes are manifestations of hypocalcemia, which could progress to cause tetany and seizure activity. The nurse should assess the client further by testing for Chvosteks sign and Trousseaus sign. Then the nurse should notify the provider. Mouth care, loosening the dressing, and orientation questions do not provide important information to prevent complications of low calcium levels.

. 3. A nurse assesses a client who is recovering from a subtotal thyroidectomy. On the first postoperative day before discharge, the client states, "I feel numbness and tingling around my mouth." What action does the nurse take? a. Offer mouth care. b. Loosen the dressing. c. Assess for muscle twitching. d. Ask the client orientation questions.

ANS: C Numbness and tingling around the mouth or in the fingers and toes are manifestations of hypocalcemia, which could progress to cause tetany and seizure activity. The nurse would assess for muscle twitching and, if present, notify the surgeon or Rapid Response Team to give calcium gluconate or other IV calcium replacement. Mouth care, loosening the dressing, and orientation questions do not provide important information to prevent complications of low calcium levels.

1. The nurse teaches an 80-year-old client with diminished peripheral sensation. Which statement would the nurse include in this client's teaching? a. "Place soft rugs in your bathroom to decrease pain in your feet." b. "Bathe in warm water to increase your circulation." c. "Look at the placement of your feet when walking." d. "Walk barefoot to decrease pressure injuries from your shoes."

ANS: C Older clients with decreased sensation are at risk of injury from the inability to sense changes in terrain when walking. To compensate for this loss, the client is instructed to look at the placement of his or her feet when walking. Throw rugs can slip and increase fall risk. Bath water that is too warm places the client at risk for thermal injury.

4. A nurse assesses clients at a community center. Which client is at greatest risk for lower back pain? a. A 24-year-old female who is 25 weeks pregnant b. A 36-year-old male who uses ergonomic techniques c. A 45-year-old male with osteoarthritis d. A 53-year-old female who uses a walker

ANS: C Osteoarthritis causes changes to support structures, increasing the clients risk for low back pain. The other clients are not at high risk.

17. A client has long-term rheumatoid arthritis that especially affects the hands. The client wants to finish quilting a baby blanket before the birth of her grandchild. What response by the nurse is appropriate? a. "Let's ask your provider about increasing your pain pills." b. "Hold ice bags against your hands before quilting." c. "Try a paraffin wax dip 20 minutes before you quilt." d. "You need to stop quilting before it destroys your fingers."

ANS: C Paraffin wax dips are beneficial for decreasing pain in arthritic hands and lead to increased mobility. Increasing pain pills may not help with movement. Ice has limited use unless the client has a "hot" or exacerbated joint. The client wants to finish the project, so the nurse would not negate its importance by telling the client it is destroying her joints.

3. A nurse cares for a client with excessive production of thyrocalcitonin (calcitonin). For which electrolyte imbalance should the nurse assess? a. Potassium b. Sodium c. Calcium d. Magnesium

ANS: C Parafollicular cells produce thyrocalcitonin (calcitonin), which regulates serum calcium levels. Calcitonin has no impact on potassium, sodium, or magnesium balances.

11. A nurse is caring for a client with paraplegia who is scheduled to participate in a rehabilitation program. The client states, I do not understand the need for rehabilitation; the paralysis will not go away and it will not get better. How should the nurse respond? a. If you dont want to participate in the rehabilitation program, Ill let the provider know. b. Rehabilitation programs have helped many clients with your injury. You should give it a chance. c. The rehabilitation program will teach you how to maintain the functional ability you have and prevent further disability. d. When new discoveries are made regarding paraplegia, people in rehabilitation programs will benefit first.

ANS: C Participation in rehabilitation programs has many purposes, including prevention of disability, maintenance of functional ability, and restoration of function. The other responses do not meet this clients needs.

4. After teaching a client with bacterial cystitis who is prescribed phenazopyridine (Pyridium), the nurse assesses the clients understanding. Which statement made by the client indicates a correct understanding of the teaching? a. I will not take this drug with food or milk. b. If I think I am pregnant, I will stop the drug. c. An orange color in my urine should not alarm me. d. I will drink two glasses of cranberry juice daily.

ANS: C Phenazopyridine discolors urine, most commonly to a deep reddish orange. Many clients think they have blood in their urine when they see this. In addition, the urine can permanently stain clothing. Phenazopyridine is safe to take if the client is pregnant. There are no dietary restrictions or needs while taking this medication.

26. A nurse provides phone triage to a pregnant client. The client states, I am experiencing a burning pain when I urinate. How should the nurse respond? a. This means labor will start soon. Prepare to go to the hospital. b. You probably have a urinary tract infection. Drink more cranberry juice. c. Make an appointment with your provider to have your infection treated. d. Your pelvic wall is weakening. Pelvic muscle exercises should help.

ANS: C Pregnant clients with a urinary tract infection require prompt and aggressive treatment because cystitis can lead to acute pyelonephritis during pregnancy. The nurse should encourage the client to make an appointment and have the infection treated. Burning pain when urinating does not indicate the start of labor or weakening of pelvic muscles.

3. The client's electronic health record indicates a sensorineural hearing loss. What assessment question does the nurse ask to determine the possible cause? a. "Do you feel like something is in your ear?" b. "Do you have frequent ear infections?" c. "Have you been exposed to loud noises?" d. "Have you been told your ear bones don't move?"

ANS: C Sensorineural hearing loss can occur from damage to the cochlea, the eighth cranial nerve, or the brain. Exposure to loud music is one etiology. The other questions are related to conductive hearing loss.

3. The clients chart indicates a sensorineural hearing loss. What assessment question does the nurse ask to determine the possible cause? a. Do you feel like something is in your ear? b. Do you have frequent ear infections? c. Have you been exposed to loud noises? d. Have you been told your ear bones dont move?

ANS: C Sensorineural hearing loss can occur from damage to the cochlea, the eighth cranial nerve, or the brain. Exposure to loud music is one etiology. The other questions relate to conductive hearing loss.

17. A client is placed on fluid restrictions because of chronic kidney disease (CKD). Which assessment finding would alert the nurse that the clients fluid balance is stable at this time? a. Decreased calcium levels b. Increased phosphorus levels c. No adventitious sounds in the lungs d. Increased edema in the legs

ANS: C The absence of adventitious sounds upon auscultation of the lungs indicates a lack of fluid overload and fluid balance in the clients body. Decreased calcium levels and increased phosphorus levels are common findings with CKD. Edema would indicate a fluid imbalance.

1. The nurse assesses a client who has possible bladder cancer. What common assessment finding associated with this type of cancer would the nurse expect? a. Urinary retention b. Urinary incontinence c. Painless hematuria d. Difficulty urinating

ANS: C The classic and most common finding in clients who have bladder cancer is painless and intermittent hematuria that can be with gross or microscopic. Dysuria, frequency, and urgency occur in clients who have bladder infection or obstruction.

2. The nurse is assessing a client for risk of developing metabolic syndrome. Which risk factor is associated with this health condition? a. Hypotension b. Hyperthyroidism c. Abdominal obesity d. Hypoglycemia

ANS: C The client at risk for metabolic syndrome typically has hypertension, abdominal obesity, hyperlipidemia, and hyperglycemia.

8. A nurse assesses a client who is recovering from the implantation of a vagal nerve stimulation device. For which clinical manifestations should the nurse assess as common complications of this procedure? (Select all that apply.) a. Bleeding b. Infection c. Hoarseness d. Dysphagia e. Seizures

ANS: C, D Complications of surgery to implant a vagal nerve stimulation device include hoarseness (most common), dyspnea, neck pain, and dysphagia. The device is tunneled under the skin with an electrode connected to the vagus nerve to control simple or complex partial seizures. Bleeding is not a common complication of this procedure, and infection would not occur during the recovery period.

25. A nurse cares for an older adult client who is recovering from a leg amputation surgery. The client states, I dont want to live with only one leg. I should have died during the surgery. How should the nurse respond? a. Your vital signs are good, and you are doing just fine right now. b. Your children are waiting outside. Do you want them to grow up without a father? c. This is a big change for you. What support system do you have to help you cope? d. You will be able to do some of the same things as before you became disabled.

ANS: C The client feels like less of a person following the amputation. The nurse should help the client to identify coping mechanisms that have worked in the past and current support systems to assist the client with coping. The nurse should not ignore the clients feelings by focusing on vital signs. The nurse should not try to make the client feel guilty by alluding to family members. The nurse should not refer to the client as being disabled as this labels the client and may fuel the clients poor body image.

16. A nurse cares for a client recovering from an above-the-knee amputation of the right leg. The client reports pain in the right foot. Which prescribed medication should the nurse administer first? a. Intravenous morphine b. Oral acetaminophen c. Intravenous calcitonin d. Oral ibuprofen

ANS: C The client is experiencing phantom limb pain, which usually manifests as intense burning, crushing, or cramping. IV infusions of calcitonin during the week after amputation can reduce phantom limb pain. Opioid analgesics such as morphine are not as effective for phantom limb pain as they are for residual limb pain. Oral acetaminophen and ibuprofen are not used in treating phantom limb pain.

5. A nurse assesses a client with a spinal cord injury at level T5. The client's blood pressure is 184/95 mm Hg, and the client presents with a flushed face and blurred vision. After raising the head of the bed, what action would the nurse take next? a. Initiate oxygen via a nasal cannula. b. Recheck the client's blood pressure. c. Palpate the bladder for distention. d. Administer a prescribed beta blocker.

ANS: C The client is manifesting symptoms of autonomic dysreflexia. Common causes include bladder distention, tight clothing, increased room temperature, and fecal impaction. If persistent, the client could experience neurologic injury such as s stroke. The other actions are not appropriate for this complication.

7. A nurse assesses a client with a spinal cord injury at level T5. The clients blood pressure is 184/95 mm Hg, and the client presents with a flushed face and blurred vision. Which action should the nurse take first? a. Initiate oxygen via a nasal cannula. b. Place the client in a supine position. c. Palpate the bladder for distention. d. Administer a prescribed beta blocker.

ANS: C The client is manifesting symptoms of autonomic dysreflexia. Common causes include bladder distention, tight clothing, increased room temperature, and fecal impaction. If persistent, the client could experience neurologic injury. Precipitating conditions should be eliminated and the physician notified. The other actions would not be appropriate.

28. A nurse reviews these laboratory values of a client who returned from kidney transplantation 12 hours ago: Sodium 136 mEq/L Potassium 5 mEq/L Blood urea nitrogen (BUN) 44 mg/dL Serum creatinine 2.5 mg/dL What initial intervention would the nurse anticipate? a. Start hemodialysis immediately. b. Discuss the need for peritoneal dialysis. c. Increase the dose of immunosuppression. d. Return the client to surgery for exploration.

ANS: C The client may need a higher dose of immunosuppressive medication as evidenced by the elevated BUN and serum creatinine levels. This increased dose may reverse the possible acute rejection of the transplanted kidney. The client does not need hemodialysis, peritoneal dialysis, or further surgery at this point.

15. A nurse sees clients in an osteoporosis clinic. Which client should the nurse see first? a. Client taking calcium with vitamin D (Os-Cal) who reports flank pain 2 weeks ago b. Client taking ibandronate (Boniva) who cannot remember when the last dose was c. Client taking raloxifene (Evista) who reports unilateral calf swelling d. Client taking risedronate (Actonel) who reports occasional dyspepsia

ANS: C The client on raloxifene needs to be seen first because of the manifestations of deep vein thrombosis, which is an adverse effect of raloxifene. The client with flank pain may have had a kidney stone but is not acutely ill now. The client who cannot remember taking the last dose of ibandronate can be seen last. The client on risedronate may need to change medications.

23. A nurse assesses a client with a brain tumor. The client opens his eyes when the nurse calls his name, mumbles in response to questions, and follows simple commands. How should the nurse document this clients assessment using the Glasgow Coma Scale shown below? a. 8 b. 10 c. 12 d. 14

ANS: C The client opens his eyes to speech (Eye opening: To sound = 3), mumbles in response to questions (Verbal response: Inappropriate words = 3), and follows simple commands (Motor response: Obeys commands = 6). Therefore, the clients Glasgow Coma Scale score is: 3 + 3 + 6 = 12.

26. A nurse is caring for four clients in the neurologic/neurosurgical intensive care unit. Which client should the nurse assess first? a. Client who has been diagnosed with meningitis with a fever of 101 F (38.3 C) b. Client who had a transient ischemic attack and is waiting for teaching on clopidogrel (Plavix) c. Client receiving tissue plasminogen activator (t-PA) who has a change in respiratory pattern and rate d. Client who is waiting for subarachnoid bolt insertion with the consent form already signed

ANS: C The client receiving t-PA has a change in neurologic status while receiving this fibrinolytic therapy. The nurse assesses this client first as he or she may have an intracerebral bleed. The client with meningitis has expected manifestations. The client waiting for discharge teaching is a lower priority. The client waiting for surgery can be assessed quickly after the nurse sees the client who is receiving t-PA, or the nurse could delegate checking on this client to another nurse.

40.After teaching a client who is newly diagnosed with type 2 diabetes mellitus, the nurse assesses the clients understanding. Which statement made by the client indicates a need for additional teaching? a. I should increase my intake of vegetables with higher amounts of dietary fiber. b. My intake of saturated fats should be no more than 10% of my total calorie intake. c. I should decrease my intake of protein and eliminate carbohydrates from my diet. d. My intake of water is not restricted by my treatment plan or medication regimen.

ANS: C The client should not completely eliminate carbohydrates from the diet, and should reduce protein if microalbuminuria is present. The client should increase dietary intake of complex carbohydrates, including vegetables, and decrease intake of fat. Water does not need to be restricted unless kidney failure is present.

8. The nurse is caring for a client who is diagnosed with diabetes insipidus (DI). For what common complication will the nurse monitor? a. Hypertension b. Bradycardia c. Dehydration d. Pulmonary embolus

ANS: C The client who has DI has fluid loss through excessive urination. Decreased fluid volume, or dehydration, is manifested by tachycardia, hypotension, and possibly elevated temperature. Pulmonary embolism (PE) could possible as a clot in the lower extremity (caused by dehydration) could fragment and travel to the lungs.

36. The nurse is caring for a newly admitted client who is diagnosed with hyperglycemic-hyperosmolar state (HHS). What is the nurse's priority action at this time? a. Assess the client's blood glucose level. b. Monitor the client's urinary output every hour. c. Establish intravenous access to provide fluids. d. Give regular insulin per agency policy.

ANS: C The first priority in caring for a client with HHS is to increase blood volume to prevent shock or severe hypotension from dehydration. The nurse would monitor vital signs, urinary output, and blood glucose to determine if interventions were effective. Regular insulin is also indicated but not as the first priority action.

6. The nurse is performing an assessment of a client with possible plantar fasciitis in the right foot. What assessment finding would the nurse expect in the right foot? a. Multiple toe deformities b. Numbness and paresthesias c. Severe pain in the arch of the foot d. Redness and severe swelling

ANS: C The most common assessment finding is the client's report of severe pain in the arch of the foot, especially when walking. The other findings are not typical in clients with this health problem.

25. A nurse cares for a client with urinary incontinence. The client states, I am so embarrassed. My bladder leaks like a young childs bladder. How should the nurse respond? a. I understand how you feel. I would be mortified. b. Incontinence pads will minimize leaks in public. c. I can teach you strategies to help control your incontinence. d. More women experience incontinence than you might think.

ANS: C The nurse should accept and acknowledge the clients concerns, and assist the client to learn techniques that will allow control of urinary incontinence. The nurse should not diminish the clients concerns with the use of pads or stating statistics about the occurrence of incontinence.

21. The charge nurse is orienting a float nurse to an assigned client with an arteriovenous (AV) fistula for hemodialysis in her left arm. Which action by the float nurse would be considered unsafe? a. Palpating the access site for a bruit or thrill b. Using the right arm for a blood pressure reading c. Administering intravenous fluids through the AV fistula d. Checking distal pulses in the left arm

ANS: C The nurse should not use the arm with the AV fistula for intravenous infusion, blood pressure readings, or venipuncture. Compression and infection can result in the loss of the AV fistula. The AV fistula should be monitored by auscultating or palpating the access site. Checking the distal pulse would be an appropriate assessment.

20. A nurse cares for a client with a spinal cord injury. With which interdisciplinary team member should the nurse consult to assist the client with activities of daily living? a. Social worker b. Physical therapist c. Occupational therapist d. Case manager

ANS: C The occupational therapist instructs the client in the correct use of all adaptive equipment. In collaboration with the therapist, the nurse instructs family members or the caregiver about transfer skills, feeding, bathing, dressing, positioning, and skin care. The other team members are consulted to assist the client with unrelated issues.

3. A nurse reviews laboratory results for a client with glomerulonephritis. The clients glomerular filtration rate (GFR) is 40 mL/min as measured by a 24-hour creatinine clearance. How should the nurse interpret this finding? (Select all that apply.) a. Excessive GFR b. Normal GFR c. Reduced GFR d. Potential for fluid overload e. Potential for dehydration

ANS: C, D The GFR refers to the initial amount of urine that the kidneys filter from the blood. In the healthy adult, the normal GFR ranges between 100 and 120 mL/min, most of which is reabsorbed in the kidney tubules. A GFR of 40 mL/min is drastically reduced, with the client experiencing fluid retention and risks for hypertension and pulmonary edema as a result of excess vascular fluid.

3. A nurse works on the postoperative floor and has four clients who are being discharged tomorrow. Which one has the greatest need for the nurse to consult other members of the health care team for post-discharge care? a. Married young adult who is the primary caregiver for children b. Middle-aged client who is post knee replacement, needs physical therapy c. Older adult who lives at home despite some memory loss d. Young client who lives alone, has family and friends nearby

ANS: C The older adult has the most potentially complex discharge needs. With memory loss, the client may not be able to follow the prescribed home regimen. The clients physical abilities may be limited by chronic illness. This client has several safety needs that should be assessed. The other clients all have evidence of a support system and no known potential for serious safety issues.

30. A client has a traumatic brain injury and a positive halo sign. The client is in the intensive care unit, sedated and on a ventilator, and is in critical but stable condition. What collaborative problem takes priority at this time? a. Inability to communicate b. Nutritional deficit c. Risk for acquiring an infection d. Risk for skin breakdown

ANS: C The positive halo sign indicates a leak of cerebrospinal fluid. This places the client at high risk of acquiring an infection. Communication and nutrition are not priorities compared with preventing a brain infection. The client has a definite risk for a skin breakdown, but it is not the immediate danger a brain infection would be.

1. The nurse is caring for a client who has severe osteoarthritis. What primary joint problems will the nurse expect the client to report? a. Crepitus b. Effusions c. Pain d. Deformities

ANS: C The primary assessment finding typically reported by clients who have osteoarthritis is joint pain, although crepitus, effusions (fluid), and mild deformities may occur.

9. The nurse is caring for a client who had a closed reduction of the left arm and notes a large wet area of drainage on the cast. What action is the most important? a. Cut off the old cast. b. Document the assessment. c. Notify the primary health care provider. d. Wrap the cast with gauze.

ANS: C The primary health care provider should be notified to examine the client and determine the source of the drainage. The nurse's assessment should be documented, but that is not the most important action.

18. A client is scheduled for a bone biopsy. What action by the nurse takes priority? a. Administering the preoperative medications b. Answering any questions about the procedure c. Ensuring that informed consent is on the chart d. Showing the clients family where to wait

ANS: C The priority is to ensure that informed consent is on the chart. The preoperative medications should not be administered until the nurse is confident the procedure will occur and the client has already signed the consent, if the medications include anxiolytics or sedatives or opioids. The provider should answer questions about the procedure. The nurse does show the family where to wait, but this is not the priority and could be delegated.

5. A nurse assesses a client who is recovering from a lumbar laminectomy. Which complications should alert the nurse to urgently communicate with the health care provider? (Select all that apply.) a. Surgical discomfort b. Redness and itching at the incision site c. Incisional bulging d. Clear drainage on the dressing e. Sudden and severe headache

ANS: C, D, E Bulging at the incision site or clear fluid on the dressing after a laminectomy strongly suggests a cerebrospinal fluid leak, which constitutes an emergency. Loss of cerebral spinal fluid may cause a sudden and severe headache, which is also an emergency situation. Pain, redness, and itching at the site are normal.

2. A nurse is providing education to a community womens group about lifestyle changes helpful in preventing osteoporosis. What topics does the nurse cover? (Select all that apply.) a. Cut down on tobacco product use. b. Limit alcohol to two drinks a day. c. Strengthening exercises are important. d. Take recommended calcium and vitamin D. e. Walk 30 minutes at least 3 times a week.

ANS: C, D, E Lifestyle changes can be made to decrease the occurrence of osteoporosis and include strengthening and weight-bearing exercises and getting the recommended amounts of both calcium and vitamin D. Tobacco should be totally avoided. Women should not have more than one drink per day.

3. After teaching a male client with a spinal cord injury at the T4 level, the nurse assesses the clients understanding. Which client statements indicate a correct understanding of the teaching related to sexual effects of this injury? (Select all that apply.) a. I will explore other ways besides intercourse to please my partner. b. I will not be able to have an erection because of my injury. c. Ejaculation may not be as predictable as before. d. I may urinate with ejaculation but this will not cause infection. e. I should be able to have an erection with stimulation.

ANS: C, D, E Men with injuries above T6 often are able to have erections by stimulating reflex activity. For example, stroking the penis will cause an erection. Ejaculation is less predictable and may be mixed with urine. However, urine is sterile, so the clients partner will not get an infection.

7. The nurse studying osteoporosis learns that which drugs can cause this disorder? (Select all that apply.) a. Antianxiety agents b. Antibiotics c. Barbiturates d. Corticosteroids e. Loop diuretics

ANS: C, D, E Several classes of drugs can cause secondary osteoporosis, including barbiturates, corticosteroids, and loop diuretics. Antianxiety agents and antibiotics are not associated with the formation of osteoporosis.

6. A nurse assesses a client with paraplegia from a spinal cord injury and notes reddened areas over the clients hips and sacrum. Which actions should the nurse take? (Select all that apply.) a. Apply a barrier cream to protect the skin from excoriation. b. Perform range-of-motion (ROM) exercises for the hip joint. c. Re-position the client off of the reddened areas. d. Get the client out of bed and into a chair once a day. e. Obtain a low-air-loss mattress to minimize pressure.

ANS: C, E Appropriate interventions to relieve pressure on these areas include frequent re-positioning and a low-air-loss mattress. Reddened areas should not be rubbed because this action could cause more extensive damage to the already fragile capillary system. Barrier cream will not protect the skin from pressure wounds. ROM exercises are used to prevent contractures. Sitting the client in a chair once a day will decrease the clients risk of respiratory complications but will not decrease pressure on the clients hips and sacrum.

2. A nurse cares for a client with elevated triiodothyronine and thyroxine, and normal thyroid-stimulating hormone levels. Which actions should the nurse take? (Select all that apply.) a. Administer levothyroxine (Synthroid). b. Administer propranolol (Inderal). c. Monitor the apical pulse. d. Assess for Trousseaus sign. e. Initiate telemetry monitoring.

ANS: C, E The clients laboratory findings suggest that the client is experiencing hyperthyroidism. The increased metabolic rate can cause an increase in the clients heart rate, and the client should be monitored for the development of dysrhythmias. Placing the client on a telemetry monitor might also be a precaution. Levothyroxine is given for hypothyroidism. Propranolol is a beta blocker often used to lower sympathetic nervous system activity in hyperthyroidism. Trousseaus sign is a test for hypocalcemia.

8. A nurse orienting to the postoperative area learns which principles about the postoperative period? (Select all that apply.) a. All phases require the client to be in the hospital. b. Phase I care may last for several days in some clients. c. Phase I requires intensive care unit monitoring. d. Phase II ends when the client is stable and awake. e. Vital signs may be taken only once a day in phase III. f. Some clients may be discharged directly after phase I.

ANS: C,D,E There are three phases of postoperative care. Phase I is the most intense, with clients coming right from surgery until they are completely awake and hemodynamically stable. This may take hours or days and can occur in the intensive care unit or the postoperative care unit. Some patients achieve this level of recovery in phase I and can be discharged directly to home. Phase II ends when the client is at a pre-surgical level of consciousness and baseline oxygen saturation, and vital signs are stable. Phase III involves the extended-care environment and may continue at home or in an extended-care facility if needed.

2. The nurse is reviewing the laboratory profile for a client who has muscular dystrophy. Which laboratory value(s) would the nurse expect to be elevated? (Select all that apply.) a. Calcium (Ca) b. Phosphate (PO4) c. Creatine kinase (CK) d. Lactic dehydrogenase (LDH) e. Aspartate aminotransferase (AST) f. Aldolase (ALD)

ANS: C,D,E,F Muscular dystrophy causes elevations in muscle enzymes and does not affect minerals like calcium and phosphorus.

1. An emergency department nurse cares for a client who is severely dehydrated and is prescribed 3 L of intravenous fluid over 6 hours. At what rate (mL/hr) should the nurse set the intravenous pump to infuse the fluids? (Record your answer using a whole number.) ____ mL/hr

ANS: 500 mL/hr Because IV pumps deliver in units of milliliters per hour, the pump would have to be set at 500 mL/hr to deliver 3 L (3000 mL) over 6 hours. 6x = 3000 x = 500

2. A client in the emergency department is having a stroke. The client weighs 225 pounds. After the initial bolus of t-Pa, at what rate should the nurse set the IV pump? (Record your answer using a decimal rounded to the nearest tenth.) ____ mL/hr

ANS: 1.4 mL/hr The client weighs 102 kg. The dose of t-PA is 0.9 mg/kg with a maximum of 90 mg, so the clients dose is 90 mg. 10% of the dose is given as a bolus IV over the first minute (9 mg). That leaves 81 mg to run in over 59 minutes., which rounds to 1.4 mL/hr.

1. A client in the emergency department is having a stroke and the provider has prescribed the tissue plasminogen activator (t-PA) alteplase (Activase). The client weighs 146 pounds. How much medication will this client receive? (Record your answer using a whole number.) _____ mg

ANS: 60 mg The dose of t-PA is 0.9 mg/kg with a maximum dose of 90 mg. The client weighs 66.4 kg. 0.9 mg 66.4 = 59.76 mg, which rounds to 60 mg.

1. A client in the intensive care unit with acute kidney injury (AKI) must maintain a mean arterial pressure (MAP) of 65 mm Hg to promote kidney perfusion. What is the clients MAP if the blood pressure is 98/50 mm Hg? (Record your answer using a whole number.) _____ mm Hg

ANS: 66 mm Hg

2. A client is scheduled to have a glycosylated hemoglobin (A1C) drawn and asks the nurse why she has to have it. How would the nurse respond? a. "It measures your average blood glucose level for the past 3 months." b. "It determines what type of anemia you may have." c. "It measures the amount of liver glycogen you have." d. "It determines you have some type of leukemia or other blood cancer."

ANS: A

10. After teaching a client with a history of renal calculi, the nurse assesses the clients understanding. Which statement made by the client indicates a correct understanding of the teaching? a. I should drink at least 3 liters of fluid every day. b. I will eliminate all dairy or sources of calcium from my diet. c. Aspirin and aspirin-containing products can lead to stones. d. The doctor can give me antibiotics at the first sign of a stone.

ANS: A Dehydration contributes to the precipitation of minerals to form a stone. Although increased intake of calcium causes hypercalcemia and leads to excessive calcium filtered into the urine, if the client is well hydrated the calcium will be excreted without issues. Dehydration increases the risk for supersaturation of calcium in the urine, which contributes to stone formation. The nurse should encourage the client to drink more fluids, not decrease calcium intake. Ingestion of aspirin or aspirin-containing products does not cause a stone. Antibiotics neither prevent nor treat a stone.

12. A nurse is caring for four clients in the neurologic intensive care unit. After receiving the hand-off report, which client should the nurse see first? a. Client with a Glasgow Coma Scale score that was 10 and is now is 8 b. Client with a Glasgow Coma Scale score that was 9 and is now is 12 c. Client with a moderate brain injury who is amnesic for the event d. Client who is requesting pain medication for a headache

ANS: A A 2-point decrease in the Glasgow Coma Scale score is clinically significant and the nurse needs to see this client first. An improvement in the score is a good sign. Amnesia is an expected finding with brain injuries, so this client is lower priority. The client requesting pain medication should be seen after the one with the declining Glasgow Coma Scale score.

18. An emergency department nurse triages a client with diabetes mellitus who has fractured her arm. Which action should the nurse take first? a. Remove the medical alert bracelet from the fractured arm. b. Immobilize the arm by splinting the fractured site. c. Place the client in a supine position with a warm blanket. d. Cover any open areas with a sterile dressing.

ANS: A A clients medical alert bracelet should be removed from the fractured arm before the affected extremity swells. Immobilization, positioning, and dressing should occur after the bracelet is removed.

8. A nurse notes crepitation when performing range-of-motion exercises on a client with a fractured left humerus. Which action should the nurse take next? a. Immobilize the left arm. b. Assess the clients distal pulse. c. Monitor for signs of infection. d. Administer prescribed steroids.

ANS: A A grating sound heard when the affected part is moved is known as crepitation. This sound is created by bone fragments. Because bone fragments may be present, the nurse should immobilize the clients arm and tell the client not to move the arm. The grating sound does not indicate circulation impairment or infection. Steroids would not be indicated.

13. While assessing a client with Graves disease, the nurse notes that the clients temperature has risen 1 F. Which action should the nurse take first? a. Turn the lights down and shut the clients door. b. Call for an immediate electrocardiogram (ECG). c. Calculate the clients apical-radial pulse deficit. d. Administer a dose of acetaminophen (Tylenol).

ANS: A A temperature increase of 1 F may indicate the development of thyroid storm, and the provider needs to be notified. But before notifying the provider, the nurse should take measures to reduce environmental stimuli that increase the risk of cardiac complications. The nurse can then call for an ECG. The apical-radial pulse deficit would not be necessary, and Tylenol is not needed because the temperature increase is due to thyroid activity.

16. The nurse is planning teaching for a client who is starting acarbose for diabetes mellitus type 2. Which statement will the nurse include in the teaching? a. "Be sure to take the drug with each meal." b. "Take the drug every evening before bedtime." c. "Take the drug on an empty stomach in the morning." d. "Decide on the best day of the week to take the drug."

ANS: A Acarbose is an alpha-glucosidase inhibitor that works in the intestinal tract to prevent enzymes from breaking down starches into glucose. However, it must be taken with food at each meal, usually 3 times a day, to allow the drug to work as intended.

2. A nurse cares for a client with a deficiency of aldosterone. Which assessment finding should the nurse correlate with this deficiency? a. Increased urine output b. Vasoconstriction c. Blood glucose of 98 mg/dL d. Serum sodium of 144 mEq/L

ANS: A Aldosterone, the major mineralocorticoid, maintains extracellular fluid volume. It promotes sodium and water reabsorption and potassium excretion in the kidney tubules. A client with an aldosterone deficiency will have increased urine output. Vasoconstriction is not related. These sodium and glucose levels are normal; in aldosterone deficiency, the client would have hyponatremia and hyperkalemia.

2. A nurse is teaching a client newly diagnosed with osteoarthritis (OA) about drugs used to treat the disease. For which drug does the nurse plan health teaching? a. Acetaminophen b. Cyclobenzaprine hydrochloride c. Hyaluronate d. Ibuprofen

ANS: A All of these drugs may be appropriate to treat OA. However, the first-line drug is acetaminophen. Cyclobenzaprine is a muscle relaxant given to treat muscle spasms. Hyaluronate is a synthetic joint fluid implant. Ibuprofen is a nonsteroidal anti-inflammatory drug.

6. An older adult in the family practice clinic reports a decrease in hearing over a week. What action by the nurse is most appropriate? a. Assess for cerumen buildup. b. Facilitate audiological testing. c. Perform tuning fork tests. d. Review the medication list.

ANS: A All options are possible actions for the client with hearing loss. The first action the nurse should take is to look for cerumen buildup, which can decrease hearing in the older adult. If this is normal, medications should be assessed for ototoxicity. Further auditory testing may be needed for this client.

13. A nurse cares for a client admitted from a nursing home after several recent falls. What prescription should the nurse complete first? a. Obtain urine sample for culture and sensitivity. b. Administer intravenous antibiotics. c. Encourage protein intake and additional fluids. d. Consult physical therapy for gait training.

ANS: A Although all interventions are or might be important, obtaining a urine sample for urinalysis takes priority. Often urinary tract infection (UTI) symptoms in older adults are atypical, and a UTI may present with new onset of confusion or falling. The urine sample should be obtained before starting antibiotics. Dietary requirements and gait training should be implemented after obtaining the urine sample.

18. A nurse prepares a client for lumbar puncture (LP). Which assessment finding should alert the nurse to contact the health care provider? a. Shingles on the clients back b. Client is claustrophobic c. Absence of intravenous access d. Paroxysmal nocturnal dyspnea

ANS: A An LP should not be performed if the client has a skin infection at or near the puncture site because of the risk of infection. A nurse would want to notify the health care provider if shingles were identified on the clients back. If a client has shortness of breath when lying flat, the LP can be adapted to meet the clients needs. Claustrophobia, absence of IV access, and paroxysmal nocturnal dyspnea have no impact on whether an LP can be performed.

6. A nurse cares for a client with an increased blood urea nitrogen (BUN)/creatinine ratio. Which action should the nurse take first? a. Assess the clients dietary habits. b. Inquire about the use of nonsteroidal anti-inflammatory drugs (NSAIDs). c. Hold the clients metformin (Glucophage). d. Contact the health care provider immediately.

ANS: A An elevated BUN/creatinine ratio is often indicative of dehydration, urinary obstruction, catabolism, or a high- protein diet. The nurse should inquire about the clients dietary habits. Kidney damage related to NSAID use most likely would manifest with elevations in both BUN and creatinine, but no change in the ratio. The nurse should obtain more assessment data before holding any medications or contacting the provider.

2. A client had an embolic stroke and is having an echocardiogram. When the client asks why the provider ordered a test on my heart, how should the nurse respond? a. Most of these types of blood clots come from the heart. b. Some of the blood clots may have gone to your heart too. c. We need to see if your heart is strong enough for therapy. d. Your heart may have been damaged in the stroke too.

ANS: A An embolic stroke is caused when blood clots travel from one area of the body to the brain. The most common source of the clots is the heart. The other statements are inaccurate.

4. A clinic nurse is teaching a client prior to surgery. The client does not seem to comprehend the teaching, forgets a lot of what is said, and asks the same questions again and again. What action by the nurse is best? a. Assess the client for anxiety. b. Break the information into smaller bits. c. Give the client written information. d. Review the information again.

ANS: A Anxiety can interfere with learning and cooperation. The nurse should assess the client for anxiety. The other actions are appropriate too, and can be included in the teaching plan, but effective teaching cannot occur if the client is highly anxious.

12. The postanesthesia care unit (PACU) charge nurse notes vital signs on four postoperative clients. Which client would the nurse assess first? a. Client with a blood pressure of 100/50 mm Hg b. Client with a pulse of 118 beats/min c. Client with a respiratory rate of 6 breaths/min d. Client with a temperature of 96° F (35.6° C)

ANS: C The respiratory rate is the most important vital sign for any client who has undergone general anesthesia or moderate sedation, or has received opioid analgesia. This respiratory rate is too low and indicates respiratory depression. The nurse would assess this client first. A blood pressure of 100/50 mm Hg is slightly low and may be within that client's baseline. A pulse of 118 beats/min is slightly fast, which could be due to several causes, including pain and anxiety. A temperature of 96° F (35.6° C) is slightly low and the client needs to be warmed. But none of these other vital signs take priority over the respiratory rate.

1. A nurse prepares to teach a client who has experienced damage to the left temporal lobe of the brain. Which action should the nurse take when providing education about newly prescribed medications to this client? a. Help the client identify each medication by its color. b. Provide written materials with large print size. c. Sit on the clients right side and speak into the right ear. d. Allow the client to use a whiteboard to ask questions.

ANS: C The temporal lobe contains the auditory center for sound interpretation. The clients hearing will be impaired in the left ear. The nurse should sit on the clients right side and speak into the right ear. The other interventions do not address the clients left temporal lobe damage.

5. A client is distressed at body changes related to kyphosis. What response by the nurse is best? a. Ask the client to explain more about these feelings. b. Explain that these changes are irreversible. c. Offer to help select clothes to hide the deformity. d. Tell the client safety is more important than looks.

ANS: A Assessment is the first step of the nursing process, and the nurse should begin by getting as much information about the clients feelings as possible. Explaining that the changes are irreversible discounts the clients feelings. Depending on the extent of the deformity, clothing will not hide it. While safety is more objectively important than looks, the client is worried about looks and the nurse needs to address this issue.

28. After a stroke, a client has ataxia. What intervention is most appropriate to include on the clients plan of care? a. Ambulate only with a gait belt. b. Encourage double swallowing. c. Monitor lung sounds after eating. d. Perform post-void residuals.

ANS: A Ataxia is a gait disturbance. For the clients safety, he or she should have assistance and use a gait belt when ambulating. Ataxia is not related to swallowing, aspiration, or voiding.

13. A nurse is teaching a community group about preventing hearing loss. What instruction is best? a. Always wear a bicycle helmet. b. Avoid swimming in ponds or lakes. c. Dont go to fireworks displays. d. Use a soft cotton swab to clean ears.

ANS: A Avoiding head trauma is a practical way to help prevent hearing loss. Swimming can lead to hearing loss if the client has repeated infections. Fireworks displays are loud, but usually brief and only occasional. Nothing smaller than the clients fingertip should be placed in the ear canal.

10. A nurse is teaching a community group about preventing hearing loss. What instruction is appropriate? a. "Always wear a bicycle helmet." b. "Avoid swimming in ponds or lakes." c. "Don't attend fireworks shows." d. "Use a cerumen spoon to clean ears."

ANS: A Avoiding head trauma is a practical way to help prevent hearing loss. Swimming can lead to hearing loss only if the client has repeated infections. Fireworks displays/shows are loud, but usually brief and only occasional. A cerumen spoon is only used by primary health care providers to remove ear wax from in the ear canal.

2. A nurse is assessing an older client and discovers back pain with tenderness along T2 and T3. What action by the nurse is best? a. Consult with the provider about an x-ray. b. Encourage the client to use ibuprofen (Motrin). c. Have the client perform hip range of motion. d. Place the client in a rigid cervical collar.

ANS: A Back pain with tenderness is indicative of a spinal compression fracture, which is the most common type of osteoporotic fracture. The nurse should consult the provider about an x-ray. Motrin may be indicated but not until there is a diagnosis. Range of motion of the hips is not related, although limited spinal range of motion may be found with a vertebral compression fracture. Since the defect is in the thoracic spine, a cervical collar is not needed.

26. After teaching a client who is recovering from a vertebroplasty, the nurse assesses the clients understanding. Which statement by the client indicates a need for additional teaching? a. I can drive myself home after the procedure. b. I will monitor the puncture site for signs of infection. c. I can start walking tomorrow and increase my activity slowly. d. I will remove the dressing the day after discharge.

ANS: A Before discharge, a client who has a vertebroplasty should be taught to avoid driving or operating machinery for the first 24 hours. The client should monitor the puncture site for signs of infection. Usual activities can resume slowly, including walking and slowly increasing activity over the next few days. The client should keep the dressing dry and remove it the next day.

20. The postanesthesia care unit (PACU) nurse is caring for an older client following a lengthy surgery. The client's pulse is 48 beats/min which is 20 beats/min lower than the preoperative baseline. What assessment does the nurse make next? a. Temperature b. Level of consciousness c. Blood pressure d. Rate of IV infusion

ANS: A Bradycardia in the immediate postoperative client can indicate anesthesia effect or hypothermia. Older adults are at higher risk for hypothermia because of age-related changes in temperature regulation, decreased body fat, or prolonged exposure to cool environments, such as an OR suite. The nurse would first assess the client's temperature and take measures to correct any existing hypothermia. Level of consciousness, blood pressure, and IV infusion rate are not related, although all are important assessments in the postoperative period.

7. A nurse assesses a client recovering from a cerebral angiography via the clients right femoral artery. Which assessment should the nurse complete? a. Palpate bilateral lower extremity pulses. b. Obtain orthostatic blood pressure readings. c. Perform a funduscopic examination. d. Assess the gag reflex prior to eating.

ANS: A Cerebral angiography is performed by threading a catheter through the femoral or brachial artery. The extremity is kept immobilized after the procedure. The nurse checks the extremity for adequate circulation by noting skin color and temperature, presence and quality of pulses distal to the injection site, and capillary refill. Clients usually are on bedrest; therefore, orthostatic blood pressure readings cannot be performed. The funduscopic examination would not be affected by cerebral angiography. The client is given analgesics but not conscious sedation; therefore, the clients gag reflex would not be compromised.

1. A nurse is teaching a client about ear hygiene and health. What client statement indicates a need for further teaching? a. A soft cotton swab is alright to clean my ears with. b. I make sure my ears are dry after I go swimming. c. I use good earplugs when I practice with the band. d. Keeping my diabetes under control helps my ears.

ANS: A Clients should be taught not to put anything larger than their fingertip into their ears. Using a cotton swab, although soft, can cause damage to the ears and cerumen buildup. The other statements are accurate.

1. The nurse has given a community group a presentation on eye health. Which statement by a participant indicates a need for more instruction? a. I always lose my sunglasses, so I dont wear them. b. I have diabetes and get an annual eye exam. c. I will not share my contact solution with others. d. I will wear safety glasses when I mow the lawn.

ANS: A Clients should be taught to protect their eyes from ultraviolet (UV) exposure by consistently wearing sunglasses when outdoors, when tanning in tanning salons, or when working with UV light. The other statements are correct.

3. A client who has multiple sclerosis reports increased severe muscle spasticity and tremors. What nursing action is most appropriate to manage this client's concern? a. Request a prescription for an antispasmodic drug such as baclofen. b. Prepare the client for deep brain stimulation surgery. c. Refer the client to a massage therapist to relax the muscles. d. Consult with the occupational therapist for self-care assistance.

ANS: A Clients who have multiple sclerosis often have muscle spasticity which may be reduced by drug therapy, such as baclofen. While massage and assistance with self-care may be helpful, these interventions are not the most effective and therefore not the most appropriate in managing muscle spasticity. If drug therapy and other interventions do not help reduce muscle spasms, some client are candidates for deep brain stimulation as a last resort.

26.A preoperative nurse assesses a client who has type 1 diabetes mellitus prior to a surgical procedure. The clients blood glucose level is 160 mg/dL. Which action should the nurse take? a. Document the finding in the clients chart. b. Administer a bolus of regular insulin IV. c. Call the surgeon to cancel the procedure. d. Draw blood gases to assess the metabolic state.

ANS: A Clients who have type 1 diabetes and are having surgery have been found to have fewer complications, lower rates of infection, and better wound healing if blood glucose levels are maintained at between 140 and 180 mg/dL throughout the perioperative period. The nurse should document the finding and proceed with other operative care. The need for a bolus of insulin, canceling the procedure, or drawing arterial blood gases is not required.

21. A nurse delegates care for a client with Parkinson disease to an unlicensed assistive personnel (UAP). Which statement should the nurse include when delegating this clients care? a. Allow the client to be as independent as possible with activities. b. Assist the client with frequent and meticulous oral care. c. Assess the clients ability to eat and swallow before each meal. d. Schedule appointments early in the morning to ensure rest in the afternoon.

ANS: A Clients with Parkinson disease do not move as quickly and can have functional problems. The client should be encouraged to be as independent as possible and provided time to perform activities without rushing. Although oral care is important for all clients, instructing the UAP to provide frequent and meticulous oral is not a priority for this client. This statement would be a priority if the client was immune-compromised or NPO. The nurse should assess the clients ability to eat and swallow; this should not be delegated. Appointments and activities should not be scheduled early in the morning because this may cause the client to be rushed and discourage the client from wanting to participate in activities of daily living.

6. The nurse is teaching a family caregiver about how best to communicate with the client who has been diagnosed with Alzheimer disease. Which statement by the caregiver indicates a need for further teaching? a. "I will avoid communicating with the client to prevent agitation." b. "I should use simple, short sentences and one-step instructions." c. "I can try to use gestures or pictures to communicate with the client." d. "I will limit the number of choices I provide for the client."

ANS: A Communication with the client is important to provide cognitive stimulation. Using short simple sentences, using gestures and pictures, and limiting choices provided for the client will help promote communication.

4. A client presents to the emergency department reporting a foreign body in the eye. For what diagnostic testing should the nurse prepare the client? a. Corneal staining b. Fluorescein angiography c. Ophthalmoscopy d. Tonometry

ANS: A Corneal staining is used when the possibility of eye trauma exists, including a foreign body. Fluorescein angiography is used to assess problems of retinal circulation. Ophthalmoscopy looks at both internal and external eye structures. Tonometry tests the intraocular pressure.

13. A client who has rheumatoid arthritis is prescribed etanercept. What health teaching by the nurse about this drug is appropriate? a. Giving subcutaneous injections b. Having a chest x-ray once a year c. Taking the medication with food d. Using heat on the injection site

ANS: A Etanercept is given as a subcutaneous injection twice a week. The nurse would teach the client how to self-administer the medication. The other options are not appropriate for etanercept.

13. A nurse cares for a client with chronic hypercortisolism. Which action should the nurse take? a. Wash hands when entering the room. b. Keep the client in airborne isolation. c. Observe the client for signs of infection. d. Assess the clients daily chest x-ray.

ANS: A Excess cortisol reduces the number of circulating lymphocytes, inhibits maturation of macrophages, reduces antibody synthesis, and inhibits production of cytokines and inflammatory chemicals. As a result, these clients are at greater risk of infection and may not have the expected inflammatory manifestations when an infection is present. The nurse needs to take precautions to decrease the clients risk. It is not necessary to keep the client in isolation. The client does not need a daily chest x-ray.

1. A nurse promotes the prevention of lower back pain by teaching clients at a community center. Which instruction should the nurse include in this education? a. Participate in an exercise program to strengthen muscles. b. Purchase a mattress that allows you to adjust the firmness. c. Wear flat instead of high-heeled shoes to work each day. d. Keep your weight within 20% of your ideal body weight.

ANS: A Exercise can strengthen back muscles, reducing the incidence of low back pain. The other options will not prevent low back pain.

3. A client has been advised to perform weight-bearing exercises to help minimize osteoporosis. The client admits to not doing the prescribed exercises. What action by the nurse is best? a. Ask the client about fear of falling. b. Instruct the client to increase calcium. c. Suggest other exercises the client can do. d. Tell the client to try weight lifting.

ANS: A Fear of falling can limit participation in activity. The nurse should first assess if the client has this fear and then offer suggestions for dealing with it. The client may or may not need extra calcium, other exercises, or weight lifting.

5. A nurse evaluates a client with acute glomerulonephritis (GN). Which manifestation should the nurse recognize as a positive response to the prescribed treatment? a. The client has lost 11 pounds in the past 10 days. b. The clients urine specific gravity is 1.048. c. No blood is observed in the clients urine. d. The clients blood pressure is 152/88 mm Hg.

ANS: A Fluid retention is a major feature of acute GN. This weight loss represents fluid loss, indicating that the glomeruli are performing the function of filtration. A urine specific gravity of 1.048 is high. Blood is not usually seen in GN, so this finding would be expected. A blood pressure of 152/88 mm Hg is too high; this may indicate kidney damage or fluid overload.

6. A nurse cares for adult clients who experience urge incontinence. For which client should the nurse plan a habit training program? a. A 78-year-old female who is confused b. A 65-year-old male with diabetes mellitus c. A 52-year-old female with kidney failure d. A 47-year-old male with arthritis

ANS: A For a bladder training program to succeed in a client with urge incontinence, the client must be alert, aware, and able to resist the urge to urinate. Habit training will work best for a confused client. This includes going to the bathroom (or being assisted to the bathroom) at set times. The other clients may benefit from another type of bladder training.

6. A client had an embolectomy for an arteriovenous malformation (AVM). The client is now reporting a severe headache and has vomited. What action by the nurse takes priority? a. Administer pain medication. b. Assess the clients vital signs. c. Notify the Rapid Response Team. d. Raise the head of the bed.

ANS: C This client may be experiencing a rebleed from the AVM. The most important action is to call the Rapid Response Team as this is an emergency. The nurse can assess vital signs while someone else notifies the Team, but getting immediate medical attention is the priority. Administering pain medication may not be warranted if the client must return to surgery. The optimal position for the client with an AVM has not been determined, but calling the Rapid Response Team takes priority over positioning.

8. A client's intraocular pressure (IOP) is 28 mm Hg. What action would the nurse anticipate? a. Educate the client on corneal transplantation. b. Facilitate scheduling the eye surgery. c. Teach about drugs for glaucoma. d. Refer the patient to local Braille classes.

ANS: C This increased IOP indicates glaucoma. The nurse's main responsibility is teaching the client about drug therapy. Corneal transplantation is not used in glaucoma. Eye surgery is not indicated at this time. Braille classes are also not indicated at this time.

10. A nurse cares for a client who is prescribed a 24-hour urine collection. The unlicensed assistive personnel (UAP) reports that, while pouring urine into the collection container, some urine splashed his hand. Which action should the nurse take next? a. Ask the UAP if he washed his hands afterward. b. Have the UAP fill out an incident report. c. Ask the laboratory if the container has preservative in it. d. Send the UAP to Employee Health right away.

ANS: A For safety, the nurse should find out if the UAP washed his or her hands. The UAP should do this for two reasons. First, it is part of Standard Precautions to wash hands after client care. Second, if the container did have preservative in it, this would wash it away. The preservative may be caustic to the skin. The nurse can call the laboratory while the UAP is washing hands, if needed. The UAP would then need to fill out an incident or exposure report and may or may not need to go to Employee Health. The UAP also needs further education on Standard Precautions, which include wearing gloves.

16. What information does the nurse teach a womens group about osteoporosis? a. For 5 years after menopause you lose 2% of bone mass yearly. b. Men actually have higher rates of the disease but are underdiagnosed. c. There is no way to prevent or slow osteoporosis after menopause. d. Women and men have an equal chance of getting osteoporosis.

ANS: A For the first 5 years after menopause, women lose about 2% of their bone mass each year. Men have a slower loss of bone after the age of 75. Many treatmeNntUsRaSreINnGowTBa.vCaOilMable for women to slow osteoporosis after menopause.

12. A client is taking furosemide (Lasix) 40 mg/day for management of chronic kidney disease (CKD). To detect the positive effect of the medication, what action of the nurse is best? a. Obtain daily weights of the client. b. Auscultate heart and breath sounds. c. Palpate the clients abdomen. d. Assess the clients diet history.

ANS: A Furosemide (Lasix) is a loop diuretic that helps reduce fluid overload and hypertension in clients with early stages of CKD. One kilogram of weight equals about 1 liter of fluid retained in the client, so daily weights are necessary to monitor the response of the client to the medication. Heart and breath sounds should be assessed if there is fluid retention, as in heart failure. Palpation of the clients abdomen is not necessary, but the nurse should check for edema. The diet history of the client would be helpful to assess electrolyte replacement since potassium is lost with this diuretic, but this does not assess the effect of the medication.

9. The clients chart indicates genu varum. What does the nurse understand this to mean? a. Bow-legged b. Fluid accumulation c. Knock-kneed d. Spinal curvature

ANS: A Genu varum is a bow-legged deformity. A fluid accumulation is an effusion. Genu valgum is knock-kneed. A spinal curvature could be kyphosis or lordosis.

6.A nurse teaches a client who is diagnosed with diabetes mellitus. Which statement should the nurse include in this clients plan of care to delay the onset of microvascular and macrovascular complications? a. Maintain tight glycemic control and prevent hyperglycemia. b. Restrict your fluid intake to no more than 2 liters a day. c. Prevent hypoglycemia by eating a bedtime snack. d. Limit your intake of protein to prevent ketoacidosis.

ANS: A Hyperglycemia is a critical factor in the pathogenesis of long-term diabetic complications. Maintaining tight glycemic control will help delay the onset of complications. Restricting fluid intake is not part of the treatment plan for clients with diabetes. Preventing hypoglycemia and ketosis, although important, are not as important as maintaining daily glycemic control.

8. A client in the intensive care unit is started on continuous venovenous hemofiltration (CVVH). Which finding is the cause of immediate action by the nurse? a. Blood pressure of 76/58 mm Hg b. Sodium level of 138 mEq/L c. Potassium level of 5.5 mEq/L d. Pulse rate of 90 beats/min

ANS: A Hypotension can be a problem with CVVH if replacement fluid does not provide enough volume to maintain blood pressure. The specially trained nurse needs to monitor for ongoing fluid and electrolyte replacement. The sodium level is normal and the potassium level is slightly elevated, which could be normal findings for someone with acute kidney injury. A pulse rate of 90 beats/min is normal.

15. A nurse performs an assessment of pain discrimination on an older adult client. The client correctly identifies, with eyes closed, a sharp sensation on the right hand when touched with a pin. Which action should the nurse take next? a. Touch the pin on the same area of the left hand. b. Contact the provider with the assessment results. c. Ask the client about current medications. d. Continue the assessment on the clients feet.

ANS: A If testing is begun on the right hand and the client correctly identifies the pain stimulus, the nurse should continue the assessment on the left hand. This is a normal finding and does not need to be reported to the provider, but instead documented in the clients chart. Medications do not need to be assessed in response to this finding. The nurse should assess the left hand prior to assessing the feet.

24.After teaching a client who is recovering from pancreas transplantation, the nurse assesses the clients understanding. Which statement made by the client indicates a need for additional education? a. If I develop an infection, I should stop taking my corticosteroid. b. If I have pain over the transplant site, I will call the surgeon immediately. c. I should avoid people who are ill or who have an infection. d. I should take my cyclosporine exactly the way I was taught.

ANS: A Immunosuppressive agents should not be stopped without the consultation of the transplantation physician, even if an infection is present. Stopping immunosuppressive therapy endangers the transplanted organ. The other statements are correct. Pain over the graft site may indicate rejection. Anti-rejection drugs cause immunosuppression, and the client should avoid crowds and people who are ill. Changing the routine of anti- rejection medications may cause them to not work optimally.

18.An emergency department nurse assesses a client with ketoacidosis. Which clinical manifestation should the nurse correlate with this condition? a. Increased rate and depth of respiration b. Extremity tremors followed by seizure activity c. Oral temperature of 102 F (38.9 C) d. Severe orthostatic hypotension

ANS: A Ketoacidosis decreases the pH of the blood, stimulating the respiratory control areas of the brain to buffer the effects of increasing acidosis. The rate and depth of respiration are increased (Kussmaul respirations) in an attempt to excrete more acids by exhalation. Tremors, elevated temperature, and orthostatic hypotension are not associated with ketoacidosis.

5. After teaching a client who has stress incontinence, the nurse assesses the clients understanding. Which statement made by the client indicates a need for additional teaching? a. I will limit my total intake of fluids. b. I must avoid drinking alcoholic beverages. c. I must avoid drinking caffeinated beverages. d. I shall try to lose about 10% of my body weight.

ANS: A Limiting fluids concentrates urine and can irritate tissues, leading to increased incontinence. Many people try to manage incontinence by limiting fluids. AlcNoUhRolSicINaGndTBca.CffOeiMnated beverages are bladder stimulants. Obesity increases intra-abdominal pressure, causing incontinence.

15. A client has labyrinthitis and is prescribed antibiotics. What instruction by the nurse is most important for this client? a. Immediately report headache or stiff neck. b. Keep all follow-up appointments. c. Take the antibiotics with a full glass of water. d. Take the antibiotic on an empty stomach.

ANS: A Meningitis is a complication of labyrinthitis. The client should be taught to take all antibiotics as prescribed and to report manifestations of meningitis such as fever, headache, or stiff neck. Keeping follow-up appointments is important for all clients. Without knowing what antibiotic was prescribed, the nurse cannot instruct the client on how to take it.

9. A nurse reviews prescriptions for an 82-year-old client with a fractured left hip. Which prescription should alert the nurse to contact the provider and express concerns for client safety? a. Meperidine (Demerol) 50 mg IV every 4 hours b. Patient-controlled analgesia (PCA) with morphine sulfate c. Percocet 2 tablets orally every 6 hours PRN for pain d. Ibuprofen elixir every 8 hours for first 2 days

ANS: A Meperidine (Demerol) should not be used for older adults because it has toxic metabolites that can cause seizures. The nurse should question this prescription. The other prescriptions are appropriate for this clients pain management.

13. A nurse cares for a client with diabetes mellitus who is prescribed metformin (Glucophage) and is scheduled for an intravenous urography. Which action should the nurse take first? a. Contact the provider and recommend discontinuing the metformin. b. Keep the client NPO for at least 6 hours prior to the examination. c. Check the clients capillary artery blood glucose and administer prescribed insulin. d. Administer intravenous fluids to dilute and increase the excretion of dye.

ANS: A Metformin can cause lactic acidosis and renal impairment as the result of an interaction with the dye. This drug must be discontinued for 48 hours before the procedure and not started again after the procedure until urine output is well established. The clients health care provider needs to provide alternative therapy for the client until the metformin can be resumed. Keeping the client NPO, checking the clients blood glucose, and administering intravenous fluids should be part of the clients plan of care, but are not the priority, as the examination should not occur while the client is still taking metformin.

5. A nurse contacts the health care provider after reviewing a clients laboratory results and noting a blood urea nitrogen (BUN) of 35 mg/dL and a creatinine of 1.0 mg/dL. For which action should the nurse recommend a prescription? a. Intravenous fluids b. Hemodialysis c. Fluid restriction d. Urine culture and sensitivity

ANS: A Normal BUN is 10 to 20 mg/dL. Normal creatinine is 0.6 to 1.2 mg/dL (males) or 0.5 to 1.1 mg/dL (females). Creatinine is more specific for kidney function than BUN, because BUN can be affected by several factors (dehydration, high-protein diet, and catabolism). This clients creatinine is normal, which suggests a non-renal cause for the elevated BUN. A common cause of increased BUN is dehydration, so the nurse should anticipate giving the client more fluids, not placing the client on fluid restrictions. Hemodialysis is not an appropriate treatment for dehydration. The lab results do not indicate an infection; therefore, a urine culture and sensitivity is not appropriate.

12. After teaching a client with a spinal cord injury, the nurse assesses the clients understanding. Which client statement indicates a correct understanding of how to prevent respiratory problems at home? a. Ill use my incentive spirometer every 2 hours while Im awake. b. Ill drink thinned fluids to prevent choking. c. Ill take cough medicine to prevent excessive coughing. d. Ill position myself on my right side so I dont aspirate.

ANS: A Often, the person with a spinal cord injury will have weak intercostal muscles and is at higher risk for developing atelectasis and stasis pneumonia. Using an incentive spirometer every 2 hours helps the client expand the lungs more fully and prevents atelectasis. Clients should drink fluids that they can tolerate; usually thick fluids are easier to tolerate. The client should be encouraged to cough and clear secretions. Clients should be placed in high-Fowlers position to prevent aspiration.

1. The nurse is teaching a group of older adults about basic eye examinations. What would the nurse recommend about the frequency for eye examinations for most people over 65 years of age? a. Every 1 to 2 years b. Every 2 to 4 years c. Every 3 to 5 years d. When the primary health care provider recommends

ANS: A Older adults need more frequent basic eye examinations due to the increased risk of glaucoma and cataracts associated with aging. Therefore, every 1 to 2 years for eye examination in the current best practice recommendation.

9. The nurse is caring for a 60-year-old female client who sustained a thoracic spinal cord injury 10 years ago. For which potential complication will the nurse assess during this client's care? a. Fracture b. Malabsorption c. Delirium d. Anemia

ANS: A Older adults who have impaired mobility due to a health problem or injury are at risk for complications of immobility, such as osteoporosis (bone loss) which leads to fracture. Being an older woman increases that risk due to loss of estrogen to protect bone loss. The other choices are not problems of immobility. Delirium is possible but is more common in clients over 70 years of age.

10. A client has a metastatic bone tumor in the left leg. What action by the nurse is appropriate? a. Administer pain medication as prescribed. b. Elevate the extremity and apply moist heat. c. Teach the client about amputation care. d. Place the client on protective precautions.

ANS: A Pain medication should be given to control metastatic bone pain. Elevation and heat may or may not be helpful. Protective precautions are not needed for this client.

13. A client is in the clinic for a follow-up visit after a moderate traumatic brain injury. The clients spouse is very frustrated, stating that the clients personality has changed and the situation is intolerable. What action by the nurse is best? a. Explain that personality changes are common following brain injuries. b. Ask the client why he or she is acting out and behaving differently. c. Refer the client and spouse to a head injury support group. d. Tell the spouse this is expected and he or she will have to learn to cope.

ANS: A Personality and behavior often change permanently after head injury. The nurse should explain this to the spouse. Asking the client about his or her behavior isnt useful because the client probably cannot help it. A referral might be a good idea, but the nurse needs to do something in addition to just referring the couple. Telling the spouse to learn to cope belittles the spouses concerns and feelings.

6. A nurse assesses a client who is recovering from anterior cervical diskectomy and fusion. Which complication should alert the nurse to urgently communicate with the health care provider? a. Auscultated stridor b. Weak pedal pulses c. Difficulty swallowing d. Inability to shrug shoulders

ANS: A Postoperative swelling can narrow the trachea, cause a partial airway obstruction, and manifest as stridor. The client may also have trouble swallowing, but maintaining an airway takes priority. Weak pedal pulses and an inability to shrug the shoulders are not complications of this surgery.

15. A 70-kg adult with chronic renal failure is on a 40-g protein diet. The client has a reduced glomerular filtration rate and is not undergoing dialysis. Which result would give the nurse the most concern? a. Albumin level of 2.5 g/dL b. Phosphorus level of 5 mg/dL c. Sodium level of 135 mmol/L d. Potassium level of 5.5 mmol/L

ANS: A Protein restriction is necessary with chronic renal failure due to the buildup of waste products from protein breakdown. The nurse would be concerned with the low albumin level since this indicates that the protein in the diet is not enough for the clients metabolic needs. The electrolyte values are not related to the protein- restricted diet.

17. A client is scheduled to have a tumor of the middle ear removed. What teaching topic is most important for the nurse to cover? a. Expecting hearing loss in the affected ear b. Managing postoperative pain c. Maintaining NPO status prior to surgery d. Understanding which medications are allowed the day of surgery

ANS: A Removal of an inner ear tumor will likely destroy hearing in the affected ear. The other teaching topics are appropriate for any surgical client.

5.A nurse cares for a client who has a family history of diabetes mellitus. The client states, My father has type 1 diabetes mellitus. Will I develop this disease as well? How should the nurse respond? a. Your risk of diabetes is higher than the general population, but it may not occur. b. No genetic risk is associated with the development of type 1 diabetes mellitus. c. The risk for becoming a diabetic is 50% because of how it is inherited. d. Female children do not inherit diabetes mellitus, but male children will.

ANS: A Risk for type 1 diabetes is determined by inheritance of genes coding for HLA-DR and HLA-DQ tissue types. Clients who have one parent with type 1 diabetes are at increased risk for its development. Diabetes (type 1) seems to require interaction between inherited risk and environmental factors, so not everyone with these genes develops diabetes. The other statements are not accurate.

14. A nurse cares for a client placed in skeletal traction. The client asks, What is the primary purpose of this type of traction? How should the nurse respond? a. Skeletal traction will assist in realigning your fractured bone. b. This treatment will prevent future complications and back pain. c. Traction decreases muscle spasms that occur with a fracture. d. This type of traction minimizes damage as a result of fracture treatment.

ANS: A Skeletal traction pins or screws are surgically inserted into the bone to aid in bone alignment. As a last resort, traction can be used to relieve pain, decrease muscle spasm, and prevent or correct deformity and tissue damage. These are not primary purposes of skeletal traction.

16. A nurse obtains the health history of a client with a suspected diagnosis of bladder cancer. Which question should the nurse ask when determining this clients risk factors? a. Do you smoke cigarettes? b. Do you use any alcohol? c. Do you use recreational drugs? d. Do you take any prescription drugs?

ANS: A Smoking is known to be a factor that greatly increases the risk of bladder cancer. Alcohol use, recreational drug use, and prescription drug use (except medications that contain phenacetin) are not known to increase the risk of developing bladder cancer.

12. A client hospitalized for a wound infection has a blood urea nitrogen of 45 mg/dL and creatinine of 4.2 mg/dL. What action by the nurse is best? a. Assess the ordered antibiotics for ototoxicity. b. Explain how kidney damage causes hearing loss. c. Use ibuprofen (Motrin) for pain control. d. Teach that hearing loss is temporary.

ANS: A Some medications are known to be ototoxic. Diminished kidney function slows the excretion of drugs from the body, worsening the ototoxic effects. The nurse should assess the antibiotics the client is receiving for ototoxicity. The other options are not warranted.

25. A client has a shoulder injury and is scheduled for a magnetic resonance imaging (MRI). The nurse notes the presence of an aneurysm clip in the clients record. What action by the nurse is best? a. Ask the client how long ago the clip was placed. b. Have the client sign an informed consent form. c. Inform the provider about the aneurysm clip. d. Reschedule the client for computed tomography.

ANS: A Some older clips are metal, which would preclude the use of MRI. The nurse should determine how old the clip is and relay that information to the MRI staff. They can determine if the client is a suitable candidate for this examination. The client does not need to sign informed consent. The provider will most likely not know if the client can have an MRI with this clip. The nurse does not independently change the type of diagnostic testing the client receives.

12. A client with hyperaldosteronism is being treated with spironolactone (Aldactone) before surgery. Which precautions does the nurse teach this client? a. Read the label before using salt substitutes. b. Do not add salt to your food when you eat. c. Avoid exposure to sunlight. d. Take Tylenol instead of aspirin for pain.

ANS: A Spironolactone is a potassium-sparing diuretic used to control potassium levels. Its use can lead to hyperkalemia. Although the goal is to increase the clients potassium, unknowingly adding potassium can cause complications. Some salt substitutes are composed of potassium chloride and should be avoided by clients on spironolactone therapy. Depending on the client, he or she may benefit from a low-sodium diet before surgery, but this may not be necessary. Avoiding sunlight and Tylenol is not necessary.

4. A nurse assesses a client who is prescribed a medication that stimulates beta1 receptors. Which assessment finding should alert the nurse to urgently contact the health care provider? a. Heart rate of 50 beats/min b. Respiratory rate of 18 breaths/min c. Oxygenation saturation of 92% d. Blood pressure of 144/69 mm Hg

ANS: A Stimulation of beta1 receptor sites in the heart has positive chronotropic and inotropic actions. The nurse expects an increase in heart rate and increased cardiac output. The client with a heart rate of 50 beats/min would be cause for concern because this would indicate that the client was not responding to the medication. The other vital signs are within normal limits and do not indicate a negative response to the medication.

4. A nurse assesses a client who is prescribed a medication that stimulates beta1 receptors. Which assessment finding would indicate that the medication is effective? a. Heart rate of 92 beats/min b. Respiratory rate of 18 breaths/min c. Oxygenation saturation of 92% d. Blood pressure of 144/69 mm Hg

ANS: A Stimulation of beta1 receptor sites in the heart has positive chronotropic and inotropic actions. The nurse expects an increase in heart rate and increased cardiac output. The other vital signs are within normal limits and do not indicate any response to the medication.

6. A client in a nursing home refuses to take medications. She is at high risk for osteomalacia. What action by the nurse is best? a. Ensure the client gets 15 minutes of sun exposure daily. b. Give the client daily vitamin D injections. c. Hide vitamin D supplements in favorite foods. d. Plan to serve foods naturally high in vitamin D.

ANS: A Sunlight is a good source of vitamin D, and the nursing staff can ensure some sun exposure each day. Vitamin D is not given by injection. Hiding the supplement in food is unethical. Very few foods are naturally high in vitamin D, but some are supplemented.

9. The perioperative nurse manager and the postoperative unit manager are concerned about the increasing number of surgical infections in their hospital. What action by the managers is best? a. Audit charts to see if the Surgical Care Improvement Project (SCIP) outcomes were met. b. Encourage staff on both units to provide peer pressure to adhere to hand hygiene policy. c. Hold educational meetings with the nursing and surgical staff on infection prevention. d. Monitor staff on both units for consistent adherence to established hand hygiene practices.

ANS: A The SCIP project contains core measures to reduce surgical complications. Examples of focus included administration of prophylactic antibiotics, correct hair removal processes, the timing of discontinuation of urinary catheterization after surgery, and venous thromboembolism prophylaxis. These practices are now standard in surgical care. Prevention of infection is a heavy emphasis, so the managers would start by reviewing charts to see if the guidelines of this project were implemented. The other actions may be necessary too, but first the managers need to assess the situation.

13.After teaching a client with diabetes mellitus to inject insulin, the nurse assesses the clients understanding. Which statement made by the client indicates a need for additional teaching? a. The lower abdomen is the best location because it is closest to the pancreas. b. I can reach my thigh the best, so I will use the different areas of my thighs. c. By rotating the sites in one area, my chance of having a reaction is decreased. d. Changing injection sites from the thigh to the arm will change absorption rates.

ANS: A The abdominal site has the fastest rate of absorption because of blood vessels in the area, not because of its proximity to the pancreas. The other statements are accurate assessments of insulin administration.

17. The nurse is teaching a client how to use a cane after a right surgical fractured fibula repair. What health teaching would the nurse include? a. "Place the cane on your left side." b. "Move the cane and your left leg at the same time." c. "Be sure the cane is parallel to your waist." d. "Use the cane only when your right leg is painful."

ANS: A The cane should be placed on tGheRuAnDafEfeScLteAdBsi.deC(OleMft for this client) and moved forward with the injured leg (right for this client) to provide support. The cane should be parallel to the stylus of the wrist and used at all times when ambulating.

7. A client had a myringotomy. The nurse provides which discharge teaching? a. Buy dry shampoo to use for a week. b. Drink liquids through a straw. c. Flying is not allowed for 1 month. d. Hot water showers will help the pain.

ANS: A The client cannot shower or get the head wet for 1 week after surgery, so using dry shampoo is a good suggestion. The other instructions are incorrect: straws are not allowed for 2 to 3 weeks, flying is not allowed for 2 to 3 weeks, and the client should not shower.

5. A nurse assesses an older adult client who was admitted 2 days ago with a fractured hip. The nurse notes that the client is confused and restless. The clients vital signs are heart rate 98 beats/min, respiratory rate 32 breaths/min, blood pressure 132/78 mm Hg, and SpO2 88%. Which action should the nurse take first? a. Administer oxygen via nasal cannula. b. Re-position to a high-Fowlers position. c. Increase the intravenous flow rate. d. Assess response to pain medications.

ANS: A The client is at high risk for a fat embolism and has some of the clinical manifestations of altered mental status and dyspnea. Although this is a life-threatening emergency, the nurse should take the time to administer oxygen first and then notify the health care provider. Oxygen administration can reduce the risk for cerebral damage from hypoxia. The nurse would not restrain a client who is confused without further assessment and orders. Sitting the client in a high-Fowlers position will not decrease hypoxia related to a fat embolism. The IV rate is not related. Pain medication most likely would not cause the client to be restless.

21. A nurse assesses a client and notes the clients position as indicated in the illustration below: How should the nurse document this finding? a. Decorticate posturing b. Decerebrate posturing c. Atypical hyperreflexia d. Spinal cord degeneration

ANS: A The client is demonstrating decorticate posturing, which is seen with interruption in the corticospinal pathway. This finding is abnormal and is a sign that the clients condition has deteriorated. The physician, the charge nurse, and other health care team members should be notified immediately of this change in status. Decerebrate posturing consists of external rotation and extension of the extremities. Hyperreflexes present as increased reflex responses. Spinal cord degeneration presents frequently with pain and discomfort.

20. A nurse cares for a client who is recovering from a single-photon emission computed tomography (SPECT) with a radiopharmaceutical agent. Which statement should the nurse include when discussing the plan of care with this client? a. You may return to your previous activity level immediately. b. You are radioactive and must use a private bathroom. c. Frequent assessments of the injection site will be completed. d. We will be monitoring your renal functions closely.

ANS: A The client may return to his or her previous activity level immediately. Radioisotopes will be eliminated in the urine after SPECT, but no monitoring or special precautions are required. The injection site will not need to be assessed after the procedure is complete.

14. A nurse teaches a young female client who is prescribed cephalexin for a urinary tract infection. Which statement would the nurse include in this client's teaching? a. "Use a second form of birth control while on this medication." b. "You will experience increased menstrual bleeding while on this drug." c. "You may experience an irregular heartbeat while on this drug." d. "Watch for blood in your urine while taking this medication."

ANS: A The client should use a second form of birth control because antibiotic therapy reduces the effectiveness of estrogen-containing contraceptives. She should not experience increased menstrual bleeding, an irregular heartbeat, or blood in her urine while taking the drug.

18. A nurse teaches a young female client who is prescribed amoxicillin (Amoxil) for a urinary tract infection. Which statement should the nurse include in this clients teaching? a. Use a second form of birth control while on this medication. b. You will experience increased menstrual bleeding while on this drug. c. You may experience an irregular heartbeat while on this drug. d. Watch for blood in your urine while taking this medication.

ANS: A The client should use a second form of birth control because penicillin seems to reduce the effectiveness of estrogen-containing contraceptives. She should not experience increased menstrual bleeding, an irregular heartbeat, or blood in her urine while taking the medication.

3. The nurse is caring for a client who has acromegaly. What physical change would the nurse expect to observe? a. Large hands and face b. Thin, dry skin c. Short height d. Truncal obesity

ANS: A The client who has acromegaly has an excess of growth hormone as an adult and therefore has a large musculoskeletal structure that is readily observed.

5. A nurse assesses a client who demonstrates a positive Rombergs sign with eyes closed but not with eyes open. Which condition does the nurse associate with this finding? a. Difficulty with proprioception b. Peripheral motor disorder c. Impaired cerebellar function d. Positive pronator drift

ANS: A The client who sways with eyes closed (positive Rombergs sign) but not with eyes open most likely has a disorder of proprioception and uses vision to compensate for it. The other options do not describe a positive Rombergs sign.

21. A client with chronic kidney disease (CKD) has an elevated serum phosphorus level. What drug would the nurse anticipate to be prescribed for this client? a. Calcium acetate b. Doxycyline c. Magnesium sulfate d. Lisinopril

ANS: A The client with CKD often has a high phosphorus level which tends to lower the calcium level in an inverse relationship, and causes osteodystrophy. To prevent this bone disease, the client needs to take a drug that can bind with phosphorus for elimination via the GI tract. When phosphorus is lowered to within normal limits, normal calcium levels may be restored.

11. A nurse is reviewing care for a client who has syndrome of inappropriate antidiuretic hormone (SIADH) with assistive personnel. What statement by the AP indicates understanding of this client's care? a. "I will weigh the client carefully before breakfast and compare with yesterday's weight." b. "I will encourage plenty of fluids to promote urination and prevent dehydration." c. "I will teach the client not to select high-sodium or salty foods on the menu." d. "I will assess the client's mucous membranes and skin for signs of dehydration."

ANS: A The client with SIADH usually has a fluid restriction, not an increase in fluids. It is the role of the RN rather than AP to perform assessments and provide health teaching. The AP needs to weigh the client daily and report a significant weight changes.

16. A client is getting out of bed into the chair for the first time after an uncemented total hip arthroplasty. What action by the nurse is appropriate? a. Have adequate help to transfer the patient. b. Provide socks so the patient can slide easier. c. Tell the patient full weight bearing is allowed. d. Use a footstool to elevate the patient's leg.

ANS: A The client with an uncemented hip will be on toe-touch only after surgery. The nurse would ensure there is adequate help to transfer the patient while preventing falls. Slippery socks may cause a fall. Elevating the leg is not going to assist with the client's transfer.

14.A nurse assesses a client with diabetes mellitus and notes the client only responds to a sternal rub by moaning, has capillary blood glucose of 33 g/dL, and has an intravenous line that is infiltrated with 0.45% normal saline. Which action should the nurse take first? a. Administer 1 mg of intramuscular glucagon. b. Encourage the client to drink orange juice. c. Insert a new intravenous access line. d. Administer 25 mL dextrose 50% (D50) IV push.

ANS: A The clients blood glucose level is dangerously low. The nurse needs to administer glucagon IM immediately to increase the clients blood glucose level. The nurse should insert a new IV after administering the glucagon and can use the new IV site for future doses of D50 if the clients blood glucose level does not rise. Once the client is awake, orange juice may be administered orally along with a form of protein such as a peanut butter.

13. A client had a bunionectomy with osteotomy. The client asks why healing may take up to 3 months. What explanation by the nurse is best? a. Your feet have less blood flow, so healing is slower. b. The bones in your feet are hard to operate on. c. The surrounding bones and tissue are damaged. d. Your feet bear weight so they never really heal.

ANS: A The feet are the most distal to the heart and receive less blood flow than other organs and tissues, prolonging the healing time after surgery. The other explanations are not correct.

12. A client with bone cancer is hospitalized for a limb salvage procedure. How can the nurse best address the clients psychosocial needs? a. Assess the clients coping skills and support systems. b. Explain that the surgery leads to a longer life expectancy. c. Refer the client to the social worker or hospital chaplain. d. Reinforce physical therapy to aid with ambulating normally.

ANS: A The first step in the nursing process is assessment. The nurse should assess coping skills and possible support systems that will be helpful in this clients treatment. Explaining that a limb salvage procedure will extend life does not address the clients psychosocial needs. Referrals may be necessary, but the nurse should assess first. Reinforcing physical therapy is also helpful but again does not address the psychosocial needs of the client.

11. A client with bone cancer is hospitalized for a limb salvage procedure. How can the nurse best address the client's psychosocial needs? a. Assess the client's coping skills and support systems. b. Explain that the surgery leads to a longer life expectancy. c. Refer the client to the social worker or hospital chaplain. d. Reinforce physical therapy to aid with ambulating normally.

ANS: A The first step in the nursing process is assessment. The nurse would assess coping skills and possible support systems that will be helpful in this client's treatment. Explaining that a limb salvage procedure will extend life does not address the client's psychosocial needs. Referrals may be necessary, but the nurse should assess first. Reinforcing physical therapy is also helpful but again does not address the psychosocial needs of the client.

4. A hospitalized client's strength of the upper extremities is rated at a 4. What does the nurse understand about this client's ability to perform activities of daily living (ADLs)? a. The client is able to perform ADLs but not lift some items. b. The client is unable to perform ADLs alone. c. No difficulties are expected with ADLs. d. The client would need almost total assistance with ADLs.

ANS: C This rating indicates good muscle strength with full range of motion.

5. A nurse who is applying eyedrops to a client holds pressure against the corner of the eye nearest the nose after instilling the drops. The client asks what the nurse is doing. What response by the nurse is best? a. Doing this allows time for absorption. b. I am keeping the drops in the eye. c. This prevents systemic absorption. d. I am stopping you from rubbing your eye.

ANS: C This technique, called punctal occlusion, prevents eyedrops from being absorbed systemically. The other answers are inaccurate.

11. A male client with chronic kidney disease (CKD) is refusing to take his medication and has missed two hemodialysis appointments. What is the best initial action for the nurse? a. Discuss what the treatment regimen means to him. b. Refer the client to a mental health nurse practitioner. c. Reschedule the appointments to another date and time. d. Discuss the option of peritoneal dialysis.

ANS: A The initial action for the nurse is to assess anxiety, coping styles, and the clients acceptance of the required treatment for CKD. The client may be in denial of the diagnosis. While rescheduling hemodialysis appointments may help, and referral to a mental health practitioner and the possibility of peritoneal dialysis are all viable options, assessment of the clients acceptance of the treatment should come first.

13. A nurse assesses a client diagnosed with adrenal hypofunction. Which client statement should the nurse correlate with this diagnosis? a. I have a terrible craving for potato chips. b. I cannot seem to drink enough water. c. I no longer have an appetite for anything. d. I get hungry even after eating a meal.

ANS: A The nurse correlates a clients salt craving with adrenal hypofunction. Excessive thirst is related to diabetes insipidus or diabetes mellitus. Clients who have hypothyroidism often have a decrease in appetite. Excessive hunger is associated with diabetes mellitus.

12. The nurse is preparing to give tolvaptan for a client who has syndrome of inappropriate antidiuretic hormone (SIADH). For which potentially life-threatening adverse effect would the nurse monitor? a. Increased intracranial pressure b. Myocardial infarction c. Rapid-onset hypernatremia d. Bowel perforation

ANS: C Tolvaptan has a black box warning that rapid increases in serum sodium levels have been associated with central nervous system demyelination that can lead to serious complications and death.

11. A client who is nearly blind is admitted to the hospital. What action by the nurse is most important? a. Allow the client to feel his or her way around. b. Let the client arrange objects on the bedside table. c. Orient the client to the room using a focal point. d. Speak loudly and slowing when talking to the client.

ANS: C Using a focal point, orient the client to the room by giving descriptions of items as they relate to the focal point. Letting the client arrange the bedside table is appropriate, but not as important as orienting the client to the room for safety. Allowing the client to just feel around may cause injury. Unless the client is also hearing impaired, use a normal tone of voice.

29. A client in the emergency department is having a stroke and needs a carotid artery angioplasty with stenting. The clients mental status is deteriorating. What action by the nurse is most appropriate? a. Attempt to find the family to sign a consent. b. Inform the provider that the procedure cannot occur. c. Nothing; no consent is needed in an emergency. d. Sign the consent form for the client.

ANS: A The nurse should attempt to find the family to give consent. If no family is present or can be found, under the principle of emergency consent, a life-saving procedure can be performed without formal consent. The nurse should not just sign the consent form.

3. A client had an arthroscopy 1 hour ago on the left knee. The nurse finds the left lower leg to be pale and cool, with 1+/4+ pedal pulses. What action by the nurse is best? a. Assess the neurovascular status of the right leg. b. Document the findings in the clients chart. c. Elevate the left leg on at least two pillows. d. Notify the provider of the findings immediately.

ANS: A The nurse should compare findings of the two legs as these findings may be normal for the client. If a difference is observed, the nurse notifies the provider. Documentation should occur after the nurse has all the data. Elevating the left leg will not improve perfusion if there is a problem.

11. A client has a traumatic brain injury. The nurse assesses the following: pulse change from 82 to 60 beats/min, pulse pressure increase from 26 to 40 mm Hg, and respiratory irregularities. What action by the nurse takes priority? a. Call the provider or Rapid Response Team. b. Increase the rate of the IV fluid administration. c. Notify respiratory therapy for a breathing treatment. d. Prepare to give IV pain medication.

ANS: A These manifestations indicate Cushings syndrome, a potentially life-threatening increase in intracranial pressure (ICP), which is an emergency. Immediate medical attention is necessary, so the nurse notifies the provider or the Rapid Response Team. Increasing fluids would increase the ICP. The client does not need a breathing treatment or pain medication.

18. A client with chronic kidney disease (CKD) is experiencing nausea, vomiting, visual changes, and anorexia. Which action by the nurse is best? a. Check the clients digoxin (Lanoxin) level. b. Administer an anti-nausea medication. c. Ask if the client is able to eat crackers. d. Get a referral to a gastrointestinal provider.

ANS: A These signs and symptoms are indications of digoxin (Lanoxin) toxicity. The nurse should check the level of this medication. Administering antiemetics, asking if the client can eat, and obtaining a referral to a specialist all address the clients symptoms but do not lead to the cause of the symptoms.

4. An emergency department nurse cares for a client who sustained a crush injury to the right lower leg. The client reports numbness and tingling in the affected leg. Which action should the nurse take first? a. Assess the pedal pulses. b. Apply oxygen by nasal cannula. c. Increase the IV flow rate. d. Loosen the traction.

ANS: A These symptoms represent early warning signs of acute compartment syndrome. In acute compartment syndrome, sensory deficits such as paresthesias precede changes in vascular or motor signs. If the nurse finds a decrease in pedal pulses, the health care provider should be notified as soon as possible. Vital signs need to be obtained to determine if oxygen and intravenous fluids are necessary. Traction, if implemented, should never be loosened without a providers prescription.

11. A nurse cares for a patient who is prescribed pioglitazone. After 6 months of therapy, the client reports that he has a new onset of ankle edema. What assessment question would the nurse take? a. "Have you gained unexpected weight this week?" b. "Has your urinary output declined recently?" c. "Have you had fever and achiness this week?" d. "Have you had abdominal pain recently?"

ANS: A Thiazolidinediones (including pioglitazone) can cause cardiovascular adverse effects including health failure which is manifested by peripheral edema and unintentional weight gain. The client should have been taught to weigh every week and report sudden increases in weight.

2. A marathon runner comes into the clinic and states I have not urinated very much in the last few days. The nurse notes a heart rate of 110 beats/min and a blood pressure of 86/58 mm Hg. Which action by the nurse is the priority? a. Give the client a bottle of water immediately. b. Start an intravenous line for fluids. c. Teach the client to drink 2 to 3 liters of water daily. d. Perform an electrocardiogram.

ANS: A This athlete is mildly dehydrated as evidenced by the higher heart rate and lower blood pressure. The nurse can start hydrating the client with a bottle of water first, followed by teaching the client to drink 2 to 3 liters of water each day. An intravenous line may be ordered later, after the clients degree of dehydration is assessed. An electrocardiogram is not necessary at this time.

17. A client who had a severe traumatic brain injury is being discharged home, where the spouse will be a full- time caregiver. What statement by the spouse would lead the nurse to provide further education on home care? a. I know I can take care of all these needs by myself. b. I need to seek counseling because I am very angry. c. Hopefully things will improve gradually over time. d. With respite care and support, I think I can do this.

ANS: A This caregiver has unrealistic expectations about being able to do everything without help. Acknowledging anger and seeking counseling show a realistic outlook and plans for accomplishing goals. Hoping for improvement over time is also realistic, especially with the inclusion of the word hopefully. Realizing the importance of respite care and support also is a realistic outlook.

7. A client is in the internal medicine clinic reporting bone pain. The clients alkaline phosphatase level is 180 units/L. What action by the nurse is most appropriate? a. Assess the client for leg bowing. b. Facilitate an oncology workup. c. Instruct the client on fluid restrictions. d. Teach the client about ibuprofen (Motrin).

ANS: A This client has manifestations of Pagets disease. The nurse should assess for other manifestations such as bowing of the legs. Other care measures can be instituted once the client has a confirmed diagnosis.

17. A client with osteoporosis is going home, where the client lives alone. What action by the nurse is best? a. Arrange a home safety evaluation. b. Ensure the client has a walker at home. c. Help the client look into assisted living. d. Refer the client to Meals on Wheels.

ANS: A This client has several risk factors that place him or her at a high risk for falling. The nurse should consult social work or home health care to conduct a home safety evaluation. The other options may or may not be needed based upon the clients condition at discharge.

5. The nurse assesses a client after a total hip arthroplasty. The client's surgical leg is visibly shorter than the other one and the client reports extreme pain. While a co-worker calls the surgeon, what action by the nurse is appropriate? a. Assess neurovascular status in both legs. b. Elevate the surgical leg and apply ice. c. Prepare to administer pain medication. d. Try to place the surgical leg in abduction.

ANS: A This client has signs and symptoms of hip dislocation, a potential complication of this surgery. Hip dislocation can cause neurovascular compromise. The nurse would assess neurovascular status while comparing both legs. The nurse would not try to move the extremity to elevate or abduct it. Pain medication may be administered if possible, but first the nurse would thoroughly assess the client.

10. A nurse assesses a client in the preoperative holding area and finds brittle nails and hair, dry skin turgor, and muscle wasting. What action by the nurse is best? a. Consult the primary health care provider about a dietitian referral. b. Document the findings thoroughly in the client's chart. c. Encourage the client to eat more after recovering from surgery. d. Refer the client to Meals on Wheels after discharge.

ANS: A This client has signs of malnutrition, which can impact recovery from surgery. The nurse should consult the primary health care provider about prescribing a consultation with a dietitian in the postoperative period. The nurse should document the findings but needs to do more. Encouraging the client to eat more may be helpful, but the client needs a professional nutritional assessment so that the appropriate diet and supplements can be ordered. The client may or may not need Meals on Wheels after discharge.

31.A nurse assesses a client who has diabetes mellitus and notes the client is awake and alert, but shaky, diaphoretic, and weak. Five minutes after administering a half-cup of orange juice, the clients clinical manifestations have not changed. Which action should the nurse take next? a. Administer another half-cup of orange juice. b. Administer a half-ampule of dextrose 50% intravenously. c. Administer 10 units of regular insulin subcutaneously. d. Administer 1 mg of glucagon intramuscularly.

ANS: A This client is experiencing mild hypoglycemia. For mild hypoglycemic manifestations, the nurse should administer oral glucose in the form of orange juice. If the symptoms do not resolve immediately, the treatment should be repeated. The client does not need intravenous dextrose, insulin, or glucagon.

23. A client has a subarachnoid bolt. What action by the nurse is most important? a. Balancing and recalibrating the device b. Documenting intracranial pressure readings c. Handling the fiberoptic cable with care to avoid breakage d. Monitoring the clients phlebostatic axis

ANS: A This device needs frequent balancing and recalibration in order to read correctly. Documenting readings is important, but it is more important to ensure the devices accuracy. The fiberoptic transducer-tipped catheter has a cable that must be handled carefully to avoid breaking it, but ensuring the devices accuracy is most important. The phlebostatic axis is not related to neurologic monitoring.

22. A nurse assesses the left plantar reflexes of an adult client and notes the response shown in the photograph below: Which action should the nurse take next? a. Contact the provider with this abnormal finding. b. Assess bilateral legs for temperature and edema. c. Ask the client about pain in the lower leg and calf. d. Document the finding and continue the assessment.

ANS: A This finding indicates Babinskis sign. In clients older than 2 years of age, Babinskis sign is considered abnormal and indicates central nervous system disease. The nurse should notify the health care provider and other members of the health care team because further investigation is warranted. This finding does not relate to perfusion of the leg or to pain. This is an abnormal assessment finding and should be addressed immediately.

7. The nurse enters an examination room to help with an eye examination. The client is directed toward the assessment chart shown below: What is the provider assessing? a. Color vision b. Depth perception c. Spatial perception d. Visual acuity

ANS: A This is an Ishihara chart, which is used for assessing color vision. Depth and spatial perception are not typically assessed in a routine vision assessment. Visual acuity is usually tested with a Snellen chart.

4. A hospitalized clients strength of the upper extremities is rated at 3. What does the nurse understand about this clients ability to perform activities of daily living (ADLs)? a. The client is able to perform ADLs but not lift some items. b. No difficulties are expected with ADLs. c. The client is unable to perform ADLs alone. d. The client would need near-total assistance with ADLs.

ANS: A This rating indicates fair muscle strength with full range of motion against gravity but not resistance. The client could complete ADLs independently unless they required lifting objects.

8. A school nurse is conducting scoliosis screening. In screening the client, what technique is most appropriate? a. Bending forward from the hips b. Sitting upright with arms outstretched c. Walking across the room and back d. Walking with both eyes closed

ANS: A To assess for scoliosis, a spinal deformity, the student should bend forward at the hips. Standing behind the student, the nurse looks for a lateral curve in the spine. The other actions are not correct.

10. A client has a foreign body in one eye. What action by the nurse is appropriate for the client's care? a. Administering ordered antibiotics b. Assessing the patient's visual acuity c. Obtaining consent for enucleation d. Removing the object immediately

ANS: A To prevent infection, antibiotics are provided. Visual acuity in the affected eye cannot be assessed. The client may or may not need enucleation. The object is only removed by the ophthalmologist.

14. A nurse witnesses a client with late-stage Alzheimers disease eat breakfast. Afterward the client states, I am hungry and want breakfast. How should the nurse respond? a. I see you are still hungry. I will get you some toast. b. You ate your breakfast 30 minutes ago. c. It appears you are confused this morning. d. Your family will be here soon. Lets get you dressed.

ANS: A Use of validation therapy with clients who have Alzheimers disease involves acknowledgment of the clients feelings and concerns. This technique has proved more effective in later stages of the disease, when using reality orientation only increases agitation. Telling the client that he or she already ate breakfast may agitate the client. The other statements do not validate the clients concerns.

7. A nurse is providing community education about preventing traumatic musculoskeletal injuries related to car crashes. Which group does the nurse target as the priority for this education? a. High school football team b. High school homeroom class c. Middle-aged men d. Older adult women

ANS: A Young men are at highest risk for musculoskeletal injury due to trauma, especially due to motor vehicle crashes. The high school football team, with its roster of young males, is the priority group.

10. A nurse is seeing many clients in the neurosurgical clinic. With which clients should the nurse plan to do more teaching? (Select all that apply.) a. Client with an aneurysm coil placed 2 months ago who is taking ibuprofen (Motrin) for sinus headaches b. Client with an aneurysm clip who states that his family is happy there is no chance of recurrence c. Client who had a coil procedure who says that there will be no problem following up for 1 year d. Client who underwent a flow diversion procedure 3 months ago who is taking docusate sodium (Colace) for constipation e. Client who underwent surgical aneurysm ligation 3 months ago who is planning to take a Caribbean cruise

ANS: A, B After a coil procedure, up to 20% of clients experience re-bleeding in the first year. The client with this coil should not be taking drugs that interfere with clotting. An aneurysm clip can move up to 5 years after placement, so this client and family need to be watchful for changing neurologic status. The other statements show good understanding.

3. A nurse assesses clients who have endocrine disorders. Which assessment findings are paired correctly with the endocrine disorder? (Select all that apply.) a. Excessive thyroid-stimulating hormone Increased bone formation b. Excessive melanocyte-stimulating hormone Darkening of the skin c. Excessive parathyroid hormone Synthesis and release of corticosteroids d. Excessive antidiuretic hormone Increased urinary output e. Excessive adrenocorticotropic hormone Increased bone resorption

ANS: A, B Thyroid-stimulating hormone targets thyroid tissue and stimulates the formation of bone. Melanocyte- stimulating hormone stimulates melanocytes and promotes pigmentation or the darkening of the skin. Parathyroid hormone stimulates bone resorption. Antidiuretic hormone targets the kidney and promotes water reabsorption, causing a decrease in urinary output. Adrenocorticotropic hormone targets the adrenal cortex and stimulates the synthesis and release of corticosteroids.

5. A nurse is caring for a client who is prescribed a computed tomography (CT) scan with iodine-based contrast. Which actions should the nurse take to prepare the client for this procedure? (Select all that apply.) a. Ensure that an informed consent is present. b. Ask the client about any allergies. c. Evaluate the clients renal function. d. Auscultate bilateral breath sounds. e. Assess hematocrit and hemoglobin levels.

ANS: A, B, C A client who is scheduled to receive iodine-based contrast should be asked about allergies, especially allergies to iodine or shellfish. The clients kidney function should also be evaluated to determine if it is safe to administer contrast during the procedure. Finally, the nurse should ensure that an informed consent is present because all clients receiving iodine-based contrast must give consent. The CT will have no impact on the clients breath sounds or hematocrit and hemoglobin levels. Findings from these assessments will not influence the clients safety during the procedure.

7. A nurse assesses clients on a medical-surgical unit. Which clients should the nurse identify as at risk for secondary seizures? (Select all that apply.) a. A 26-year-old woman with a left temporal brain tumor b. A 38-year-old male client in an alcohol withdrawal program c. A 42-year-old football player with a traumatic brain injury d. A 66-year-old female client with multiple sclerosis e. A 72-year-old man with chronic obstructive pulmonary disease

ANS: A, B, C Clients at risk for secondary seizures include those with a brain lesion from a tumor or trauma, and those who are experiencing a metabolic disorder, acute alcohol withdrawal, electrolyte disturbances, and high fever. Clients with a history of stroke, heart disease, and substance abuse are also at risk. Clients with multiple sclerosis or chronic obstructive pulmonary disease are not at risk for secondary seizures.

4.A nurse provides diabetic education at a public health fair. Which disorders should the nurse include as complications of diabetes mellitus? (Select all that apply.) a. Stroke b. Kidney failure c. Blindness d. Respiratory failure e. Cirrhosis

ANS: A, B, C Complications of diabetes mellitus are caused by macrovascular and microvascular changes. Macrovascular complications include coronary artery disease, cerebrovascular disease, and peripheral vascular disease. Microvascular complications include nephropathy, retinopathy, and neuropathy. Respiratory failure and cirrhosis are not complications of diabetes mellitus.

1. A nurse cares for clients with hormone disorders. Which are common key features of hormones? (Select all that apply.) a. Hormones may travel long distances to get to their target tissues. b. Continued hormone activity requires continued production and secretion. c. Control of hormone activity is caused by negative feedback mechanisms. d. Most hormones are stored in the target tissues for use later. e. Most hormones cause target tissues to change activities by changing gene activity.

ANS: A, B, C Hormones are secreted by endocrine glands and travel through the body to reach their target tissues. Hormone activity can increase or decrease according to the bodys needs, and continued hormone activity requires continued production and secretion. Control is maintained via negative feedback. Hormones are not stored for later use, and they do not alter genetic activity.

1. The nurse is caring for five clients on the medical-surgical unit. Which clients would the nurse consider to be at risk for post-renal acute kidney injury (AKI)? (Select all that apply.) a. Man with prostate cancer b. Woman with blood clots in the urinary tract c. Client with ureterolithiasis d. Firefighter with severe burns e. Young woman with lupus

ANS: A, B, C Urine flow obstruction, such as prostate cancer, blood clots in the urinary tract, and kidney stones (ureterolithiasis), causes post-renal AKI. Severe burns would be a pre-renal cause. Lupus would be an intrarenal cause for AKI.

2. A nurse coordinates care for a client with a wet plaster cast. Which statement should the nurse include when delegating care for this client to an unlicensed assistive personnel (UAP)? a. Assess distal pulses for potential compartment syndrome. b. Turn the client every 3 to 4 hours to promote cast drying. c. Use a cloth-covered pillow to elevate the clients leg. d. Handle the cast with your fingertips to prevent indentations.

ANS: C When delegating care to a UAP for a client with a wet plaster cast, the UAP should be directed to ensure that the extremity is elevated on a cloth pillow instead of a plastic pillow to promote drying. The client should be assessed for impaired arterial circulation, a complication of compartment syndrome; however, the nurse should not delegate assessments to a UAP. The client should be turned every 1 to 2 hours to allow air to circulate and dry all parts of the cast. Providers should handle the cast with the palms of the hands to prevent indentations.

7. A client has meningitis following brain surgery. What comfort measures may the nurse delegate to the unlicensed assistive personnel (UAP)? (Select all that apply.) a. Applying a cool washcloth to the head b. Assisting the client to a position of comfort c. Keeping voices soft and soothing d. Maintaining low lighting in the room e. Providing antipyretics for fever

ANS: A, B, C, D The client with meningitis often has high fever, pain, and some degree of confusion. Cool washcloths to the forehead are comforting and help with pain. Allowing the client to assume a position of comfort also helps manage pain. Keeping voices low and lights dimmed also helps convey caring in a nonthreatening manner. The nurse provides antipyretics for fever.

19. A postoperative nurse is caring for a client who received a neuromuscular blocking agent during surgery. On assessment the nurse notes the client has weak hand grasps. What assessment does the nurse conduct next? a. Ability to raise head off the bed b. Blood pressure and pulse c. Signs of oxygenation d. Level of orientation

ANS: C When neuromuscular blocking agents are retained, muscle weakness could affect the diaphragm and impair gas exchange. Symptoms include the inability to maintain a head lift, weak hand grasps, and an abdominal breathing pattern. Since the client has weak hand grasps, the nurse would assess for signs of systemic oxygenation next. The nurse would assess head lift ability, but this does not take priority over oxygenation. Blood pressure, pulse, and level of orientation are all important in the postoperative period, but oxygenation would come first.

13. A client is diagnosed with chronic kidney disease (CKD). What is an ideal goal of treatment set by the nurse in the care plan to reduce the risk of pulmonary edema? a. Maintaining oxygen saturation of 89% b. Minimal crackles and wheezes in lung sounds c. Maintaining a balanced intake and output d. Limited shortness of breath upon exertion

ANS: C With an optimal fluid balance, the client will be more able to eject blood from the left ventricle without increased pressure in the left ventricle and pulmonary vessels. Other ideal goals are oxygen saturations greater than 92%, no auscultated crackles or wheezes, and no demonstrated shortness of breath.

12. A client in the preoperative holding room has received sedation and now needs to urinate. What action by the nurse is best? a. Allow the client to walk to the bathroom. b. Delegate assisting the client to the nurses aide. c. Give the client a bedpan or urinal to use. d. Insert a urinary catheter now instead of waiting.

ANS: C Although possibly uncomfortable or embarrassing for the client, the client should not be allowed out of bed after receiving sedation. The nurse should get the client a bedpan or urinal. The client may or may not need a urinary catheter.

5. A nurse teaches a client with polycystic kidney disease (PKD). Which statements should the nurse include in this clients discharge teaching? (Select all that apply.) a. Take your blood pressure every morning. b. Weigh yourself at the same time each day. c. Adjust your diet to prevent diarrhea. d. Contact your provider if you have visual disturbances. e. Assess your urine for renal stones.

ANS: A, B, D A client who has PKD should measure and record his or her blood pressure and weight daily, limit salt intake, and adjust dietary selections to prevent constipation. The client should notify the provider if urine smells foul or has blood in it, as these are signs of a urinary tract infection or glomerular injury. The client should also notify the provider if visual disturbances are experienced, as this is a sign of a possible berry aneurysm, which is a complication of PKD. Diarrhea and renal stones are not manifestations or complications of PKD; therefore, teaching related to these concepts would be inappropriate.

4. A nurse assesses a client with hypothyroidism who is admitted with acute appendicitis. The nurse notes that the clients level of consciousness has decreased. Which actions should the nurse take? (Select all that apply.) a. Infuse intravenous fluids. b. Cover the client with warm blankets. c. Monitor blood pressure every 4 hours. d. Maintain a patent airway. e. Administer oral glucose as prescribed.

ANS: A, B, D A client with hypothyroidism and an acute illness is at risk for myxedema coma. A decrease in level of consciousness is a symptom of myxedema. The nurse should infuse IV fluids, cover the client with warm blankets, monitor blood pressure every hour, maintain a patent airway, and administer glucose intravenously as prescribed.

1. A nurse assesses a client who has had two episodes of bacterial cystitis in the last 6 months. Which questions should the nurse ask? (Select all that apply.) a. How much water do you drink every day? b. Do you take estrogen replacement therapy? c. Does anyone in your family have a history of cystitis? d. Are you on steroids or other immune-suppressing drugs? e. Do you drink grapefruit juice or orange juice daily?

ANS: A, B, D Fluid intake, estrogen levels, and immune suppression all can increase the chance of recurrent cystitis. Family history is usually insignificant, and cranberry juice, not grapefruit or orange juice, has been found to increase the acidic pH and reduce the risk for bacterial cystitis.

6. A client is undergoing hemodialysis. The clients blood pressure at the beginning of the procedure was 136/88 mm Hg, and now it is 110/54 mm Hg. What actions should the nurse perform to maintain blood pressure? (Select all that apply.) a. Adjust the rate of extracorporeal blood flow. b. Place the client in the Trendelenburg position. c. Stop the hemodialysis treatment. d. Administer a 250-mL bolus of normal saline. e. Contact the health care provider for orders.

ANS: A, B, D Hypotension occurs often during hemodialysis treatments as a result of vasodilation from the warmed dialysate. Modest decreases in blood pressure, as is the case with this client, can be maintained with rate adjustment, Trendelenburg positioning, and a fluid bolus. If the blood pressure drops considerably after two boluses and cooling dialysate, the hemodialysis can be stopped and the health care provider contacted.

6. A nurse assesses clients with potential endocrine disorders. Which clients are at high risk for adrenal insufficiency? (Select all that apply.) a. A 22-year-old female with metastatic cancer b. A 43-year-old male with tuberculosis c. A 51-year-old female with asthma d. A 65-year-old male with gram-negative sepsis e. A 70-year-old female with hypertension

ANS: A, B, D Metastatic cancer, tuberculosis, and gram-negative sepsis are primary causes of adrenal insufficiency. Active tuberculosis is a contributing factor for syndrome of inappropriate antidiuretic hormone. Hypertension is a key manifestation of Cushings disease. These are not risk factors for adrenal insufficiency.

7. A client is unsure of the decision to undergo peritoneal dialysis (PD) and wishes to discuss the advantages of this treatment with the nurse. Which statements by the nurse are accurate regarding PD? (Select all that apply.) a. You will not need vascular access to perform PD. b. There is less restriction of protein and fluids. c. You will have no risk for infection with PD. d. You have flexible scheduling for the exchanges. e. It takes less time than hemodialysis treatments.

ANS: A, B, D PD is based on exchanges of waste, fluid, and electrolytes in the peritoneal cavity. There is no need for vascular access. Protein is lost in the exchange, which allows for more protein and fluid in the diet. There is flexibility in the time for exchanges, but the treatment takes a longer period of time compared to hemodialysis. There still is risk for infection with PD, especially peritonitis.

4. A nurse plans care for a client who is recovering from open reduction and internal fixation (ORIF) surgery for a right hip fracture. Which interventions should the nurse include in this clients plan of care? (Select all that apply.) a. Elevate heels off the bed with a pillow. b. Ambulate the client on the first postoperative day. c.Push the clients patient-controlled analgesia button. d. Re-position the client every 2 hours. e. Use pillows to encourage subluxation of the hip.

ANS: A, B, D Postoperative care for a client who has ORIF of the hip includes elevating the clients heels off the bed and re- positioning every 2 hours to prevent pressure and skin breakdown. It also includes ambulating the client on the first postoperative day, and using pillows or an abduction pillow to prevent subluxation of the hip. The nurse should teach the client to use the patient-controlled analgesia pump, but the nurse should never push the button for the client.

5. A nurse cares for clients with urinary incontinence. Which types of incontinence are correctly paired with their clinical manifestation? (Select all that apply.) a. Stress incontinence Urine loss with physical exertion b. Urge incontinence Large amount of urine with each occurrence c. Functional incontinence Urine loss results from abnormal detrusor contractions d. Overflow incontinence Constant dribbling of urine e. Reflex incontinence Leakage of urine without lower urinary tract disorder

ANS: A, B, D Stress incontinence is a loss of urine with physical exertion, coughing, sneezing, or exercising. Urge incontinence presents with an abrupt and strong urge to void and usually has a large amount of urine released with each occurrence. Overflow incontinence occurs with bladder distention and results in a constant dribbling of urine. Functional incontinence is the leakage of urine caused by factors other than a disorder of the lower urinary tract. Reflex incontinence results from abnormal detrusor contractions from a neurologic abnormality.

7. A nurse delegates care for an older adult client to the unlicensed assistive personnel (UAP). Which statements should the nurse include when delegating this clients care? (Select all that apply.) a. Plan to bathe the client in the evening when the client is most alert. b. Encourage the client to use a cane when ambulating. c. Assess the client for symptoms related to pain and discomfort. d. Remind the client to look at foot placement when walking. e. Schedule additional time for teaching about prescribed therapies.

ANS: A, B, D The nurse should tell the UAP to schedule activities when the client is normally awake, encourage the client to use a cane when ambulating, and remind the client to look where feet are placed when walking. The nurse should assess the client for symptoms of pain and should provide sufficient time for older adults to process information, including new teaching. These are not items the nurse can delegate.

1. A nurse reviews a clients laboratory results. Which results from the clients urinalysis should the nurse identify as normal? (Select all that apply.) a. pH: 6 b. Specific gravity: 1.015 c. Protein: 1.2 mg/dL d. Glucose: negative e. Nitrate: small f. Leukocyte esterase: positive

ANS: A, B, D The pH, specific gravity, and glucose are all within normal ranges. The other values are abnormal.

5.A nurse collaborates with the interdisciplinary team to develop a plan of care for a client who is newly diagnosed with diabetes mellitus. Which team members should the nurse include in this interdisciplinary team meeting? (Select all that apply.) a. Registered dietitian b. Clinical pharmacist c. Occupational therapist d. Health care provider e. Speech-language pathologist

ANS: A, B, D When planning care for a client newly diagnosed with diabetes mellitus, the nurse should collaborate with a registered dietitian, clinical pharmacist, and health care provider. The focus of treatment for a newly diagnosed client would be nutrition, medication therapy, and education. The nurse could also consult with a diabetic educator. There is no need for occupational therapy or speech therapy at this time.

8. A client is suspected to have muscular dystrophy. About what diagnostic testing does the nurse educate the client? (Select all that apply.) a. Electromyography b. Muscle biopsy c. Nerve conduction studies d. Serum aldolase e. Serum creatinine kinase

ANS: A, B, D, E Diagnostic testing for muscular dystrophy includes electromyography, muscle biopsy, serum aldolase and creatinine kinase levels. Nerve conduction is not related to this disorder.

1. A nurse is assessing a community group for dietary factors that contribute to osteoporosis. In addition to inquiring about calcium, the nurse also assesses for which other dietary components? (Select all that apply.) a. Alcohol b. Caffeine c. Fat d. Carbonated beverages e. Vitamin D

ANS: A, B, D, E Dietary components that affect the development of osteoporosis include alcohol, caffeine, high phosphorus intake, carbonated beverages, and vitamin D. Tobacco is also a contributing lifestyle factor. Fat intake does not contribute to osteoporosis.

3. The nursing student learns that age-related changes affect the eyes and vision. Which changes does this include? (Select all that apply.) a. Decreased eye muscle tone b. Development of arcus senilis c. Increase in far point of near vision d. Decrease in general color perception e. Increase in point of near vision

ANS: A, B, D, E Normal age-related changes include decreased eye muscle tone, development of arcus senilis, decreased color perception, and increased point of near vision. The far point of near vision typically decreases.

4. When assessing gait, what features does the nurse inspect? (Select all that apply.) a. Balance b. Ease of stride c. Goniometer readings d. Length of stride e. Steadiness

ANS: A, B, D, E To assess gait, look at balance, ease and length of stride, and steadiness. Goniometer readings assess flexion and extension or joint range of motion.

8. A nurse plans care for a client with a halo fixator. Which interventions should the nurse include in this clients plan of care? (Select all that apply.) a. Tape a halo wrench to the clients vest. b. Assess the pin sites for signs of infection. c. Loosen the pins when sleeping. d. Decrease the clients oral fluid intake. e. Assess the chest and back for skin breakdown.

ANS: A, B, E A special halo wrench should be taped to the clients vest in case of a cardiopulmonary emergency. The nurse should assess the pin sites for signs of infection or loose pins and for complications from the halo. The nurse should also increase fluids and fiber to decrease bowel straining and assess the clients chest and back for skin breakdown from the halo vest.

2. After teaching a client with a spinal cord tumor, the nurse assesses the clients understanding. Which statements by the client indicate a correct understanding of the teaching? (Select all that apply.) a. Even though turning hurts, I will remind you to turn me every 2 hours. b. Radiation therapy can shrink the tumor but also can cause more problems. c. Surgery will be scheduled to remove the tumor and reverse my symptoms. d. I put my affairs in order because this type of cancer is almost always fatal. e. My family is moving my bedroom downstairs for when I am discharged home.

ANS: A, B, E Although surgery may relieve symptoms by reducing pressure on the spine and debulking the tumor, some motor and sensory deficits may remain. Spinal tumors usually cause disability but are not usually fatal. Radiation therapy is often used to shrink spinal tumors but can cause progressive spinal cord degeneration and neurologic deficits. The client should be turned every 2 hours to prevent skin breakdown and arrangements should be made at home so that the client can complete activities of daily living without needing to go up and down stairs.

4. A nurse assesses a client who is experiencing a cluster headache. Which clinical manifestations should the nurse expect to find? (Select all that apply.) a. Ipsilateral tearing of the eye b. Miosis c. Abrupt loss of consciousness d. Neck and shoulder tenderness e. Nasal congestion f. Exophthalmos

ANS: A, B, E Cluster headache is usually accompanied by ipsilateral tearing, miosis, rhinorrhea or nasal congestion, ptosis, eyelid edema, and facial sweating. Abrupt loss of consciousness, neck and shoulder tenderness, and exophthalmos are not associated with cluster headaches.

1. A nurse teaches a client with a fractured tibia about external fixation. Which advantages of external fixation for the immobilization of fractures should the nurse share with the client? (Select all that apply.) a. It leads to minimal blood loss. b. It allows for early ambulation. c. It decreases the risk of infection. d. It increases blood supply to tissues. e. It promotes healing.

ANS: A, B, E External fixation is a system in which pins or wires are inserted through the skin and bone and then connected to a ridged external frame. With external fixation, blood loss is less than with internal fixation, but the risk for infection is much higher. The device allows early ambulation and exercise, maintains alignment, stabilizes the fracture site, and promotes healing. The device does not increase blood supply to the tissues. The nurse should assess for distal circulation, movement, and sensation, which can be disturbed by fracture injuries and treatments.

6. A nurse teaches a client who is at risk for carpal tunnel syndrome. Which health promotion activities should the nurse include in this clients teaching? (Select all that apply.) a. Frequently assess the ergonomics of the equipment being used. b. Take breaks to stretch fingers and wrists during working hours. c. Do not participate in activities that require repetitive actions. d. Take ibuprofen (Motrin) to decrease pain and swelling in wrists. e. Adjust chair height to allow for good posture.

ANS: A, B, E Health promotion activities to prevent carpal tunnel syndrome include assessing the ergonomics of the equipment being used, taking breaks to stretch fingers and wrists during working hours, and adjusting chair height to allow for good posture. The client should be allowed to participate in activities that require repetitive actions as long as precautions are taken to promote health. Pain medications are not part of health promotion activities.

4. A nurse prepares a client for a percutaneous kidney biopsy. Which actions should the nurse take prior to this procedure? (Select all that apply.) a. Keep the client NPO for 4 to 6 hours. b. Obtain coagulation study results. c. Maintain strict bedrest in a supine position. d. Assess for blood in the clients urine. e. Administer antihypertensive medications.

ANS: A, B, E Prior to a percutaneous kidney biopsy, the client should be NPO for 4 to 6 hours. Coagulation studies should be completed to prevent bleeding after the biopsy. Blood pressure medications should be administered to prevent hypertension before and after the procedure. There is no need to keep the client on bedrest or assess for blood in the clients urine prior to the procedure; these interventions should be implemented after a percutaneous kidney biopsy.

8. A nurse teaches a female client who has stress incontinence. Which statements should the nurse include about pelvic muscle exercises? (Select all that apply.) a. When you start and stop your urine stream, you are using your pelvic muscles. b. Tighten your pelvic muscles for a slow count of 10 and then relax for a slow count of 10. c. Pelvic muscle exercises should only be performed sitting upright with your feet on the floor. d. After you have been doing these exercises for a couple days, your control of urine will improve. e. Like any other muscle in your body, you can make your pelvic muscles stronger by contracting them.

ANS: A, B, E The client should be taught that the muscles used to start and stop urination are pelvic muscles, and that pelvic muscles can be strengthened by contracting and relaxing them. The client should tighten pelvic muscles for a slow count of 10 and then relax the muscles for a slow count of 10, and perform this exercise 15 times while in lying-down, sitting-up, and standing positions. The client should begin to notice improvement in control of urine after several weeks of exercising the pelvic muscles.

4. A nurse assesses a client with a brain tumor. Which newly identified assessment findings should alert the nurse to urgently communicate with the health care provider? (Select all that apply.) a. Glasgow Coma Scale score of 8 b. Decerebrate posturing c. Reactive pupils d. Uninhibited speech e. Diminished cognition

ANS: A, B, E The nurse should urgently communicate changes in a clients neurologic status, including a decrease in the Glasgow Coma Scale score, abnormal flexion or extension, changes in cognition or speech, and pinpointed, dilated, and nonreactive pupils.

5. A nurse plans care for an older adult client. Which interventions should the nurse include in this clients plan of care to promote kidney health? (Select all that apply.) a. Ensure adequate fluid intake. b. Leave the bathroom light on at night. c. Encourage use of the toilet every 6 hours. d. Delegate bladder training instructions to the unlicensed assistive personnel (UAP). e. Provide thorough perineal care after each voiding. f. Assess for urinary retention and urinary tract infection.

ANS: A, B, E, F The nurse should ensure that the client receives adequate fluid intake and has adequate lighting to ambulate safely to the bathroom at night, encourage the client to use the toilet every 2 hours, provide thorough perineal care after each voiding, and assess for urinary retention and urinary tract infections. The nurse should not delegate any teaching to the UAP, including bladder training instructions. The UAP may participate in bladder training activities, including encouraging and assisting the client to the bathroom at specific times.

3.A nurse teaches a client with diabetes mellitus about foot care. Which statements should the nurse include in this clients teaching? (Select all that apply.) a. Do not walk around barefoot. b. Soak your feet in a tub each evening. c. Trim toenails straight across with a nail clipper. d. Treat any blisters or sores with Epsom salts. e. Wash your feet every other day.

ANS: A, C Clients who have diabetes mellitus are at high risk for wounds on the feet secondary to peripheral neuropathy and poor arterial circulation. The client should be instructed to not walk around barefoot or wear sandals with open toes. These actions place the client at higher risk for skin breakdown of the feet. The client should be instructed to trim toenails straight across with a nail clipper. Feet should be washed daily with lukewarm water and soap, but feet should not be soaked in the tub. The client should contact the provider immediately if blisters or sores appear and should not use home remedies to treat these wounds.

6. The nurse is assessing a client for chronic osteomyelitis. Which features distinguish this from the acute form of the disease? (Select all that apply.) a. Draining sinus tracts b. High fevers c. Presence of foot ulcers d. Swelling and redness e. Tenderness or pain

ANS: A, C Draining sinus tracts and foot ulcers are seen in chronic osteomyelitis. High fever, swelling, and redness are more often seen in acute osteomyelitis. Pain or tenderness can be in either case.

3. A client is scheduled for a tympanoplasty. What actions by the nurse are most appropriate? (Select all that apply.) a. Administer preoperative antibiotics. b. Assess for allergies to local anesthetics. c. Ensure that informed consent is on the chart. d. Give ordered antivertigo medications. e. Teach that hearing improves immediately.

ANS: A, C Preoperatively, the nurse administers antibiotics and ensures that informed consent is on the chart. Local anesthetics can be used, but general anesthesia is used more often. Antivertigo medications are not used. Hearing will be decreased immediately after the operation until the ear packing is removed.

1. A nursing student studying the neurologic system learns which information? (Select all that apply.) a. An aneurysm is a ballooning in a weakened part of an arterial wall. b. An arteriovenous malformation is the usual cause of strokes. c. Intracerebral hemorrhage is bleeding directly into the brain. d. Reduced perfusion from vasospasm often makes stroke worse. e. Subarachnoid hemorrhage is caused by high blood pressure.

ANS: A, C, D An aneurysm is a ballooning of the weakened part of an arterial wall. Intracerebral hemorrhage is bleeding directly into the brain. Vasospasm often makes the damage from the initial stroke worse because it causes decreased perfusion. An arteriovenous malformation (AVM) is unusual. Subarachnoid hemorrhage is usually caused by a ruptured aneurysm or AVM.

5. A nurse assesses a client with a cast for potential compartment syndrome. Which clinical manifestations are correctly paired with the physiologic changes of compartment syndrome? (Select all that apply.) a. Edema Increased capillary permeability b. Pallor Increased blood blow to the area c. Unequal pulses Increased production of lactic acid d. Cyanosis Anaerobic metabolism e. Tingling A release of histamine

ANS: A, C, D Clinical manifestations of compartment syndrome are caused by several physiologic changes. Edema is caused by increased capillary permeability, release of histamine, decreased tissue perfusion, and vasodilation. Unequal pulses are caused by an increased production of lactic acid. Cyanosis is caused by anaerobic metabolism. Pallor is caused by decreased oxygen to tissues, and tingling is caused by increased tissue pressure.

3. A nurse evaluates the results of diagnostic tests on a clients cerebrospinal fluid (CSF). Which fluid results alerts the nurse to possible viral meningitis? (Select all that apply.) a. Clear b. Cloudy c. Increased protein level d. Normal glucose level e. Bacterial organisms present f. Increased white blood cells

ANS: A, C, D In viral meningitis, CSF fluid is clear, protein levels are slightly increased, and glucose levels are normal. Viral meningitis does not cause cloudiness or increased turbidity of CSF. In bacterial meningitis, the presence of bacteria and white blood cells causes the fluid to be cloudy.

6. A nurse cares for older clients who have traumatic brain injury. What should the nurse understand about this population? (Select all that apply.) a. Admission can overwhelm the coping mechanisms for older clients. b. Alcohol is typically involved in most traumatic brain injuries for this age group. c. These clients are more susceptible to systemic and wound infections. d. Other medical conditions can complicate treatment for these clients. e. Very few traumatic brain injuries occur in this age group.

ANS: A, C, D Older clients often tolerate stress poorly, which includes being admitted to a hospital that is unfamiliar and noisy. Because of decreased protective mechanisms, they are more susceptible to both local and systemic infections. Other medical conditions can complicate their treatment and recovery. Alcohol is typically not related to traumatic brain injury in this population; such injury is most often from falls and motor vehicle crashes. The 65- to 76-year-old age group has the second highest rate of brain injuries compared to other age groups.

1. A nursing student studying the musculoskeletal system learns about important related hormones. What information does the student learn? (Select all that apply.) a. A lack of vitamin D can lead to rickets. b. Calcitonin increases serum calcium levels. c. Estrogens stimulate osteoblastic activity. d. Parathyroid hormone stimulates osteoclastic activity. e. Thyroxine stimulates estrogen release.

ANS: A, C, D Vitamin D is needed to absorb calcium and phosphorus. A deficiency of vitamin D can lead to rickets. Estrogen stimulates osteoblastic activity. Parathyroid hormone stimulates osteoclastic activity. Calcitonin decreases serum calcium levels when they get too high. Thyroxine increases the rate of protein synthesis in all tissue types.

2. An emergency department nurse assesses a client who was struck in the temporal lobe with a baseball. For which clinical manifestations that are related to a temporal lobe injury should the nurse assess? (Select all that apply.) a. Memory loss b. Personality changes c. Difficulty with sound interpretation d. Speech difficulties e. Impaired taste

ANS: A, C, D Wernickes area (language area) is located in the temporal lobe and enables the processing of words into coherent thought as well as the understanding of written or spoken words. The temporal lobe also is responsible for the auditory centers interpretation of sound and complicated memory patterns. Personality changes are related to frontal lobe injury. Impaired taste is associated with injury to the parietal lobe.

2. The nurse working in the emergency department assesses a client who has symptoms of stroke. For what modifiable risk factors should the nurse assess? (Select all that apply.) a. Alcohol intake b. Diabetes c. High-fat diet d. Obesity e. Smoking

ANS: A, C, D, E Alcohol intake, a high-fat diet, obesity, and smoking are all modifiable risk factors for stroke. Diabetes is not modifiable but is a risk factor that can be control led with medical intervention.

4. A client has a hearing aid. What care instructions does the nurse provide the unlicensed assistive personnel (UAP) in the care of this client? (Select all that apply.) a. Be careful not to drop the hearing aid when handling. b. Soak the hearing aid in hot water for 20 minutes. c. Turn the hearing aid off when the client goes to bed. d. Use a toothpick to clean debris from the device. e. Wash the device with soap and a small amount of warm water.

ANS: A, C, D, E All these actions except soaking the hearing aid are proper instructions for the nurse to give to the UAP. While some water is used to clean the hearing aid, excessive wetting should be avoided.

2. A nurse assesses clients with potential endocrine disorders. Which clients are at high risk for hypopituitarism? (Select all that apply.) a. A 20-year-old female with benign pituitary tumors b. A 32-year-old male with diplopia c. A 41-year-old female with anorexia nervosa d. A 55-year-old male with hypertension e. A 60-year-old female who is experiencing shock f. A 68-year-old male who has gained weight recently

ANS: A, C, D, E Pituitary tumors, anorexia nervosa, hypertension, and shock are all conditions that can cause hypopituitarism. Diplopia is a manifestation of hypopituitarism, and weight gain is a manifestation of Cushings disease and syndrome of inappropriate antidiuretic hormone. They are not risk factors for hypopituitarism.

1. A nursing student studying the auditory system learns about the structures of the inner ear. What structures does this include? (Select all that apply.) a. Cochlea b. Epitympanum c. Organ of Corti d. Semicircular canals e. Vestibule

ANS: A, C, D, E The cochlea, organ of Corti, semicircular canals, and vestibule are all part of the inner ear. The epitympanum is in the middle ear.

1. The student learning about vision should remember which facts related to the eyes? (Select all that apply.) a. Aqueous humor controls intraocular pressure. b. Cones work in low light conditions. c. Glaucoma occurs due to increased pressure in the eye. d. Muscles of the iris control light entering the eye. e. Rods work in low light conditions.

ANS: A, C, D, E The inflow and outflow of aqueous humor controls the intraocular pressure. Glaucoma results when the pressure is chronically high. Muscles of the iris relax and constrict to control the amount of light entering the eye. Rods work in low light conditions. Cones work in bright light conditions.

1. A nurse assesses a client with anterior pituitary hyperfunction. Which clinical manifestations should the nurse expect? (Select all that apply.) a. Protrusion of the lower jaw b. High-pitched voice c. Enlarged hands and feet d. Kyphosis e. Barrel-shaped chest f. Excessive sweating

ANS: A, C, D, E, F Anterior pituitary hyperfunction typically will cause protrusion of the lower jaw, deepening of the voice, enlarged hands and feet, kyphosis, barrel-shaped chest, and excessive sweating.

1. A nurse assesses a client who recently experienced a traumatic spinal cord injury. Which assessment data should the nurse obtain to assess the clients coping strategies? (Select all that apply.) a. Spiritual beliefs b. Level of pain c. Family support d. Level of independence e. Annual income f. Previous coping strategies

ANS: A, C, D, F Information about the clients preinjury psychosocial status, usual methods of coping with illness, difficult situations, and disappointments should be obtained. Determine the clients level of independence or dependence and his or her comfort level in discussing feelings and emotions with family members or close friends. Clients who are emotionally secure and have a positive self-image, a supportive family, and financial and job security often adapt to their injury. Information about the clients spiritual and religious beliefs or cultural background also assists the nurse in developing the plan of care. The other options do not supply as much information about coping.

3. A nurse assesses a client who potentially has hyperaldosteronism. Which serum laboratory values should the nurse associate with this disorder? (Select all that apply.) a. Sodium: 150 mEq/L b. Sodium: 130 mEq/L c. Potassium: 2.5 mEq/L d. Potassium: 5.0 mEq/L e. pH: 7.28 f. pH: 7.50

ANS: A, C, E Aldosterone increases reabsorption of sodium and excretion of potassium. Hyperaldosteronism causes hypern 4. A nurse teaches a client with Cushings disease. Which dietary requirements should the nurse include in this clients teaching? (Select all that apply.) a. Low calcium b. Low carbohydrate c. Low protein d. Low calories e. Low sodium* ANS: B, D, E The client with Cushings disease has weight gain, muscle loss, hyperglycemia, and sodium retention. Dietary modifications need to include reduction of carbohydrates and total calories to prevent or reduce the degree of hyperglycemia. Sodium retention causes water retention and hypertension. Clients are encouraged to restrict their sodium intake moderately. Clients often have bone density loss and need more calcium. Increased protein intake will help decrease muscle loss.

2.A nurse assesses a client who is experiencing diabetic ketoacidosis (DKA). For which manifestations should the nurse monitor the client? (Select all that apply.) a. Deep and fast respirations b. Decreased urine output c. Tachycardia d. Dependent pulmonary crackles e. Orthostatic hypotension

ANS: A, C, E DKA leads to dehydration, which is manifested by tachycardia and orthostatic hypotension. Usually clients have Kussmaul respirations, which are fast and deep. Increased urinary output (polyuria) is severe. Because of diuresis and dehydration, peripheral edema and crackles do not occur.

2. A student nurse learns about changes that occur to the musculoskeletal system due to aging. Which changes does this include? (Select all that apply.) a. Bone changes lead to potential safety risks. b. Increased bone density leads to stiffness. c. Osteoarthritis occurs due to cartilage degeneration. d. Osteoporosis is a universal occurrence. e. Some muscle tissue atrophy occurs with aging.

ANS: A, C, E Many age-related changes occur in the musculoskeletal system, including decreased bone density, degeneration of cartilage, and some degree of muscle tissue atrophy. Osteoporosis, while common, is not universal. Bone density decreases with age, not increases.

3. A client is hospitalized in the oliguric phase of acute kidney injury (AKI) and is receiving tube feedings. The nurse is teaching the clients spouse about the kidney-specific formulation for the enteral solution compared to standard formulas. What components should be discussed in the teaching plan? (Select all that apply.) a. Lower sodium b. Higher calcium c. Lower potassium d. Higher phosphorus e. Higher calories

ANS: A, C, E Many clients with AKI are too ill to meet caloric goals and require tube feedings with kidney-specific formulas that are lower in sodium, potassium, and phosphorus, and higher in calories than are standard formulas.

4. A client with chronic osteomyelitis is being discharged from the hospital. What information is important for the nurse to teach this client and family? (Select all that apply.) a. Adherence to the antibiotic regimen b. Correct intramuscular injection technique c. Eating high-protein and high-carbohydrate foods d. Keeping daily follow-up appointments e. Proper use of the intravenous equipment

ANS: A, C, E The client going home with chronic osteomyelitis will need long-term antibiotic therapy first intravenous, then oral. The client needs education on how to properly administer IV antibiotics, care for the IV line, adhere to the regimen, and eat a healthy diet to encourage wound healing. The antibiotics are not given by IM injection. The client does not need daily follow-up.

3. A nurse teaches a client with hyperthyroidism. Which dietary modifications should the nurse include in this clients teaching? (Select all that apply.) a. Increased carbohydrates b. Decreased fats c. Increased calorie intake d. Supplemental vitamins e. Increased proteins

ANS: A, C, E The client is hypermetabolic and has an increased need for carbohydrates, calories, and proteins. Proteins are especially important because the client is at risk for a negative nitrogen balance. There is no need to decrease fat intake or take supplemental vitamins.

2. A nurse is caring for a postoperative 70-kg client who had major blood loss during surgery. Which findings by the nurse should prompt immediate action to prevent acute kidney injury? (Select all that apply.) a. Urine output of 100 mL in 4 hours b. Urine output of 500 mL in 12 hours c. Large amount of sediment in the urine d. Amber, odorless urine e. Blood pressure of 90/60 mm Hg

ANS: A, C, E The low urine output, sediment, and blood pressure should be reported to the provider. Postoperatively, the nurse should measure intake and output, check the characteristics of the urine, and report sediment, hematuria, and urine output of less than 0.5 mL/kg/hour for 3 to 4 hours. A urine output of 100 mL is low, but a urine output of 500 mL in 12 hours should be within normal limits. Perfusion to the kidneys is compromised with low blood pressure. The amber odorless urine is normal.

2. A nurse cares for a client with a hypofunctioning anterior pituitary gland. Which hormones should the nurse expect to be affected by this condition? (Select all that apply.) a. Thyroid-stimulating hormone b. Vasopressin c. Follicle-stimulating hormone d. Calcitonin e. Growth hormone

ANS: A, C, E Thyroid-stimulating hormone, follicle-stimulating hormone, and growth hormone all are secreted by the anterior pituitary gland. Vasopressin is secreted from the posterior pituitary gland. Calcitonin is secreted from the thyroid gland.

2. A nurse teaches a client about self-catheterization in the home setting. Which statements should the nurse include in this clients teaching? (Select all that apply.) a. Wash your hands before and after self-catheterization. b. Use a large-lumen catheter for each catheterization. c. Use lubricant on the tip of the catheter before insertion. d. Self-catheterize at least twice a day or every 12 hours. e. Use sterile gloves and sterile technique for the procedure. f. Maintain a specific schedule for catheterization.

ANS: A, C, F The key points in self-catheterization include washing hands, using lubricants, and maintaining a regular schedule to avoid distention and retention of urine that leads to bacterial growth. A smaller rather than a larger lumen catheter is preferred. The client needs to catheterize more often than every 12 hours. Self-catheterization in the home is a clean procedure.

3. A nurse assesses a client recovering from a cystoscopy. Which assessment findings should alert the nurse to urgently contact the health care provider? (Select all that apply.) a. Decrease in urine output b. Tolerating oral fluids c. Prescription for metformin d. Blood clots present in the urine e. Burning sensation when urinating

ANS: A, D The nurse should monitor urine output and contact the provider if urine output decreases or becomes absent. The nurse should also assess for blood in the clients urine. The urine may be pink-tinged, but gross bleeding or blood clots should not be present. If bleeding is present, the nurse should urgently contact the provider. Tolerating oral fluids is a positive outcome and does not need intervention. Metformin would be a concern if the client received dye; no dye is used in a cystoscopy procedure. The client may experience a burning sensation when urinating after this procedure; this would not require a call to the provider.

5. A nurse teaches a client who is prescribed an unsealed radioactive isotope. Which statements should the nurse include in this clients education? (Select all that apply.) a. Do not share utensils, plates, and cups with anyone else. b. You can play with your grandchildren for 1 hour each day. c. Eat foods high in vitamins such as apples, pears, and oranges. d. Wash your clothing separate from others in the household. e. Take a laxative 2 days after therapy to excrete the radiation.

ANS: A, D, E A client who is prescribed an unsealed radioactive isotope should be taught to not share utensils, plates, and cups with anyone else; to avoid contact with pregnant women and children; to avoid eating foods with cores or bones, which will leave contaminated remnants; to wash clothing separate from others in the household and run an empty cycle before washing other peoples clothing; and to take a laxative on days 2 and 3 after receiving treatment to help excrete the contaminated stool faster.

7. A nurse teaches a client about prosthesis care after amputation. Which statements should the nurse include in this clients teaching? (Select all that apply.) a. The device has been custom made specifically for you. b. Your prosthetic is good for work but not for exercising. c. A prosthetist will clean your inserts for youNeUacRhSmINoGnTthB..COM d. Make sure that you wear the correct liners with your prosthetic. e. I have scheduled a follow-up appointment for you.

ANS: A, D, E A client with a new prosthetic should be taught that the prosthetic device is custom made for the client, taking into account the clients level of amputation, lifestyle (including exercise preferences), and occupation. In collaboration with a prosthetist, the client should be taught proper techniques for cleansing the sockets and inserts, wearing the correct liners, and assessing shoe wear. Follow-up care and appointments are important for ongoing assessment.

4. A nurse has applied to work at a hospital that has National Stroke Center designation. The nurse realizes the hospital adheres to eight Core Measures for ischemic stroke care. What do these Core Measures include? (Select all that apply.) a. Discharging the client on a statin medication b. Providing the client with comprehensive therapies c. Meeting goals for nutrition within 1 week d. Providing and charting stroke education e. Preventing venous thromboembolism

ANS: A, D, E Core Measures established by The Joint Commission include discharging stroke clients on statins, providing and recording stroke education, and taking measures to prevent venous thromboembolism. The client must be assessed for therapies but may go elsewhere for them. Nutrition goals are not part of the Core Measures.

1.A nurse assesses clients at a health fair. Which clients should the nurse counsel to be tested for diabetes? (Select all that apply.) a. 56-year-old African-American male b. Female with a 30-pound weight gain during pregnancy c. Male with a history of pancreatic trauma d. 48-year-old woman with a sedentary lifestyle e. Male with a body mass index greater than 25 kg/m2 f. 28-year-old female who gave birth to a baby weighing 9.2 pounds

ANS: A, D, E, F Risk factors for type 2 diabetes include certain ethnic/racial groups (African Americans, American Indians, Hispanics), obesity and physical inactivity, and giving birth to large babies. Pancreatic trauma and a 30-pound gestational weight gain are not risk factors.

5. A nurse cares for a client who is prescribed vasopressin (DDAVP) for diabetes insipidus. Which assessment findings indicate a therapeutic response to this therapy? (Select all that apply.) a. Urine output is increased. b. Urine output is decreased. c. Specific gravity is increased. d. Specific gravity is decreased. e. Urine osmolality is increased. f. Urine osmolality is decreased.

ANS: A, D, F Diabetes insipidus causes urine output to be greatly increased, with a low urine osmolality, as evidenced by a low specific gravity. Effective treatment results in decreased urine output that is more concentrated, as evidenced by an increased specific gravity.

1. A nurse plans care for a client with epilepsy who is admitted to the hospital. Which interventions should the nurse include in this clients plan of care? (Select all that apply.) a. Have suction equipment at the bedside. b. Place a padded tongue blade at the bedside. c. Permit only clear oral fluids. d. Keep bed rails up at all times. e. Maintain the client on strict bedrest. f. Ensure that the client has IV access.

ANS: A, D, F Oxygen and suctioning equipment with an airway must be readily available. The bed rails should be up at all times while the client is in the bed to prevent injury from a fall if the client has a seizure. If the client does not have an IV access, insert a saline lock, especially for those clients who are at significant risk for generalized tonic-clonic seizures. The saline lock provides ready access if IV drug therapy must be given to stop the seizure. Padded tongue blades may pose a danger to the client during a seizure and should not be used. Dietary restrictions and strict bedrest are not interventions associated with epilepsy. The client should be encouraged to eat a well-balanced diet and ambulate while in the hospital.

8. A nurse assesses a client who is recovering from an open traditional anterior cervical fusion. Which assessment findings would alert the nursing to a complication from this procedure? (Select all that apply.) a. Difficulty swallowing b. Hoarse voice c. Constipation d. Bradycardia e. Hypertension

ANS: A,B Complications of the open traditional anterior cervical discectomy and fusion include dysphagia and hoarseness. Constipation, bradycardia, and hypertension are not complications of this procedure.

5. The nurse takes a history on a male client reporting chronic back pain. Which factor(s) in the client's history may have contributed to his pain? (Select all that apply.) a. Had a motor vehicle crash 10 years ago. b. Played football in college and high school. c. Has installed carpet and other flooring for 30 years. d. Typically takes walks 3 to 4 days each week. e. Eats two servings of dark, green leafy vegetables daily.

ANS: A,B,C A history of trauma caused by an accident, occupation, or contact sports can result in chronic back pain. Regular exercise and diet helps to promote bone health.

5. The nurse is assessing a client admitted to the emergency department with possible retinal detachment. What assessment findings would the nurse expect? (Select all that apply.) a. Presence of bright light flashes b. Decreased visual field in affected eye c. Feeling like a curtain is over one eye d. Gradual changes in visual acuity e. Painful throbbing in the affected eye

ANS: A,B,C Changes that occur in clients experiencing retinal detachment are usually sudden and painless. Typical changes that occur include bright light flashes, sudden decrease in visual filed, and a feeling like a curtain is over all or part of the affected eye.

6. The nurse is caring for a client who has severe hypoglycemia and is experiencing a seizure. What actions will the nurse take at this time? (Select all that apply.) a. Administer glucagon 1 mg subcutaneously. b. Be sure the bed side rails are in the up position. c. Notify the primary health care provider immediately. d. Monitor the client's blood glucose level. e. Increase the intravenous infusion rate immediately.

ANS: A,B,C,D The client who has severe hypoglycemia often has a blood sugar of less than 20 mg/dL (1.0 mmol/L) and may be unconscious or seizing. Therefore, the client cannot swallow and needs glucagon. To keep the client safe during the seizure, the nurse ensures that the side rails are up to prevent the client from falling out of bed. The nurse would also monitor the client's blood sugar to evaluate the effectiveness of the interventions.

1. A female client is preparing to have open magnetic resonance imaging (MRI) of the spine. What action(s) by the nurse is (are) most important to assess before the test? (Select all that apply.) a. Ask if the client has a history of kidney disease. b. Ask the client if she could possibly be pregnant. c. Ensure that the patient has no metal or electronic implants. d. Assess the client for the ability to communicate. e. Assess the client for a history of claustrophobia.

ANS: A,B,C,D The contrast agent that is used for an MRI is gadolinium which can cause complications if the client is pregnant or has kidney disease. The client needs to be able to communicate and should not have any metal or electronic implants due to the magnetic nature of the machine. For an open MRI, claustrophobia is not an issue because the client is not encased in the device.

6. The nurse is teaching a client about care after surgery to repair a retinal detachment. What health teaching would the nurse include? (Select all that apply.) a. "Report sudden pain in the surgical eye." b. "Report if the surgical eye remains dilated." c. "Avoid close vision activities in the first week." d. "Avoid activities that increase intraocular pressure." e. "Report sudden reduced visual acuity."

ANS: A,B,C,D,E All of these instructions are important for the client who has a retinal detachment repair.

3. A client who had a recent total knee arthroplasty will be using a continuous passive motion (CPM) machine after discharge at home. What health teaching about the CPM machine will the nurse include? (Select all that apply.) a. "Keep the machine padded well to prevent skin breakdown." b. "Ensure that your leg is placed properly on the machine." c. "Use the machine as prescribed but not at mealtime." d. "When the machine is not being used, do not store it on the floor." e. "Check that the cycle and range of motion is kept at the level prescribed."

ANS: A,B,C,D,E Although not used as often today, some clients are prescribed to use the CPM machine to increase range of motion in the surgical knee. All of these teaching points are important for any client who uses a CPM machine.

6. The nurse is assessing a client with acute pyelonephritis. What assessment findings would the nurse expect? (Select all that apply.) a. Fever b. Chills c. Tachycardia d. Tachypnea e. Flank or back pain f. Fatigue

ANS: A,B,C,D,E,F All of these assessment findings commonly occur in clients who have acute pyelonephritis because this health problem is a kidney infection.

5. The nurse is teaching a family member who is caring for a client who is hearing impaired. What health teaching would the nurse include about communicating with the client? (Select all that apply.) a. "Make sure that the room is well lighted." b. "Speak slowly and clearly." c. "Do not shout but you may need to speak loudly." d. "Have conversations in a quiet room with minimal noise." e. "Get the client's attention before you begin to speak." f. "Move closer to the better hearing ear if possible."

ANS: A,B,C,D,E,F All of these recommendations are useful when communicating with clients who are hearing impaired.

4. The nurse is teaching a client and family regarding symptoms to report to the primary health care provider after cataract surgery. Which symptoms would the nurse include in the teaching? (Select all that apply.) a. Sharp sudden pain in the surgical eye b. Green or yellow discharge from the surgical eye c. Eyelid swelling of the surgical eye d. Decreased vision in the surgical eye e. Blindness in the surgical eye f. Flashes or floaters seen in the surgical eye

ANS: A,B,C,D,E,F All of these symptoms are not normal and should be reported immediately to the surgeon or other appropriate primary health care provider.

4. The nurse is caring for a client after ear surgery. What health teaching instruction(s) would the nurse provide for this client to promote healing? (Select all that apply.) a. "Avoid straining when having a bowel movement." b. "Avoid drinking through a straw for 2 to 3 weeks." c. "Avoid air travel for 2 to 3 weeks after surgery." d. "Avoid crowds and people with infection, especially respiratory infection." e. "Avoid moving your head quickly, jumping, or bending over for 2 to 3 weeks." f. "Blow your nose very gently without blocking either nostril and keep your mouth open."

ANS: A,B,C,D,E,F It is imperative that the patient having ear surgery is free from ear infection. The other precautions help to prevent increased intra-ear pressure which can affect the surgical procedure.

5. The nurse is teaching assistive personnel about postoperative care for an older adult who had a posterolateral total hip arthroplasty. What teaching will the nurse include? (Select all that apply.) a. "Move the client slowly to prevent dizziness and a possible fall." b. "Encourage the client to deep breathe and cough at least every 2 hours." c. "Help the client use the incentive spirometer at least every 2 hours." d. "Keep the abduction pillow in place at all times while the client is in bed." e. "Let me know if the client has an elevated temperature or pulse." f. "Keep in mind that the client may be a little confused after surgery." g. "Please let me know if you see any reddened or open skin areas during bathing."

ANS: A,B,C,D,E,F Older adults are at risk for complications of decreased mobility after surgery, including atelectasis, pneumonia, pressure injuries, and orthostatic hypotension. Therefore these precautions are to help keep the client safe and avoid complications that could be life threatening.

1. The nurse is assessing a client with long-term rheumatoid arthritis (RA) who has been taking prednisone for 10 years. For which complications of chronic drug therapy would the nurse assess? (Select all that apply.) a. Osteoporosis b. Diabetes mellitus c. Glaucoma d. Hypertension e. Hypokalemia f. Decreased immunity

ANS: A,B,C,D,E,F Prednisone is a corticosteroid that is sometimes used for autoimmune disorders like RA when other drugs are not effective or cannot be tolerated. However, it can cause many complications when used long-term, including all of the health problems listed in the choices.

8. A client has rheumatoid arthritis (RA) and the nurse is conducting a home assessment. What options can the nurse suggest for the client to maintain independence in activities of daily living (ADLs)? (Select all that apply.) a. Grab bars to reach high items b. Long-handled bath scrub brush c. Soft rocker-recliner chair d. Toothbrush with built-up handle e. Wheelchair cushion for comfort

ANS: A,B,D Grab bars, long-handled bath brushes, and toothbrushes with built-up handles all provide modifications for daily activities, making it easier for the client with RA to complete ADLs independently. The rocker-recliner and wheelchair cushion are comfort measures but do not help increase independence. Most clients who have RA are not wheelchair-bound.

7. The nurse is teaching a client about postoperative care after a LASIK procedure. Which common complications/adverse effects could occur either immediately or later after this type of surgery? (Select all that apply.) a. Halos around lights b. Blurred vision c. Blindness d. Infection e. Dry eyes

ANS: A,B,D,E All of these common problems can occur after LASIK surgery except for blindness. Some decrease in visual acuity can occur, however.

9. A nurse is visiting a client discharged home after a total hip arthroplasty. What safety precautions would the nurse recommend to the client and family? (Select all that apply.) a. Buy and install an elevated toilet seat. b. Install grab bars in the shower and by the toilet. c. Step into the bathtub with the affected leg first. d. Remove all throw rugs throughout the house. e. Use a shower chair while taking a shower.

ANS: A,B,D,E Buying and installing an elevated toilet seat, installing grab bars, removing throw rugs, and using a shower chair will all promote safety for this client. The client is still on partial weight bearing, so he or she cannot step into the bathtub leading with the operative side.

1. A nurse assesses a client with an injury to the medulla. Which clinical manifestations would the nurse expect to find? (Select all that apply.) a. Decreased respiratory rate b. Impaired swallowing c. Visual changes d. Inability to shrug shoulders e. Loss of gag reflex

ANS: A,B,D,E Cranial nerves IX (glossopharyngeal), X (vagus), XI (accessory), and XII (hypoglossal) emerge from the medulla, as do portions of cranial nerves VII (facial) and VIII (acoustic). Damage to these nerves causes decreased respirations, impaired swallowing, inability to shrug shoulders, and loss of the gag reflex. The other manifestations are not associated with damage to the medulla.

1. The nurse teaches assistive personnel about age-related changes that affect the eyes and vision. Which changes would the nurse include? (Select all that apply.) a. Decreased eye muscle tone b. Development of arcus senilis c. Increase in far point of near vision d. Decrease in general color perception e. Increase in point of near vision

ANS: A,B,D,E Normal age-related changes include decreased eye muscle tone, development of arcus senilis, decreased color perception, and increased point of near vision. The far point of near vision typically decreases.

4. A nurse prepares a client for a percutaneous kidney biopsy. What actions should the nurse take prior to this procedure? (Select all that apply.) a. Keep the client NPO for 4 to 6 hours. b. Review coagulation study results. c. Maintain strict bedrest in a supine position. d. Assess for blood in the client's urine. e. Administer client's antihypertensive medications.

ANS: A,B,E Prior to a percutaneous kidney biopsy, the patient should be NPO for 4 to 6 hours. Coagulation studies should be completed to prevent bleeding after the biopsy. Blood pressure medications should be administered to prevent hypertension before and after the procedure. There is no need to keep the patient on bedrest or assess for blood in the client's urine prior to the procedure; these interventions should be implemented after a percutaneous kidney biopsy.

1. A client is admitted with a possible diagnosis of diabetes insipidus (DI). What assessment findings would the nurse expect? (Select all that apply.) a. Hypotension b. Increased urinary output c. Concentrated urine d. Decreased thirst e. Poor skin turgor f. Bradycardia

ANS: A,B,E The client who has DI has excessive urination and dehydration. Clients who are dehydrated have decreased blood pressure, increased pulse (tachycardia), and poor skin turgor. The urine is dilute with a low specific gravity.

2. The nurse assesses a client who is experiencing a common migraine without an aura. Which assessment finding(s) would the nurse expect? (Select all that apply.) a. Headache lasting up to 72 hours b. Unilateral and pulsating headache c. Abrupt loss of consciousness d. Acute confusion e. Pain worsens with physical activities f. Photophobia

ANS: A,B,E,F A common migraine with an aura is usually accompanied by photophobia, phonophobia, unilateral and pulsating pain, and nausea and/or vomiting. These migraines usually last 4 to 72 hours and are aggravated by physical activity. Loss of consciousness and acute confusion are not associated with a common migraine without an aura.

2. The nurse is preparing a client for a percutaneous kidney biopsy. Which laboratory tests results would the nurse review prior to the procedure? (Select all that apply.) a. Hemoglobin b. Hematocrit c. Sodium d. Potassium e. Platelet count f. Prothrombin time

ANS: A,B,E,F Kidneys are very vascular and the client is at risk for bleeding after a biopsy. Therefore, it is essential that the nurse review preprocedure laboratory test results for anemia and coagulation problems.

5. A nurse plans care for an older adult patient. Which interventions should the nurse include in this client's plan of care to promote kidney health? (Select all that apply.) a. Ensure adequate fluid intake. b. Leave the bathroom light on at night. c. Encourage use of the toilet every 6 hours. d. Delegate bladder training instructions to the assistive personnel (AP). e. Provide thorough perineal care after each voiding. f. Assess for urinary retention and urinary tract infection.

ANS: A,B,E,F The nurse should ensure that the client receives adequate fluid intake and has adequate lighting to ambulate safely to the bathroom at night, encourage the client to use the toilet every 2 hours, provide thorough perineal care after each voiding, and assess for urinary retention and urinary tract infections. The nurse would not delegate any teaching to the AP, including bladder training instructions. The AP may participate in bladder training activities, including encouraging and assisting the client to the bathroom at specific times.

6. The nurse is caring for an older client who had a total knee arthroplasty. Prior to surgery, the client lived alone independently. With which interprofessional health care team members will the nurse collaborate to ensure positive client outcomes? (Select all that apply.) a. Case manager b. Mental health counselor c. Physical therapist d. Occupational therapist e. Speech-language pathologist f. Clergy/Spiritual leader

ANS: A,C The client was independent and living alone prior to surgery but will likely need help for a short time at home. However, if the client was ADL independent, he or she will not need referral to an occupational therapist. Therefore, a case manager can assess the living situation and identify any special needs to be addressed. The physical therapist will help the client learn to ambulate independently with a walker. There is no indication that the client needs referral for mental, spiritual, or speech-language services.

5. The nurse is reviewing the results of a client's urinalysis. The client has a diagnosis of acute glomerulonephritis. Which urine findings would the nurse expect? (Select all that apply.) a. Presence of protein b. Presence of red blood cells c. Presence of white blood cells d. Acidic urine e. Dilute urine

ANS: A,C,D The nurse would expect all of these findings except that the urine is usually concentrated with a high specific gravity.

8. A nurse teaches the spouse of a client who has Alzheimer disease. Which statements should the nurse include in this teaching related to caregiver stress reduction? (Select all that apply.) a. "Establish advanced directives early." b. "Trust that family and friends will help." c. "Set aside time each day to be away from the client." d. "Use discipline to correct inappropriate behaviors." e. "Seek respite care periodically for longer periods of time."

ANS: A,C,D To reduce caregiver stress, the spouse should be encouraged to establish advanced directives early, set aside time each day for rest or recreation away from the client, seek respite care periodically for longer periods of time, use humor with the client, and explore alternative care settings and resources. Family and friends may not be available to help. A structured environment will assist the client with AD, but discipline will not correct inappropriate behaviors and not reduce caregiver stress.

10. A nurse is planning postoperative care for a client following a total hip arthroplasty. What nursing interventions would help prevent venous thromboembolism for this client? (Select all that apply.) a. Early ambulation b. Fluid restriction c. Quadriceps-setting exercises d. Compression stockings/devices e. Anticoagulant drug therapy

ANS: A,C,D,E Early ambulation, leg exercises, and compression stockings/devices promote venous return and peripheral circulation which helps prevent deep vein thrombi. Anticoagulants such as subcutaneous low-molecular-weight heparin (LMWH) or factor Xa inhibitors are used for all clients who have a total lower extremity joint arthroplasty. The nurse would encourage fluids to expand blood volume and promote circulation; fluids would not be restricted.

3. The nurse is caring for a client who recently sustained a sports injury to his right leg. What nursing interventions are appropriate for this client? (Select all that apply.) a. Immobilize the right leg. b. Apply heat immediately after the injury. c. Use compression to support the leg. d. Obtain an x-ray to detect possible fracture. e. Elevate the right leg to decrease swelling. f. Administer an opioid every 4 to 6 hours.

ANS: A,C,D,E The client who experiences a sports injury should be managed using the RICE treatment plan. Rest, ice, compression, and elevation are all appropriate. Heat would increase swelling and probably pain. An x-ray would be obtained to determine if one or more fractures are present. Opioids may not be needed depending on the nature of the injury.

3. A nurse is caring for several clients in the morning prior to surgery. Which medications taken by the clients require the nurse to consult with the primary health care provider about their administration? (Select all that apply.) a. Insulin b. Omega-3 fatty acids c. Phenytoin d. Metoprolol e. Warfarin f. Prednisone

ANS: A,C,D,E,F Although the client will be on NPO status before surgery, the nurse should check with the primary health care provider about allowing the client to take medications prescribed for diabetes, hypertension, cardiac disease, seizure disorders, depression, glaucoma, anticoagulation, or depression and steroids. Metformin is used to treat diabetes; phenytoin is for seizures; metoprolol is for cardiac disease and/or hypertension; and warfarin is an anticoagulant. The omega-3 fatty acids can be held the day of surgery.

4. A client asks the nurse why she has urinary incontinence. What risk factors would the nurse recall in preparing to respond to the client's question? (Select all that apply.) a. Diuretic therapy b. Anorexia nervosa c. Stroke d. Dementia e. Arthritis f. Parkinson disease

ANS: A,C,D,E,F Drugs, such as diuretics, cause frequent voiding, often in large amounts. Diseases or disorders that limit mobility, such as stroke, arthritis, and Parkinson disease, can prevent an individual from getting to the bathroom in a timely manner. Mental/behavioral problems, such as dementia, impair cognition and the ability to recognize when he or she needs to void.

. 2. A client has a hearing aid. What care instructions does the nurse provide the assistive personnel (AP) in the care of this client? (Select all that apply.) a. "Be careful not to drop the hearing aid when handling." b. "Soak the hearing aid in hot water for 20 minutes." c. "Turn the hearing aid off when the client goes to bed." d. "Use a toothpick to clean debris from the device." e. "Wash the device with soap and a small amount of warm water." f. "Avoid using hair or cosmetic products near the hearing aid."

ANS: A,C,D,F All these actions except using water are proper instructions for the nurse to give to the AP.

1. A nurse promotes the prevention of lower back pain by teaching clients at a community center. Which statement(s) would the nurse include in this education? (Select all that apply.) a. "Participate in an exercise program to strengthen back muscles." b. "Purchase a mattress that allows you to adjust the firmness." c. "Wear flat instead of high-heeled shoes to work each day." d. "Keep your weight within 20% of your ideal body weight." e. "Avoid prolonged standing or sitting, including driving."

ANS: A,C,E Exercise can strengthen back muscles, reducing the incidence of low back pain. Women should avoid wearing high-heeled shoes because they cause misalignment of the back. Prolonged standing and sitting should also be avoided. The other options will not prevent low back pain.

3. A client with chronic osteomyelitis is being discharged from the hospital. What information is important for the nurse to teach this client and family? (Select all that apply.) a. Adherence to the antibiotic regimen b. Correct intramuscular injection technique c. Eating high-protein and high-carbohydrate foods d. Keeping daily follow-up appointments e. Proper use of the intravenous equipment

ANS: A,C,E The client going home with chronic osteomyelitis will need long-term antibiotic therapy—first intravenous, and then oral. The client needs education on how to properly administer IV antibiotics, care for the IV line, adhere to the regimen, and eat a healthy diet to encourage wound healing. The antibiotics are not given by IM injection. The client does not need daily follow-up.

9. The nurse is caring for a client who has Alzheimer disease. The client's wife states, "I am having trouble managing his behaviors at home." Which questions would the nurse ask to assess potential causes of the client's behavior problems? (Select all that apply.) a. "Does your husband bathe and dress himself independently?" b. "Do you weigh your husband each morning around the same time?" c. "Does his behavior become worse around large crowds?" d. "Does your husband eat healthy foods including fruits and vegetables?" e. "Do you have a clock and calendar in the bedroom and kitchen?"

ANS: A,C,E To minimize behavior problems, the nurse would encourage the patient to be as independent as possible with ADLs, minimize excessive simulation, and assist the patient to remain orientated. The nurse would assess these activities by asking if the patient is independent with bathing and dressing, if behavior worsens around crowds, and if a clock and single-date calendar are readily available. Diet and weight are not related to the management of behavior problems for a patient who has Alzheimer disease.

4. When caring for an older client who has hypothyroidism, what assessment findings will the nurse expect? (Select all that apply.) a. Lethargy b. Diarrhea c. Low body temperature d. Tachycardia e. Slowed speech f. Weight gain

ANS: A,C,E,F A client who has an underactive thyroid gland has a decreased metabolic rate, resulting in lethargy and lack of energy, weight gain, slowed speech, and decreased vital signs like a lowered body temperature. The client also typically has constipation (instead of diarrhea) due to slower peristalsis and bradycardia (instead of tachycardia).

3. The nurse is planning health teaching for a client starting mirabegron for urinary incontinence. What health teaching would the nurse include? (Select all that apply.) a. "Monitor blood tests carefully if you are prescribed warfarin." b. "Avoid crowds and individuals with infection." c. "Report any fever to your primary health care provider." d. "Take your blood pressure frequently at home." e. "Report palpitations or chest soreness that may occur."

ANS: A,D This drug can cause increase blood pressure and, therefore, the client's blood pressure should be monitored. Mirabegron can increase the effect of warfarin causing bleeding or bruising. The client will need additional coagulation studies to ensure that the INR is within a therapeutic range.

5. A client has received several doses of midazolam. The nurse assesses the client to be difficult to arouse with respirations of 6 breaths/min. What actions by the nurse are most important? (Select all that apply.) a. Administer oxygen per protocol. b. Obtain one dose of flumazenil. c. Obtain naloxone, 0.04 mg for IV push. d. Ensure suction is working e. Transfer the client to intensive care. f. Monitor client every 10 to 15 minutes for the next 2 hours.

ANS: A,D,E Midazolam is a benzodiazepine and its reversal agent is flumazenil. Naloxone is for opioid reversal. The nurse would apply oxygen as prescribed or by policy and obtain several doses at once because the drug can be given every 2 to 3 minutes if needed. Flumazenil can cause vomiting, so the nurse ensures suction equipment is present and working. Since flumazenil is metabolized more quickly than the midazolam, the client must be monitored every 10 to 15 minutes for the next 2 hours. The client may or may not need to be transferred.

10. The nurse is caring for a client in late-stage Alzheimer disease. Which assessment finding(s) will the nurse anticipate? (Select all that apply.) a. Immobile b. Has difficulty driving c. Wandering d. ADL dependent e. Incontinent f. Possible seizures

ANS: A,D,E,F The client in late-stage Alzheimer disease is totally bedridden and immobile, and therefore, cannot ambulate to wander or drive. The client is incontinent and ADL dependent.

2. The nurse is teaching a group of adults about ways to prevent early cataract formation. What health teaching would the nurse include? (Select all that apply.) a. "Wear eye and head protection when playing sports." b. "Be sure to get 7 to 8 hours of sleep each night." c. "Drink less carbonated beverages, especially those with caffeine." d. "Wear sunglasses when going outdoors or in ultraviolet light." e. "Increase consumption of high-protein, low-carbohydrate foods." f. "Avoid smoking or participate in a smoking cessation program."

ANS: A,D,F Although all of these choices are strategies for overall health promotion. Wearing eye and head protection and sunglasses, and avoiding or quitting smoking are specific strategies to promote eye health. Cataracts may occur earlier in a client's life if these recommendations are not followed.

7. After treating several young women for urinary tract infections (UTIs), the college nurse plans an educational offering on reducing the risk of getting a UTI. What information does the nurse include? (Select all that apply.) a. Void before and after each act of intercourse. b. Consider changing to spermicide from birth control pills. c. Do not douche or use scented feminine products. d. Wear loose-fitting nylon panties. e. Wipe or clean the perineum from front to back.

ANS: A,E Woman can reduce their risk of contracting UTIs by voiding before and after intercourse, not douching or using scented feminine products, and wiping from front to back. If spermicides are currently used, the woman should consider another form of birth control. Loose-fitting cotton underwear is best.

9. A nurse assesses cerebrospinal fluid leaking onto a client's surgical dressing. What actions would the nurse take? (Select all that apply.) a. Place the client in a flat position. b. Monitor vital signs for hypotension. c. Utilize a bedside commode. d. Assess for abdominal distension. e. Report the leak to the surgeon.

ANS: A,E If cerebrospinal fluid (CSF) is leaking from a surgical wound, the nurse would place the client in a flat position and contact the surgeon for repair of the leak. Hypotension and abdominal distension are not complications of CSF leakage.

20. The nurse is teaching the main principles of hemodialysis to a client with chronic kidney disease. Which statement by the client indicates a need for further teaching by the nurse? a. My sodium level changes by movement from the blood into the dialysate. b. Dialysis works by movement of wastes from lower to higher concentration. c. Extra fluid can be pulled from the blood by osmosis. d. The dialysate is similar to blood but without any toxins.

ANS: B Dialysis works using the passive transfer of toxins by diffusion. Diffusion is the movement of molecules from an area of higher concentration to an area of lower concentration. The other statements show a correct understanding about hemodialysis.

20. An emergency department nurse assesses a client with a history of urinary incontinence who presents with extreme dry mouth, constipation, and an inability to void. Which question should the nurse ask first? a. Are you drinking plenty of water? b. What medications are you taking? c. Have you tried laxatives or enemas? d. Has this type of thing ever happened before?

ANS: B Some types of incontinence are treated with anticholinergic medications such as propantheline (Pro-Banthine). Anticholinergic side effects include dry mouth, constipation, and urinary retention. The nurse needs to assess the clients medication list to determine whether the client is taking an anticholinergic medication. If he or she is taking anticholinergics, the nurse should further assess the clients manifestations to determine if they are related to a simple side effect or an overdose. The other questions are not as helpful to understanding the current situation.

17. A nurse on the postsurgical inpatient unit is observing a client perform leg exercises. What action by the client indicates a need for further instruction? a. Client states "This will help prevent blood clots in my legs." b. Bends both knees, pushes against the bed until calf and thigh muscles contract. c. Dorsiflexes and plantar flexes each foot several times an hour. d. Makes several clockwise then counterclockwise ankle circles with each foot.

ANS: B 18. A registered nurse (RN) is watching a new nurse change a dressing and perform care around a Penrose drain. What action by the new nurse warrants intervention? a. Cleaning around the drain per agency protocol b. Placing a new sterile gauze under the drain c. Securing the drain's safety pin to the sheets d. Using sterile technique to empty the drain* ANS: C The safety pin that prevents the drain from slipping back into the client's body would not be pinned to the client's bedding. Pinning it to the sheets will cause it to pull out when the client turns. The other actions are appropriate.

1. A client has a bone density score of 2.8. What action by the nurse is best? a. Asking the client to complete a food diary b. Planning to teach about bisphosphonates c. Scheduling another scan in 2 years d. Scheduling another scan in 6 months

ANS: B A T-score from a bone density scan at or lower than 2.5 indicates osteoporosis. The nurse should plan to teach about medications used to treat this disease. One class of such medications is bisphosphonates. A food diary is helpful to determine if the client gets adequate calcium and vitamin D, but at this point, dietary changes will not prevent the disease. Simply scheduling another scan will not help treat the disease either.

7. A client who had a complete spinal cord injury at level L5-S1 is admitted with a sacral pressure injury. What other assessment finding will the nurse anticipate for this client? a. Quadriplegia b. Flaccid bowel c. Spastic bladder d. Tetraparesis

ANS: B A low-level complete spinal cord injury (SCI) is a lower motor neuron injury because the reflect arc is damaged. Therefore, the client would be expected to have paraplegia and a flaccid bowel and bladder. Quadriplegia and tetraparesis are seen in clients with cervical or high thoracic SCIs.

10. A nurse cares for a client who has elevated levels of antidiuretic hormone (ADH). Which disorder should the nurse identify as a trigger for the release of this hormone? a. Pneumonia b. Dehydration c. Renal failure d. Edema

ANS: B ADH increases tubular permeability to water, leading to absorption of more water into the capillaries. ADH is triggered by a rising extracellular fluid osmolarity, as occurs in dehydration. Pneumonia, renal failure, and edema would not trigger the release of ADH.

9. The nurse is teaching assistive personnel (AP) about the risk for osteoporosis associated with race or ethnicity. Which population typically has a decreased incidence of osteoporosis when compared to Euro-Americans? a. Irish Americans b. African Americans c. American Indians d. Asian Americans

ANS: B African Americans usually have more bone mass when compared to Euro-Americans which makes them at a decreased risk for osteoporosis.

9. A client had a retinal detachment and has undergone surgical correction. What discharge health teaching is most important for the nurse to include? a. "Avoid reading, writing, or close work such as sewing." b. "Report immediate loss of vision of pain in the affected eye." c. "Keep the follow-up appointment with the ophthalmologist." d. "Remove your eye patch every hour for eyedrops."

ANS: B After surgery for retinal detachment, the client is advised to avoid reading, writing, and close work because these activities cause rapid eye movements. However, more importantly is the need for the client or family to report loss of vision or pain in the surgical eye. Keeping a postoperative appointment is important for any surgical patient. The eye patch is not removed for eyedrops after retinal detachment repair.

7. After teaching a client who is recovering from an endoscopic trans-nasal hypophysectomy, the nurse assesses the clients understanding. Which statement made by the client indicates a correct understanding of the teaching? a. I will wear dark glasses to prevent sun exposure. b. Ill keep food on upper shelves so I do not have to bend over. c. I must wash the incision with peroxide and redress it daily. d. I shall cough and deep breathe every 2 hours while I am awake.

ANS: B After this surgery, the client must take care to avoid activities that can increase intracranial pressure. The client should avoid bending from the waist and should not bear down, cough, or lie flat. With this approach, there is no incision to clean and dress. Protection from sun exposure is not necessary after this procedure.

43.A nurse prepares to administer prescribed regular and NPH insulin. Place the nurses actions in the correct order to administer these medications. 1. Inspect bottles for expiration dates. 2. Gently roll the bottle of NPH between the hands. 3. Wash your hands. 4. Inject air into the regular insulin. 5. Withdraw the NPH insulin. 6. Withdraw the regular insulin. 7. Inject air into the NPH bottle. 8. Clean rubber stoppers with an alcohol swab. inject first the glargine and then the regular insulin right afterward. a. 1, 3, 8, 2, 4, 6, 7, 5 b. 3, 1, 2, 8, 7, 4, 6, 5 c. 8, 1, 3, 2, 4, 6, 7, 5 d. 2, 3, 1, 8, 7, 5, 4, 6

ANS: B After washing hands, it is important to inspect the bottles and then to roll the NPH to mix the insulin. Rubber stoppers should be cleaned with alcohol after rolling the NPH and before sticking a needle into either bottle. It is important to inject air into the NPH bottle before placing the needle in a regular insulin bottle to avoid mixing of regular and NPH insulin. The shorter-acting insulin is always drawn up first.

14. A client is brought to the emergency department after a car crash. The client has a large piece of glass in the left eye. What action by the nurse takes priority? a. Administer a tetanus booster shot. b. Ensure that the client has a patent airway. c. Prepare to irrigate the client's eye. d. Turn the client on the unaffected side.

ANS: B Airway always comes first. After ensuring a patent airway and providing cervical spine precautions (do not turn the client to the side), the nurse provides other care that may include administering a tetanus shot. The client's eye may or may not be irrigated.

12. A nurse provides health screening for a community health center with a large population of African- American clients. Which priority assessment should the nurse include when working with this population? a. Measure height and weight. b. Assess blood pressure. c. Observe for any signs of abuse. d. Ask about medications.*

ANS: B All interventions are important for the visiting nurse to accomplish. However, African Americans have a high rate of hypertension leading to end-stage renal disease. Each encounter that the nurse has with an African- American client provides a chance to detect hypertension and treat it. If the client is already on antihypertensive medication, assessing blood pressure monitors therapy.

22. A client has an intraventricular catheter. What action by the nurse takes priority? a. Document intracranial pressure readings. b. Perform hand hygiene before client care. c. Measure intracranial pressure per hospital policy. d. Teach the client and family about the device.

ANS: B All of the actions are appropriate for this client However, performing hand hygiene takes priority because it prevents infection, which is a possibly devastating complication.

11. A nurse assesses a group of clients who have rheumatoid arthritis (RA). Which client would the nurse see first? a. Client who reports jaw pain when eating b. Client with a red, hot, swollen right wrist c. Client who has a puffy-looking area behind the knee d. Client with a worse joint deformity since the last visit

ANS: B All of the options are possible manifestations of RA. However, the presence of one joint that is much redder, hotter, or more swollen that the other joints may indicate infection or an exacerbation of the RA disease process. The nurse needs to see this client first.

8. A nurse obtains a focused health history for a client who is scheduled for magnetic resonance angiography. Which priority question should the nurse ask before the test? a. Have you had a recent blood transfusion? b. Do you have allergies to iodine or shellfish? c. Are you taking any cardiac medications? d. Do you currently use oral contraceptives?

ANS: B Allergies to iodine and/or shellfish need to be explored because the client may have a similar reaction to the dye used in the procedure. In some cases, the client may need to be medicated with antihistamines or steroids before the test is given. A recent blood transfusion or current use of cardiac medications or oral contraceptives would not affect the angiography.

12. A nurse cares for a client newly diagnosed with Graves disease. The clients mother asks, I have diabetes mellitus. Am I responsible for my daughters disease? How should the nurse respond? a. The fact that you have diabetes did not cause your daughter to have Graves disease. No connection is known between Graves disease and diabetes. b. An association has been noted between Graves disease and diabetes, but the fact that you have diabetes did not cause your daughter to have Graves disease. c. Graves disease is associated with autoimmune diseases such as rheumatoid arthritis, but not with a disease such as diabetes mellitus. d. Unfortunately, Graves disease is associated with diabetes, and your diabetes could have led to your daughter having Graves disease.

ANS: B An association between autoimmune diseases such as rheumatoid arthritis and diabetes mellitus has been noted. The predisposition is probably polygenic, and the mothers diabetes did not cause her daughters Graves disease. The other statements are inaccurate.

23.A nurse cares for a client who is diagnosed with acute rejection 2 months after receiving a simultaneous pancreas-kidney transplant. The client states, I was doing so well with my new organs, and the thought of having to go back to living on hemodialysis and taking insulin is so depressing. How should the nurse respond? a. Following the drug regimen more closely would have prevented this. b. One acute rejection episode does not mean that you will lose the new organs. c. Dialysis is a viable treatment option for you and may save your life. d. Since you are on the national registry, you can receive a second transplantation.

ANS: B An episode of acute rejection does not automatically mean that the client will lose the transplant. Pharmacologic manipulation of host immune responses at this time can limit damage to the organ and allow the graft to be maintained. The other statements either belittle the client or downplay his or her concerns. The client may not be a candidate for additional organ transplantation.

18. A nurse assesses a client with Huntington disease. Which motor changes should the nurse monitor for in this client? a. Shuffling gait b. Jerky hand movements c. Continuous chewing motions d. Tremors of the hands

ANS: B An imbalance between excitatory and inhibitory neurotransmitters leads to uninhibited motor movements, such as brisk, jerky, purposeless movements of the hands, face, tongue, and legs. Shuffling gait, continuous chewing motions, and tremors are associated with Parkinson disease.

2. A nurse reviews the laboratory findings of a client with a urinary tract infection. The laboratory report notes a shift to the left in a clients white blood cell count. Which action should the nurse take? a. Request that the laboratory perform a differential analysis on the white blood cells. b. Notify the provider and start an intravenous line for parenteral antibiotics. c. Collaborate with the unlicensed assistive personnel (UAP) to strain the clients urine for renal calculi. d. Assess the client for a potential allergic reaction and anaphylactic shock.

ANS: B An increase in band cells creates a shift to the left. A left shift most commonly occurs with urosepsis and is seen rarely with uncomplicated urinary tract infections. The nurse will be administering antibiotics, most likely via IV, so he or she should notify the provider and prepare to give the antibiotics. The shift to the left is part of a differential white blood cell count. The nurse would not need to strain urine for stones. Allergic reactions are associated with elevated eosinophil cells, not band cells.

14. A nurse teaches an older adult with a decreased production of estrogen. Which statement should the nurse include in this clients teaching to decrease injury? a. Drink at least 2 liters of fluids each day. b. Walk around the neighborhood for daily exercise. c. Bathe your perineal area twice a day. d. You should check your blood glucose before meals.

ANS: B An older adult client with decreased production of estrogen is at risk for decreased bone density and fractures. The nurse should encourage the client to participate in weight-bearing exercises such as walking. Drinking fluids and performing perineal care will decrease vaginal drying but not decrease injury. Older adults often have a decreased glucose tolerance, but this is not related to a decrease in estrogen.

2. A client is undergoing computed tomography (CT) of a joint. What action by the nurse is most important before the test? a. Administer sedation as prescribed. b. Assess for seafood or iodine allergy. c. Ensure that the client has no metal on the body. d. Provide preprocedure pain medication.

ANS: B Because CT uses iodine-based contrast material, the nurse assesses the client for allergies to iodine or seafood (which often contains iodine). The other actions are not needed.

1.A nurse is teaching a client with diabetes mellitus who asks, Why is it necessary to maintain my blood glucose levels no lower than about 60 mg/dL? How should the nurse respond? a. Glucose is the only fuel used by the body to produce the energy that it needs. b. Your brain needs a constant supply of glucose because it cannot store it. c. Without a minimum level of glucose, your body does not make red blood cells. d. Glucose in the blood prevents the formation of lactic acid and prevents acidosis.

ANS: B Because the brain cannot synthesize or store significant amounts of glucose, a continuous supply from the bodys circulation is needed to meet the fuel demands of the central nervous system. The nurse would want to educate the client to prevent hypoglycemia. The body can use other sources of fuel, including fat and protein, and glucose is not involved in the production of red blood cells. Glucose in the blood will encourage glucose metabolism but is not directly responsible for lactic acid formation.

1. A nurse is teaching a client who experiences migraine headaches and is prescribed a beta blocker. Which statement should the nurse include in this clients teaching? a. Take this drug only when you have prodromal symptoms indicating the onset of a migraine headache. b. Take this drug as ordered, even when feeling well, to prevent vascular changes associated with migraine headaches. c. This drug will relieve the pain during the aura phase soon after a headache has started. d. This medication will have no effect on your heart rate or blood pressure because you are taking it for migraines.

ANS: B Beta blockers are prescribed as prophylactic treatment to prevent the vascular changes that initiate migraine headaches. Heart rate and blood pressure will also be affected, and the client should monitor these side effects. The other responses do not discuss appropriate uses of the medication.

13. A client has been prescribed brinzolamide for glaucoma. What assessment by the nurse requires communication with the primary health care provider? a. Allergy to eggs b. Allergy to sulfonamides c. Use of contact lenses d. Use of beta blockers

ANS: B Brinzolamide is similar to sulfonamides, so an allergic reaction could occur. The other assessment findings are not related to brinzolamide.

2. A client who had a fractured ankle open reduction internal fixation (ORIF) 4 weeks ago reports burning pain and tingling in the affected foot. For which potential complication would the nurse anticipate? a. Delayed bone healing b. Complex regional pain syndrome c. Peripheral neuropathy d. Compartment syndrome

ANS: B Burning pain and tingling that occurs weeks or months after a fracture or other trauma may indicate complex regional pain syndrome. Compartment syndrome tends to occur within days of the initial injury.

27. A client is recovering from a kidney transplant. The clients urine output was 1500 mL over the last 12-hour period since transplantation. What is the priority assessment by the nurse? a. Checking skin turgor b. Taking blood pressure c. Assessing lung sounds d. Weighing the client

ANS: B By taking blood pressure, the nurse is assessing for hypotension that could compromise perfusion to the new kidney. The nurse then should notify the provider immediately. Skin turgor, lung sounds, and weight could give information about the fluid status of the client, but they are not the priority assessment.

22. A nurse assesses the health history of a client who is prescribed ziconotide (Prialt) for chronic back pain. Which assessment question should the nurse ask? a. Are you taking a nonsteroidal anti-inflammatory drug? b. Do you have a mental health disorder? c. Are you able to swallow medications? d. Do you smoke cigarettes or any illegal drugs?

ANS: B Clients who have a mental health or behavioral health problem should not take ziconotide. The other questions do not identify a contraindication for this medication.

9. A nurse assesses clients for potential endocrine disorders. Which client is at greatest risk for hyperparathyroidism? a. A 29-year-old female with pregnancy-induced hypertension b. A 41-year-old male receiving dialysis for end-stage kidney disease c. A 66-year-old female with moderate heart failure d. A 72-year-old male who is prescribed home oxygen therapy

ANS: B Clients who have chronic kidney disease do not completely activate vitamin D and poorly absorb calcium from the GI tract. They are chronically hypocalcemic, and this triggers overstimulation of the parathyroid glands. Pregnancy-induced hypertension, moderate heart failure, and home oxygen therapy do not place a client at higher risk for hyperparathyroidism.

5. A nurse teaches a client who is recovering from a spinal fusion. Which statement should the nurse include in this clients postoperative instructions? a. Only lift items that are 10 pounds or less. b. Wear your brace whenever you are out of bed. c. You must remain in bed for 3 weeks after surgery. d. You are prescribed medications to prevent rejection.

ANS: B Clients who undergo spinal fusion are fitted with a brace that they must wear throughout the healing process (usually 3 to 6 months) whenever they are out of bed. The client should not lift anything. The client does not need to remain in bed. Medications for rejection prevention are not necessary for this procedure.

16. A nurse teaches a client who is recovering from an open traditional cervical spinal fusion. Which statement would the nurse include in this client's postoperative instructions? a. "Only lift items that are 10 lb (4.5 kg) or less." b. "Wear your neck brace whenever you are out of bed." c. "You must remain in bed for 3 weeks after surgery." d. "You will be prescribed medications to prevent graft rejection."

ANS: B Clients who undergo spinal fusion are fitted with a neck brace that they must wear throughout the healing process whenever they are out of bed. The client should not lift anything more than 10 lb (4.5 kg). The client does not need to remain in bed. Medications for rejection prevention are not necessary for this procedure.

3. After teaching a client with early polycystic kidney disease (PKD) about nutritional therapy, the nurse assesses the clients understanding. Which statement made by the client indicates a correct understanding of the teaching? a. I will take a laxative every night before going to bed. b. I must increase my intake of dietary fiber and fluids. c. I shall only use salt when I am cooking my own food. d. Ill eat white bread to minimize gastrointestinal gas.

ANS: B Clients with PKD often have constipation, which can be managed with increased fiber, exercise, and drinking plenty of water. Laxatives should be used cautiously. Clients with PKD should be on a restricted salt diet, which includes not cooking with salt. White bread has a low fiber count and would not be included in a high- fiber diet.

17. A nurse plans care for a client who is recovering from a below-the-knee amputation of the left leg. Which intervention should the nurse include in this clients plan of care? a. Place pillows between the clients knees. b. Encourage range-of-motion exercises. c. Administer prophylactic antibiotics. d. Implement strict bedrest in a supine position.

ANS: B Clients with a below-the-knee amputation should complete range-of-motion exercises to prevent flexion contractions and prepare for a prosthesis. A pillow may be used under the limb as support. Clients recovering from this type of amputation are at low risk for infection and should not be prescribed prophylactic antibiotics. The client should be encouraged to re-position, move, and exercise frequently, and therefore should not be restricted to bedrest.

36.After teaching a client with type 2 diabetes mellitus, the nurse assesses the clients understanding. Which statement made by the client indicates a need for additional teaching? a. I need to have an annual appointment even if my glucose levels are in good control. b. Since my diabetes is controlled with diet and exercise, I must be seen only if I am sick. c. I can still develop complications even though I do not have to take insulin at this time. d. If I have surgery or get very ill, I may have to receive insulin injections for a short time.

ANS: B Clients with diabetes need to be seen at least annually to monitor for long-term complications, including visual changes, microalbuminuria, and lipid analysis. The client may develop complications and may need insulin in the future.

28.A nurse reviews the medication list of a client with a 20-year history of diabetes mellitus. The client holds up the bottle of prescribed duloxetine (Cymbalta) and states, My cousin has depression and is taking this drug. Do you think Im depressed? How should the nurse respond? a. Many people with long-term diabetes become depressed after a while. b. Its for peripheral neuropathy. Do you have burning pain in your feet or hands? c. This antidepressant also has anti-inflammatory properties for diabetic pain. d. No. Many medications can be used for several different disorders.

ANS: B Damage along nerves causes peripheral neuropathy and leads to burning pain along the nerves. Many drugs, including duloxetine (Cymbalta), can be used to treat peripheral neuropathy. The nurse should assess the client for this condition and then should provide an explanation of why this drug is being used. This medication, although it is used for depression, is not being used for that reason in this case. Duloxetine does not have anti- inflammatory properties. Telling the client that many medications are used for different disorders does not provide the client with enough information to be useful.

5. A client has been prescribed denosumab (Prolia). What instruction about this drug is most appropriate? a. Drink at least 8 ounces of water with it. b. Make appointments to come get your shot. c. Sit upright for 30 to 60 minutes after taking it. d. Take the drug on an empty stomach.

ANS: B Denosumab is given by subcutaneous injection twice a year. The client does not need to drink 8 ounces of water with this medication as it is not taken orally. The client does not need to remain upright for 30 to 60 minutes after taking this medication, nor does the client need to take the drug on an empty stomach.

16. A client with a traumatic brain injury is agitated and fighting the ventilator. What drug should the nurse prepare to administer? a. Carbamazepine (Tegretol) b. Dexmedetomidine (Precedex) c. Diazepam (Valium) d. Mannitol (Osmitrol)

ANS: B Dexmedetomidine is often used to manage agitation in the client with traumatic brain injury. Carbamazepine is an antiseizure drug. Diazepam is a benzodiazepine. Mannitol is an osmotic diuretic.

12.A nurse cares for a client with diabetes mellitus who asks, Why do I need to administer more than one injection of insulin each day? How should the nurse respond? a. You need to start with multiple injections until you become more proficient at self-injection. b. A single dose of insulin each day would not match your blood insulin levels and your food intake patterns. c. A regimen of a single dose of insulin injected each day would require that you eat fewer carbohydrates. d. A single dose of insulin would be too large to be absorbed, predictably putting you at risk for insulin shock.

ANS: B Even when a single injection of insulin contains a combined dose of different-acting insulin types, the timing of the actions and the timing of food intake may not match well enough to prevent wide variations in blood glucose levels. One dose of insulin would not be appropriate even if the client decreased carbohydrate intake. Additional injections are not required to allow the client practice with injections, nor will one dose increase the clients risk of insulin shock.

5. A client is being prepared for a mechanical embolectomy. What action by the nurse takes priority? a. Assess for contraindications to fibrinolytics. b. Ensure that informed consent is on the chart. c. Perform a full neurologic assessment. d. Review the clients medication lists.

ANS: B For this invasive procedure, the client needs to give informed consent. The nurse ensures that this is on the chart prior to the procedure beginning. Fibrinolytics are not used. A neurologic assessment and medication review are important, but the consent is the priority.

1. A nurse assesses clients for potential endocrine dysfunction. Which client is at greatest risk for a deficiency of gonadotropin and growth hormone? a. A 36-year-old female who has used oral contraceptives for 5 years b. A 42-year-old male who experienced head trauma 3 years ago c. A 55-year-old female with a severe allergy to shellfish and iodine d. A 64-year-old male with adult-onset diabetes mellitus

ANS: B Gonadotropin and growth hormone are anterior pituitary hormones. Head trauma is a common cause of anterior pituitary hypofunction. The other factors do not increase the risk of this condition.

7. After delegating care to an unlicensed assistive personnel (UAP) for a client who is prescribed habit training to manage incontinence, a nurse evaluates the UAPs understanding. Which action indicates the UAP needs additional teaching? a. Toileting the client after breakfast b. Changing the clients incontinence brief when wet c. Encouraging the client to drink fluids d. Recording the clients incontinence episodes

ANS: B Habit training is undermined by the use of absorbent incontinence briefs or pads. The nurse should re-educate the UAP on the technique of habit training. The UAP should continue to toilet the client after meals, encourage the client to drink fluids, and record incontinent episodes.

9. The nurse is caring for a client who is starting on propylthiouracil for hyperthyroidism. What statement by the client indicates a need for further teaching? a. "I will let my provider know if I have weight gain and cold intolerance." b. "I will let my provider know if I have a metallic taste or stomach upset." c. "I will avoid crowds and other people who have infection." d. "I am aware that if the drug changes the color of my urine, I should stop it."

ANS: B If the client's urine turns dark and/or the skin has a yellow appearance, the client may have possible liver toxicity from the drug. This is a serious adverse effect and needs to be reported to the primary health care provider after stopping the drug. If weight gain and cold intolerance occurs, then the client may need a lower dose of the drug. The drug should not cause GI distress or a metallic taste in his or her mouth.

6. After teaching a client with nephrotic syndrome and a normal glomerular filtration, the nurse assesses the clients understanding. Which statement made by the client indicates a correct understanding of the nutritional therapy for this condition? a. I must decrease my intake of fat. b. I will increase my intake of protein. c. A decreased intake of carbohydrates will be required. d. An increased intake of vitamin C is necessary.

ANS: B In nephrotic syndrome, the renal loss of protein is significant, leading to hypoalbuminemia and edema formation. If glomerular filtration is normal or near normal, increased protein loss should be matched by increased intake of protein. The client would not need to adjust fat, carbohydrates, or vitamins based on this disorder.

15. A nurse is caring for four clients who might be brain dead. Which client would best meet the criteria to allow assessment of brain death? a. Client with a core temperature of 95 F (35 C) for 2 days b. Client in a coma for 2 weeks from a motor vehicle crash c. Client who is found unresponsive in a remote area of a field by a hunter d. Client with a systolic blood pressure of 92 mm Hg since admission

ANS: B In order to determine brain death, clients must meet four criteria: 1) coma from a known cause, 2) normal or near-normal core temperature, 3) normal systolic blood pressure, and 4) at least one neurologic examination. The client who was in the car crash meets two of these criteria. The clients with the lower temperature and lower blood pressure have only one of these criteria. There is no data to support assessment of brain death in the client found by the hunter.

6. An inpatient nurse brings an informed consent form to a client for an operation scheduled for tomorrow. The client asks about possible complications from the operation. What response by the nurse is best? a. Answer the questions and document that teaching was done. b. Do not have the client sign the consent and call the surgeon. c. Have the client sign the consent, then call the surgeon. d. Remind the client of what teaching the surgeon has done.

ANS: B In order to give informed consent, the client needs sufficient information. Questions about potential complications should be answered by the surgeon. The nurse should notify the surgeon to come back and answer the clients questions before the client signs the consent form. The other actions are not appropriate.

6. A nurse cares for a client who is experiencing status epilepticus. Which prescribed medication should the nurse prepare to administer? a. Atenolol (Tenormin) b. Lorazepam (Ativan) c. Phenytoin (Dilantin) d. Lisinopril (Prinivil)

ANS: B Initially, intravenous lorazepam is administered to stop motor movements. This is followed by the administration of phenytoin. Atenolol, a beta blocker, and lisinopril, an angiotensin-converting enzyme inhibitor, are not administered for seizure activity. These medications are typically administered for hypertension and heart failure.

11. A nurse reviews a female clients laboratory results. Which results from the clients urinalysis should the nurse recognize as abnormal? a. pH 5.6 b. Ketone bodies present c. Specific gravity of 1.020 d. Clear and yellow color

ANS: B Ketone bodies are by-products of incomplete metabolism of fatty acids. Normally no ketones are present in urine. Ketone bodies are produced when fat sources are used instead of glucose to provide cellular energy. A pH between 4.6 and 8, specific gravity between 1.005 and 1.030, and clear yellow urine are normal findings for a female clients urinalysis.

1. A nurse assesses a client with hyperthyroidism who is prescribed lithium carbonate. Which assessment finding should alert the nurse to a side effect of this therapy? a. Blurred and double vision b. Increased thirst and urination c. Profuse nausea and diarrhea d. Decreased attention and insomnia

ANS: B Lithium antagonizes antidiuretic hormone and can cause symptoms of diabetes insipidus. This manifests with increased thirst and urination. Lithium has no effect on vision, gastric upset, or level of consciousness.

3. A nurse cares for a postmenopausal client who has had two episodes of bacterial urethritis in the last 6 months. The client asks, I never have urinary tract infections. Why is this happening now? How should the nurse respond? a. Your immune system becomes less effective as you age. b. Low estrogen levels can make the tissue more susceptible to infection. c. You should be more careful with your personal hygiene in this area. d. It is likely that you have an untreated sexually transmitted disease.

ANS: B Low estrogen levels decrease moisture and secretions in the perineal area and cause other tissue changes, predisposing it to the development of infection. Urethritis is most common in postmenopausal women for this reason. Although immune function does decrease with aging and sexually transmitted diseases are a known cause of urethritis, the most likely reason in this client is low estrogen levels. Personal hygiene usually does not contribute to this disease process.

9. An emergency department nurse cares for a client who experienced a spinal cord injury 1 hour ago. Which prescribed medication should the nurse prepare to administer? a. Intrathecal baclofen (Lioresal) b. Methylprednisolone (Medrol) c. Atropine sulfate d. Epinephrine (Adrenalin)

ANS: B Methylprednisolone (Medrol) should be given within 8 hours of the injury. Clients who receive this therapy usually show improvement in motor and sensory function. The other medications are inappropriate for this client.

9.A nurse teaches a client with type 2 diabetes mellitus who is prescribed glipizide (Glucotrol). Which statement should the nurse include in this clients teaching? a. Change positions slowly when you get out of bed. b. Avoid taking nonsteroidal anti-inflammatory drugs (NSAIDs). c. If you miss a dose of this drug, you can double the next dose. d. Discontinue the medication if you develop a urinary infection.

ANS: B NSAIDs potentiate the hypoglycemic effects of sulfonylurea agents. Glipizide is a sulfonylurea. The other statements are not applicable to glipizide.

35.A nurse cares for a client who has diabetes mellitus. The nurse administers 6 units of regular insulin and 10 units of NPH insulin at 0700. At which time should the nurse assess the client for potential problems related to the NPH insulin? a. 0800 b. 1600 c. 2000 d. 2300

ANS: B Neutral protamine Hagedorn (NPH) is an intermediate-acting insulin with an onset of 1.5 hours, peak of 4 to 12 hours, and duration of action of 22 hours. Checking the client at 0800 would be too soon. Checking the client at 2000 and 2300 would be too late. The nurse should check the client at 1600.

16. The nurse is assessing a client's medication profile to determine risk for tinnitus. Which drug classification is most likely to cause this health problem? a. Cephalosporins b. NSAIDs c. Beta-adrenergic blockers d. Osmotic diuretics

ANS: B None of these drug classifications except for NSAIDs pose a risk to clients for tinnitus as a side effect.

10. A client asks the nurse about having a total knee arthroplasty to relieve joint pain. Which factor would place the client at the highest risk for impaired postoperative healing? a. Controlled hypertension b. Obesity c. Osteoarthritis d. Mild osteopenia

ANS: B Obesity places a client at high risk for many postoperative complications including slower wound and bone healing. The other factors usually do not affect healing after surgery.

6. The nurse is assessing a group of clients for their risk of kidney disease. Which racial/ethnic group is at the greatest risk as they age? a. Latino Americans b. African Americans c. Jewish Americans d. Asian Americans

ANS: B Older African Americans have a greater age-related decrease in glomerular filtration rate when compared to other racial-ethnic groups. In addition, blood flow decreases and sodium excretion is less effective in older hypertensive African Americans. These changes make this group most at risk for kidney disease.

10. A nurse is caring for a client who was prescribed high-dose corticosteroid therapy for 1 month to treat a severe inflammatory condition. The clients symptoms have now resolved and the client asks, When can I stop taking these medications? How should the nurse respond? a. It is possible for the inflammation to recur if you stop the medication. b. Once you start corticosteroids, you have to be weaned off them. c. You must decrease the dose slowly so your hormones will work again. d. The drug suppresses your immune system, which must be built back up.

ANS: B One of the most common causes of adrenal insufficiency, a life-threatening problem, is the sudden cessation of long-term, high-dose corticosteroid therapy. This therapy suppresses the hypothalamic-pituitary-adrenal axis and must be withdrawn gradually to allow for pituitary production of adrenocorticotropic hormone and adrenal production of cortisol. Decreasing hormone therapy slowly ensures self-production of hormone, not hormone effectiveness. Building the clients immune system and rebound inflammation are not concerns related to stopping high-dose corticosteroids.

14. A client is scheduled for a percutaneous endoscopic lumbar discectomy. Which statement by the client indicates a need for further teaching? a. "I should have a lot less pain after surgery." b. "I'll be in the hospital for 2 to 3 days." c. "I should not have any major surgical complications." d. "I could possibly get an infection after surgery."

ANS: B Percutaneous endoscopic discectomy is a minimally invasive surgical procedure that requires a shorter hospital stay (23 hours or less) when compared to open traditional surgery. The risk for surgical complications is very low and clients experience less far pain from this procedure. However, due to interrupting skin integrity, infection may occur at the surgical site.

1. A nurse assesses a client with polycystic kidney disease (PKD). Which assessment finding should alert the nurse to immediately contact the health care provider? a. Flank pain b. Periorbital edema c. Bloody and cloudy urine d. Enlarged abdomen

ANS: B Periorbital edema would not be a finding related to PKD and should be investigated further. Flank pain and a distended or enlarged abdomen occur in PKD because the kidneys enlarge and displace other organs. Urine can be bloody or cloudy as a result of cyst rupture or infection.

1. The nurse is assessing a client with a diagnosis of pre-renal acute kidney injury (AKI). Which condition would the nurse expect to find in the clients recent history? a. Pyelonephritis b. Myocardial infarction c. Bladder cancer d. Kidney stones

ANS: B Pre-renal causes of AKI are related to a decrease in perfusion, such as with a myocardial infarction. Pyelonephritis is an intrinsic or intrarenal cause of AKI related to kidney damage. Bladder cancer and kidney stones are post-renal causes of AKI related to urine flow obstruction.

1. The nurse is assessing a client with a diagnosis of prerenal acute kidney injury (AKI). Which condition would the nurse expect to find in the patient's recent history? a. Pyelonephritis b. Dehydration c. Bladder cancer d. Kidney stones

ANS: B Prerenal causes of AKI are related to a decrease in perfusion, such as in clients who have prolonged dehydration. Pyelonephritis is an intrinsic or intrarenal cause of AKI related to kidney damage. Bladder cancer and kidney stones are postrenal causes of AKI related to urine flow obstruction.

29.A nurse assesses a client with diabetes mellitus. Which clinical manifestation should alert the nurse to decreased kidney function in this client? a. Urine specific gravity of 1.033 b. Presence of protein in the urine c. Elevated capillary blood glucose level d. Presence of ketone bodies in the urine

ANS: B Renal dysfunction often occurs in the client with diabetes. Proteinuria is a result of renal dysfunction. Specific gravity is elevated with dehydration. Elevated capillary blood glucose levels and ketones in the urine are consistent with diabetes mellitus but are not specific to renal function.

4. A nurse assesses a client with a history of epilepsy who experiences stiffening of the muscles of the arms and legs, followed by an immediate loss of consciousness and jerking of all extremities. How should the nurse document this activity? a. Atonic seizure b. Tonic-clonic seizure c. Myoclonic seizure d. Absence seizure

ANS: B Seizure activity that begins with stiffening of the arms and legs, followed by loss of consciousness and jerking of all extremities, is characteristic of a tonic-clonic seizure. An atonic seizure presents as a sudden loss of muscle tone followed by postictal confusion. A myoclonic seizure presents with a brief jerking or stiffening of extremities that may occur singly or in groups. Absence seizures present with automatisms, and the client is unaware of his or her environment.

16. A nurse cares for a client with advanced Alzheimers disease. The clients caregiver states, She is always wandering off. What can I do to manage this restless behavior? How should the nurse respond? a. This is a sign of fatigue. The client would benefit from a daily nap. b. Engage the client in scheduled activities throughout the day. c. It sounds like this is difficult for you. I will consult the social worker. d. The provider can prescribe a mild sedative for restlessness.

ANS: B Several strategies may be used to cope with restlessness and wandering. One strategy is to engage the client in structured activities. Another is to take the client for frequent walks. Daily naps and a mild sedative will not be as effective in the management of restless behavior. Consulting the social worker does not address the caregivers concern.

8.A nurse teaches a client about self-monitoring of blood glucose levels. Which statement should the nurse include in this clients teaching to prevent bloodborne infections? a. Wash your hands after completing each test. b. Do not share your monitoring equipment. c. Blot excess blood from the strip with a cotton ball. d. Use gloves when monitoring your blood glucose.

ANS: B Small particles of blood can adhere to the monitoring device, and infection can be transported from one user to another. Hepatitis B in particular can survive in a dried state for about a week. The client should be taught to avoid sharing any equipment, including the lancet holder. The client should be taught to wash his or her hands before testing. The client would not need to blot excess blood away from the strip or wear gloves.

7. A client has a great deal of pain when coughing and deep breathing after abdominal surgery despite having pain medication. What action by the nurse is best? a. Call the provider to request more analgesia. b. Demonstrate how to splint the incision. c. Have the client take shallower breaths. d. Tell the client a little pain is expected.

ANS: B Splinting an incision provides extra support during coughing and activity and helps decrease pain. If the client is otherwise comfortable, no more analgesia is required. Shallow breathing can lead to atelectasis and pneumonia. The client should know some pain is normal and expected after surgery, but that answer alone does not provide any interventions to help the client.

15. A nurse teaches a client with a cortisol deficiency who is prescribed prednisone (Deltasone). Which statement should the nurse include in this clients instructions? a. You will need to learn how to rotate the injection sites. b. If you work outside in the heat, you may need another drug. c. You need to follow a diet with strict sodium restrictions. d. Take one tablet in the morning and two tablets at night.

ANS: B Steroid dosage adjustment may be needed if the client works outdoors and might be difficult, especially in hot weather, when the client is sweating a great deal more than normal. Clients take prednisone orally, have no need for a salt restriction, and usually start the regimen with two tablets in the morning and one at night.

7. A student nurse is preparing morning medications for a client who had a stroke. The student plans to hold the docusate sodium (Colace) because the client had a large stool earlier. What action by the supervising nurse is best? a. Have the student ask the client if it is desired or not. b. Inform the student that the docusate should be given. c. Tell the student to document the rationale. d. Tell the student to give it unless the client refuses.

ANS: B Stool softeners should be given to clients with neurologic disorders in order to prevent an elevation in intracranial pressure that accompanies the Valsalva maneuver when constipated. The supervising nurse should instruct the student to administer the docusate. The other options are not appropriate. The medication could be held for diarrhea.

3. A nurse obtains a health history on a client prior to administering prescribed sumatriptan succinate (Imitrex) for migraine headaches. Which condition should alert the nurse to hold the medication and contact the health care provider? a. Bronchial asthma b. Prinzmetals angina c. Diabetes mellitus d. Chronic kidney disease

ANS: B Sumatriptan succinate effectively reduces pain and other associated symptoms of migraine headache by binding to serotonin receptors and triggering cranial vasoconstriction. Vasoconstrictive effects are not confined to the cranium and can cause coronary vasospasm in clients with Prinzmetals angina. The other conditions would not affect the clients treatment.

13. After teaching a client with renal cancer who is prescribed temsirolimus (Torisel), the nurse assesses the clients understanding. Which statement made by the client indicates a correct understanding of the teaching? a. I will take this medication with food and plenty of water. b. I shall keep my appointment at the infusion center each week. c. Ill limit my intake of green leafy vegetables while on this medication. d. I must not take this medication if I have an infection or am feeling ill.

ANS: B Temsirolimus is administered as a weekly intravenous infusion. This medication blocks protein that is needed for cell division and therefore inhibits cell cycle progression. This medication is not taken orally, and clients do not need to follow a specific diet.

3. A nurse cares for a male client with hypopituitarism who is prescribed testosterone hormone replacement therapy. The client asks, How long will I need to take this medication? How should the nurse respond? a. When your blood levels of testosterone are normal, the therapy is no longer needed. b. When your beard thickens and your voice deepens, the dose is decreased, but treatment will continue forever. c. When your sperm count is high enough to demonstrate fertility, you will no longer need this therapy. d. With age, testosterone levels naturally decrease, so the medication can be stopped when you are 50 years old.

ANS: B Testosterone therapy is initiated with high-dose testosterone derivatives and is continued until virilization is achieved. The dose is then decreased, but therapy continues throughout life. Therapy will continue throughout life; therefore, it will not be discontinued when blood levels are normal, at the age of 50 years, or when sperm counts are high.

10. A clients mean arterial pressure is 60 mm Hg and intracranial pressure is 20 mm Hg. Based on the clients cerebral perfusion pressure, what should the nurse anticipate for this client? a. Impending brain herniation b. Poor prognosis and cognitive function c. Probable complete recovery d. Unable to tell from this information

ANS: B The cerebral perfusion pressure (CPP) is the intracranial pressure subtracted from the mean arterial pressure: in this case, 60 20 = 40. For optimal outcomes, CPP should be at least 70 mm Hg. This client has very low CPP, which will probably lead to a poorer prognosis with significant cognitive dysfunction should the client survive. This data does not indicate impending brain herniation or complete recovery.

44.A nurse reviews the chart and new prescriptions for a client with diabetic ketoacidosis: Vital Signs and Assessment Blood pressure: 90/62 mm Hg Pulse: 120 beats/min Respiratory rate: 28 breaths/min Urine output: 20 mL/hr via catheter Which action should the nurse take? Laboratory Results Serum potassium: 2.6 mEq/L Medications Potassium chloride 40 mEq IV bolus STAT Increase IV fluid to 100 mL/hr a. Administer the potassium and then consult with the provider about the fluid order. b. Increase the intravenous rate and then consult with the provider about the potassium prescription. c. Administer the potassium first before increasing the infusion flow rate. d. Increase the intravenous flow rate before administering the potassium.

ANS: B The client is acutely ill and is severely dehydrated and hypokalemic. The client requires more IV fluids and potassium. However, potassium should not be infused unless the urine output is at least 30 mL/hr. The nurse should first increase the IV rate and then consult with the provider about the potassium.

41.A nurse reviews laboratory results for a client with diabetes mellitus who is prescribed an intensified insulin regimen: Fasting blood glucose: 75 mg/dL Postprandial blood glucose: 200 mg/dL Hemoglobin A1c level: 5.5% How should the nurse interpret these laboratory findings? a. Increased risk for developing ketoacidosis b. Good control of blood glucose c. Increased risk for developing hyperglycemia d. Signs of insulin resistance

ANS: B The client is maintaining blood glucose levels within the defined ranges for goals in an intensified regimen. Because the clients glycemic control is good, he or she is not at higher risk for ketoacidosis or hyperglycemia and is not showing signs of insulin resistance.

9. After teaching a client with hypertension secondary to renal disease, the nurse assesses the clients understanding. Which statement made by the client indicates a need for additional teaching? a. I can prevent more damage to my kidneys by managing my blood pressure. b. If I have increased urination at night, I need to drink less fluid during the day. c. I need to see the registered dietitian to discuss limiting my protein intake. d. It is important that I take my antihypertensive medications as directed.

ANS: B The client should not restrict fluids during the day due to increased urination at night. Clients with renal disease may be prescribed fluid restrictions. These clients should be assessed thoroughly for potential dehydration. Increased nocturnal voiding can be decreased by consuming fluids earlier in the day. Blood pressure control is needed to slow the progression of renal dysfunction. When dietary protein is restricted, refer the client to the registered dietitian as needed.

21. A nurse teaches a client who is starting urinary bladder training. Which statement should the nurse include in this clients teaching? a. Use the toilet when you first feel the urge, rather than at specific intervals. b. Try to consciously hold your urine until the scheduled toileting time. c. Initially try to use the toilet at least every half hour for the first 24 hours. d. The toileting interval can be increased once you have been continent for a week.

ANS: B The client should try to hold the urine consciously until the next scheduled toileting time. Toileting should occur at specific intervals during the training. The toileting interval should be no less than every hour. The interval can be increased once the client becomes comfortable with the interval.

2. A client with early-stage Alzheimer disease is admitted to the hospital with chest pain. Which nursing action is most appropriate to manage this client's dementia? a. Provide animal-assisted therapy as needed. b. Ensure a structured and consistent environment. c. Assist the client with activities of daily living (ADLs). d. Use validation therapy when communicating with the client.

ANS: B The client who has early Alzheimer disease (AD) does not require assistance with ADLs or validation therapy. While animal-assisted therapy may be helpful, some health care agencies do not allow this intervention. Therefore, the most appropriate action is to provide a structured and consistent environment while the client is hospitalized to prevent worsening of the client's symptoms.

20. A nurse cares for a client with a fractured fibula. Which assessment should alert the nurse to take immediate action? a. Pain of 4 on a scale of 0 to 10 b. Numbness in the extremity c. Swollen extremity at the injury site d. Feeling cold while lying in bed

ANS: B The client with numbness and/or tingling of the extremity may be displaying the first signs of acute compartment syndrome. This is an acute problem that requires immediate intervention because of possible decreased circulation. Moderate pain and swelling is an expected assessment after a fracture. These findings can be treated with comfort measures. Being cold can be treated with additional blankets or by increasing the temperature of the room.

24. A nurse cares for a client who had a long-leg cast applied last week. The client states, I cannot seem to catch my breath and I feel a bit light-headed. Which action should the nurse take next? a. Auscultate the clients lung fields anteriorly and posteriorly. b. Administer oxygen to keep saturations greater than 92%. c. Check the clients blood glucose level. d. Ask the client to take deep breaths.

ANS: B The clients symptoms are consistent with the development of pulmonary embolism caused by leg immobility in the long cast. The nurse should check the clients pulse oximetry reading and provide oxygen to keep saturations greater than 92%. Auscultating lung fields, checking blood glucose level, or deep breathing will not assist this client.

8. A nurse is giving a client instructions for showering with special antimicrobial soap the night before surgery. What instruction is most appropriate? a. After you wash the surgical site, shave that area with your own razor. b. Be sure to wash the area where you will have surgery very thoroughly. c. Use a washcloth to wash the surgical site; do not take a full shower or bath. d. Wash the surgical site first, then shampoo and wash the rest of your body.

ANS: B The entire proposed surgical site needs to be washed thoroughly and completely with the antimicrobial soap. Shaving, if absolutely necessary, should be done in the operative suite immediately before the operation begins, using sterile equipment. The client needs a full shower or bath (shower preferred). The client should wash the surgical site last; dirty water from shampooing will run over the cleansed site if the site is washed first.

1. A nurse assesses a client with a fracture who is being treated with skeletal traction. Which assessment should alert the nurse to urgently contact the health provider? a. Blood pressure increases to 130/86 mm Hg b. Traction weights are resting on the floor c. Oozing of clear fluid is noted at the pin site d. Capillary refill is less than 3 seconds

ANS: B The immediate action of the nurse should be to reapply the weights to give traction to the fracture. The health care provider must be notified that the weights were lying on the floor, and the client should be realigned in bed. The clients blood pressure is slightly elevated; this could be related to pain and muscle spasms resulting from lack of pressure to reduce the fracture. Oozing of clear fluid is normal, as is the capillary refill time.

24. A nurse assesses a male client who is recovering from a urologic procedure. Which assessment finding indicates an obstruction of urine flow? a. Severe pain b. Overflow incontinence c. Hypotension d. Blood-tinged urine

ANS: B The most common manifestation of urethral stricture after a urologic procedure is obstruction of urine flow. This rarely causes pain and has no impact on blood pressure. The client may experience overflow incontinence with the involuntary loss of urine when the bladder is distended. Blood in the urine is not a manifestation of the obstruction of urine flow.

6. The nurse is administering eyedrops to a client with an infection in the right eye. The drops go in both eyes, and two different bottles are used to administer the drops. The nurse accidentally uses the left eye bottle for the right eye. What action by the nurse is best? a. Inform the provider of the issue. b. Obtain a new bottle of eyedrops. c. Rinse the clients right eye thoroughly. d. Wipe the left eye bottle with alcohol.

ANS: B The nurse has contaminated the clean bottle by using it on the infected eye. The nurse needs to obtain a new bottle of solution to use on the left eye. The other actions are not appropriate.

19. A nurse assesses a client who is recovering from a lumbar puncture (LP). Which complication of this procedure should alert the nurse to urgently contact the health care provider? a. Weak pedal pulses b. Nausea and vomiting c. Increased thirst d. Hives on the chest

ANS: B The nurse should immediately contact the provider if the client experiences a severe headache, nausea, vomiting, photophobia, or a change in level of consciousness after an LP, which are all signs of increased intracranial pressure. Weak pedal pulses, increased thirst, and hives are not complications of an LP.

21. A client who had a traumatic above-the-knee amputation states that he fears he will never have an intimate relationship again. What is the nurse's best response? a. "You'll be able to get a leg prosthesis soon." b. "You think you won't be able to have sex again?" c. "I will ask the social worker to talk with you." d. "Are you married now or have a girl friend?"

ANS: B The nurse's response needs to allow further exploration of the client's feelings. Referring the client to another health professional might be appropriate at a later time but discounts the client's current feelings. Asking about marriage or a girlfriend assumes that the client is heterosexual.

22. A nurse cares for an older adult client with multiple fractures. Which action should the nurse take to manage this clients pain? a. Meperidine (Demerol) injections every 4 hours around the clock b. Patient-controlled analgesia (PCA) pump with morphine c. Ibuprofen (Motrin) 600 mg orally every 4 hours PRN for pain d. Morphine 4 mg intravenous push every 2 hours PRN for pain

ANS: B The older adult client should never be treated with meperidine because toxic metabolites can cause seizures. The client should be managed with a PCA pump to control pain best. Motrin most likely would not provide complete pain relief with multiple fractures. IV morphine PRN would not control pain as well as a pump that the client can control.

9. A postoperative client has an abdominal drain. What assessment by the nurse indicates that goals for the priority client problems related to the drain are being met? a. Drainage from the surgical site is 30 mL less than yesterday. b. There is no redness, warmth, or drainage at the insertion site. c. The client reports adequate pain control with medications. d. Urine is clear yellow and urine output is greater than 40 mL/hr.

ANS: B The priority client problem related to a surgical drain is the potential for infection. An insertion site that is free of redness, warmth, and drainage indicates that goals for this client problem are being met. The other assessments are normal, but not related to the drain.

12. A nurse assesses a client who is recovering from extracorporeal shock wave lithotripsy for renal calculi. The nurse notes an ecchymotic area on the clients right lower back. Which action should the nurse take? a. Administer fresh-frozen plasma. b. Apply an ice pack to the site. c. Place the client in the prone position. d. Obtain serum coagulation test results.

ANS: B The shock waves from lithotripsy can cause bleeding into the tissues through which the waves pass. Application of ice can reduce the extent and discomfort of the bruising. Although coagulation test results and fresh-frozen plasma are used to assess and treat bleeding disorders, ecchymosis after this procedure is not unusual and does not warrant a higher level of intervention. Changing the clients position will not decrease bleeding.

11.A nurse cares for a client who is prescribed pioglitazone (Actos). After 6 months of therapy, the client reports that his urine has become darker since starting the medication. Which action should the nurse take? a. Assess for pain or burning with urination. b. Review the clients liver function study results. c. Instruct the client to increase water intake. d. Test a sample of urine for occult blood.

ANS: B Thiazolidinediones (including pioglitazone) can affect liver function; liver function should be assessed at the start of therapy and at regular intervals while the client continues to take these drugs. Dark urine is one indicator of liver impairment because bilirubin is increased in the blood and is excreted in the urine. The nurse should check the clients most recent liver function studies. The nurse does not need to assess for pain or burning with urination and does not need to check the urine for occult blood. The client does not need to be told to increase water intake.

19. After a craniotomy, the nurse assesses the client and finds dry, sticky mucous membranes and restlessness. The client has IV fluids running at 75 mL/hr. What action by the nurse is best? a. Assess the clients magnesium level. b. Assess the clients sodium level. c. Increase the rate of the IV infusion. d. Provide oral care every hour.

ANS: B This client has manifestations of hypernatremia, which is a possible complication after craniotomy. The nurse should assess the clients serum sodium level. Magnesium level is not related. The nurse does not independently increase the rate of the IV infusion. Providing oral care is also a good option but does not take priority over assessing laboratory results.

4. A client is admitted with acute kidney injury (AKI) and a urine output of 2000 mL/day. What is the major concern of the nurse regarding this clients care? a. Edema and pain b. Electrolyte and fluid imbalance c. Cardiac and respiratory status d. Mental health status

ANS: B This client may have an inflammatory cause of AKI with proteins entering the glomerulus and holding the fluid in the filtrate, causing polyuria. Electrolyte loss and fluid balance is essential. Edema and pain are not usually a problem with fluid loss. There could be changes in the clients cardiac, respiratory, and mental health status if the electrolyte imbalance is not treated.

1. A client is in the emergency department reporting a brief episode during which he was dizzy, unable to speak, and felt like his legs were very heavy. Currently the clients neurologic examination is normal. About what drug should the nurse plan to teach the client? a. Alteplase (Activase) b. Clopidogrel (Plavix) c. Heparin sodium d. Mannitol (Osmitrol)

ANS: B This clients manifestations are consistent with a transient ischemic attack, and the client would be prescribed aspirin or clopidogrel on discharge. Alteplase is used for ischemic stroke. Heparin and mannitol are not used for this condition.

14. A client has scoliosis with a 65-degree curve to the spine. What action by the nurse takes priority? a. Allow the client to rest in a position of comfort. b. Assess the clients cardiac and respiratory systems. c. Assist the client with ambulating and position changes. d. Position the client on one side propped with pillows.

ANS: B This degree of curvature of the spine affects cardiac and respiratory function. The nurses priority is to assess those systems. Positioning is up to the client. The client may or may not need assistance with movement.

1. A client is having a myelography. What action by the nurse is most important? a. Assess serum aspartate aminotransferase (AST) levels. b. Ensure that informed consent is on the chart. c. Position the client flat after the procedure. d. Reinforce the dressing if it becomes saturated.

ANS: B This diagnostic procedure is invasive and requires informed consent. The AST does not need to be assessed prior to the procedure. The client is positioned with the head of the bed elevated after the test to keep the contrast material out of the brain. The dressing should not become saturated; if it does, the nurse calls the provider.

9. A nurse cares for a client with a urine specific gravity of 1.018. Which action should the nurse take? a. Evaluate the clients intake and output for the past 24 hours. b. Document the finding in the chart and continue to monitor. c. Obtain a specimen for a urine culture and sensitivity. d. Encourage the client to drink more fluids, especially water.

ANS: B This specific gravity is within the normal range for urine. There is no need to evaluate the clients intake and output, obtain a urine specimen, or increase fluid intake.

14. A postoperative client vomited. After cleaning and comforting the client, which action by the nurse is most important? a. Allow the client to rest. b. Auscultate lung sounds. c. Document the episode. d. Encourage the client to eat dry toast.

ANS: B Vomiting after surgery has several complications, including aspiration. The nurse would listen to the client's lung sounds. The client should be allowed to rest after an assessment. Documenting is important, but the nurse needs to be able to document fully, including an assessment. The client should not eat until nausea has subsided.

33.A nurse teaches a client with diabetes mellitus about sick day management. Which statement should the nurse include in this clients teaching? a. When ill, avoid eating or drinking to reduce vomiting and diarrhea. b. Monitor your blood glucose levels at least every 4 hours while sick. c. If vomiting, do not use insulin or take your oral antidiabetic agent. d. Try to continue your prescribed exercise regimen even if you are sick.

ANS: B When ill, the client should monitor his or her blood glucose at least every 4 hours. The client should continue taking the medication regimen while ill. The client should continue to eat and drink as tolerated but should not exercise while sick.

19.A nurse assesses a client who has diabetes mellitus. Which arterial blood gas values should the nurse identify as potential ketoacidosis in this client? a. pH 7.38, HCO3 22 mEq/L, PCO2 38 mm Hg, PO2 98 mm Hg b.pH7.28,HCO 18mEq/L,PCO 28mmHg,PO 98mmHg 322 c. pH 7.48, HCO3 28 mEq/L, PCO2 38 mm Hg, PO2 98 mm Hg d. pH 7.32, HCO3 22 mEq/L, PCO2 58 mm Hg, PO2 88 mm Hg

ANS: B When the lungs can no longer offset acidosis, the pH decreases to below normal. A client who has diabetic ketoacidosis would present with arterial blood gas values that show primary metabolic acidosis with decreased bicarbonate levels and a compensatory respiratory alkalosis with decreased carbon dioxide levels.

8. A nurse cares for a client who possibly has syndrome of inappropriate antidiuretic hormone (SIADH). The clients serum sodium level is 114 mEq/L. Which action should the nurse take first? a. Consult with the dietitian about increased dietary sodium. b. Restrict the clients fluid intake to 600 mL/day. c. Handle the client gently by using turn sheets for re-positioning. d. Instruct unlicensed assistive personnel to measure intake and output.

ANS: B With SIADH, clients often have dilutional hyponatremia. The client needs a fluid restriction, sometimes to as little as 500 to 600 mL/24 hr. Adding sodium to the clients diet will not help if he or she is retaining fluid and diluting the sodium. The client is not at increased risk for fracture, so gentle handling is not an issue. The client should be on intake and output; however, this will monitor only the clients intake, so it is not the best answer. Reducing intake will help increase the clients sodium.

16. A nurse is teaching a client with multiple sclerosis who is prescribed cyclophosphamide (Cytoxan) and methylprednisolone (Medrol). Which statement should the nurse include in this clients discharge teaching? a. Take warm baths to promote muscle relaxation. b. Avoid crowds and people with colds. c. Relying on a walker will weaken your gait. d. Take prescribed medications when symptoms occur.

ANS: B The client should be taught to avoid people with any type of upper respiratory illness because these medications are immunosuppressive. Warm baths will exacerbate the clients symptoms. Assistive devices may be required for safe ambulation. Medication should be taken at all times and should not be stopped.

3. A client with Pagets disease is hospitalized for an unrelated issue. The client reports pain and it is not yet time for more medication. What comfort measures can the nurse delegate to the unlicensed assistive personnel (UAP)? (Select all that apply.) a. Administering ibuprofen (Motrin) b. Applying a heating pad c. Providing a massage d. Referring the client to a support group e. Using a bed cradle to lift sheets off the feet

ANS: B, C Comfort measures for Pagets disease include heat and massage. Administering medications and referrals are done by the nurse. A bed cradle is not necessary.

2. A client has Mnires disease with frequent attacks. About what drugs does the nurse plan to teach the client? (Select all that apply.) a. Broad-spectrum antibiotics b. Chlorpromazine hydrochloride (Thorazine) c. Diphenhydramine (Benadryl) d. Meclizine (Antivert) e. Nonsteroidal anti-inflammatory drugs (NSAIDs)

ANS: B, C, D Drugs such as chlorpromazine, diphenhydramine, and meclizine can all be used to treat Mnires disease. Antibiotics and NSAIDs are not used.

2. A nurse assesses clients on the medical-surgical unit. Which clients are at risk for kidney problems? (Select all that apply.) a. A 24-year-old pregnant woman prescribed prenatal vitamins b. A 32-year-old bodybuilder taking synthetic creatine supplements c. A 56-year-old who is taking metformin for diabetes mellitus d. A 68-year-old taking high-dose nonsteroidal anti-inflammatory drugs (NSAIDs) for chronic back pain e. A 75-year-old with chronic obstructive pulmonary disease (COPD) who is prescribed an albuterol nebulizer

ANS: B, C, D Many medications can affect kidney function. Clients who take synthetic creatine supplements, metformin, and high-dose or long-term NSAIDs are at risk for kidney dysfunction. Prenatal vitamins and albuterol nebulizers do not place these clients at risk.

5. A nurse is giving discharge instructions to a client recently diagnosed with chronic kidney disease (CKD). Which statements made by the client indicate a correct understanding of the teaching? (Select all that apply.) a. I can continue to take antacids to relieve heartburn. b. I need to ask for an antibiotic when scheduling a dental appointment. c. Ill need to check my blood sugar often to prevent hypoglycemia. d. The dose of my pain medication may have to be adjusted. e. I should watch for bleeding when taking my anticoagulants.

ANS: B, C, D, E In discharge teaching, the nurse must emphasize that the client needs to have an antibiotic prophylactically before dental procedures to prevent infection. There may be a need for dose reduction in medications if the kidney is not excreting them properly (antacids with magnesium, antibiotics, antidiabetic drugs, insulin, opioids, and anticoagulants).

6. A client is scheduled for a stapedectomy in 2 weeks. What teaching instructions are most appropriate? (Select all that apply.) a. Avoid alcohol use before surgery. b. Blow the nose gently if needed. c. Clean the telephone often. d. Sneeze with the mouth open. e. Wash the external ear daily.

ANS: B, C, D, E It is imperative that the client having a stapedectomy is free from ear infection. Teaching includes ways to prevent such infections, such as blowing the nose gently, cleaning objects that come into contact with the ear, sneezing with the mouth open, and washing the external ear daily. Avoiding alcohol will not help prevent ear infections.

8. A nurse working in the preoperative holding area performs which functions to ensure client safety? (Select all that apply.) a. Allow small sips of plain water. b. Check that consent is on the chart. c. Ensure the client has an armband on. d. Have the client help mark the surgical site. e. Allow the client to use the toilet before giving sedation.

ANS: B, C, D, E Providing for client safety is a priority function of the preoperative nurse. Checking for appropriately completed consent, verifying the clients identity, having the client assist in marking the surgical site if applicable, and allowing the client to use the toilet prior to sedating him or her are just some examples of important safety measures. The preoperative client should be NPO, so water should not be provided.

1. A nurse assesses a client who has a family history of polycystic kidney disease (PKD). For which clinical manifestations should the nurse assess? (Select all that apply.) a. Nocturia b. Flank pain c. Increased abdominal girth d. Dysuria e. Hematuria f. Diarrhea

ANS: B, C, E Clients with PKD experience abdominal distention that manifests as flank pain and increased abdominal girth. Bloody urine is also present with tissue damage secondary to PKD. Clients with PKD often experience constipation, but would not report nocturia or dysuria.

2. An emergency nurse assesses a client who is admitted with a pelvic fracture. Which assessments should the nurse monitor to prevent a complication of this injury? (Select all that apply.) a. Temperature b. Urinary output c. Blood pressure d. Pupil reaction e. Skin color

ANS: B, C, E With a pelvic fracture, internal organ damage may result in bleeding and hypovolemic shock. The nurse monitors the clients heart rate, blood pressure, urine output, skin color, and level of consciousness frequently to determine whether shock is manifesting. It is important to monitor the urine for blood to assess whether the urinary system has been damaged with the pelvic fracture. Changes in temperature and pupil reactions are not directly associated with hypovolemic shock. Temperature changes are usually associated with hypo- or hyperthermia or infectious processes. Pupillary changes occur with brain injuries, bleeds, or neurovascular accidents.

3. After administering a medication that stimulates the sympathetic division of the autonomic nervous system, the nurse assesses the client. For which clinical manifestations should the nurse assess? (Select all that apply.) a. Decreased respiratory rate b. Increased heart rate c. Decreased level of consciousness d. Increased force of contraction e. Decreased blood pressure

ANS: B, D Stimulation of the sympathetic nervous system initiates the fight-or-flight response, increasing both the heart rate and the force of contraction. A medication that stimulates the sympathetic nervous system would also increase the clients respiratory rate, blood pressure, and level of consciousness.

1. A nurse evaluates the following laboratory results for a client who has hypoparathyroidism: Based on these results, which medications should the nurse anticipate administering? (Select all that apply.) Calcium 7.2 mg/dL Sodium 144 mEq/L Magnesium 1.2 mEq/L Potassium 5.7 mEq/L a. Oral potassium chloride b. Intravenous calcium chloride c. 3% normal saline IV solution d. 50% magnesium sulfate e. Oral calcitriol (Rocaltrol)

ANS: B, D The client has hypocalcemia (treated with calcium chloride) and hypomagnesemia (treated with magnesium sulfate). The potassium level is high, so replacement is not needed. The clients sodium level is normal, so hypertonic IV solution is not needed. No information about a vitamin D deficiency is evident, so calcitriol is not needed.

11. A nurse is dismissing a client from the emergency department who has a mild traumatic brain injury. What information obtained from the client represents a possible barrier to self-management? (Select all that apply.) a. Does not want to purchase a thermometer b. Is allergic to acetaminophen (Tylenol) c. Laughing, says Strenuous? Whats that? d. Lives alone and is new in town with no friends e. Plans to have a beer and go to bed once home

ANS: B, D, E Clients should take acetaminophen for headache. An allergy to this drug may mean the client takes aspirin or ibuprofen (Motrin), which should be avoided. The client needs neurologic checks every 1 to 2 hours, and this client does not seem to have anyone available who can do that. Alcohol needs to be avoided for at least 24 hours. A thermometer is not needed. The client laughing at strenuous activity probably does not engage in any kind of strenuous activity, but the nurse should confirm this.

1. A nurse assesses a client with an injury to the medulla. Which clinical manifestations should the nurse expect to find? (Select all that apply.) a. Loss of smell b. Impaired swallowing c. Visual changes d. Inability to shrug shoulders e. Loss of gag reflex

ANS: B, D, E Cranial nerves IX (glossopharyngeal), X (vagus), XI (accessory), and XII (hypoglossal) emerge from the medulla, as do portions of cranial nerves VII (facial) and VIII (acoustic). Damage to these nerves causes impaired swallowing, inability to shrug shoulders, and loss of the gag reflex. The other manifestations are not associated with damage to the medulla.

10. A nurse assesses a client who has encephalitis. Which manifestations should the nurse recognize as signs of increased intracranial pressure (ICP), a complication of encephalitis? (Select all that apply.) a. Photophobia b. Dilated pupils c. Headache d. Widened pulse pressure e. Bradycardia

ANS: B, D, E Increased ICP is a complication of encephalitis. The nurse should monitor for signs of increased ICP, including dilated pupils, widened pulse pressure, bradycardia, irregular respirations, and less responsive pupils. Photophobia and headache are not related to increased ICP.

2. A nurse is teaching a client who has chronic headaches. Which statements about headache triggers should the nurse include in this clients plan of care? (Select all that apply.) a. Increase your intake of caffeinated beverages. b. Incorporate physical exercise into your daily routine. c. Avoid all alcoholic beverages. d. Participate in a smoking cessation program. e. Increase your intake of fruits and vegetables.

ANS: B, D, E Triggers for headaches include caffeine, smoking, and ingestion of pickled foods, so these factors should be avoided. Clients are taught to eat a balanced diet and to get adequate exercise and rest. Alcohol does not trigger chronic headaches but can enhance headaches during the headache period.

4. The nurse is teaching a client with diabetes mellitus how to prevent or delay chronic kidney disease (CKD). Which client statements indicate a lack of understanding of the teaching? (Select all that apply.) a. I need to decrease sodium, cholesterol, and protein in my diet. b. My weight should be maintained at a body mass index of 30. c. Smoking should be stopped as soon as I possibly can. d. I can continue to take an aspirin every 4 to 8 hours for my pain. e. I really only need to drink a couple of glasses of water each day.

ANS: B, D, E Weight should be maintained at a body mass index (BMI) of 22 to 25. A BMI of 30 indicates obesity. The use of nonsteroidal anti-inflammatory drugs such as aspirin should be limited to the lowest time at the lowest dose due to interference with kidney blood flow. The client should drink at least 2 liters of water daily. Diet adjustments should be made by restricting sodium, cholesterol, and protein. Smoking causes constriction of blood vessels and decreases kidney perfusion, so the client should stop smoking.

4. A nurse cares for a client with a lower motor neuron injury who is experiencing a flaccid bowel elimination pattern. Which actions should the nurse take to assist in relieving this clients constipation? (Select all that apply.) a. Pour warm water over the perineum. b. Provide a diet high in fluids and fiber. c. Administer daily tap water enemas. d. Implement a consistent daily time for elimination. e. Massage the abdomen from left to right. f. Perform manual disimpaction.

ANS: B, D, F For the client with a lower motor neuron injury, the resulting flaccid bowel may require a bowel program for the client that includes stool softeners, increased fluid intake, a high-fiber diet, and a consistent elimination time. If the client becomes impacted, the nurse would need to perform manual disimpaction. Pouring warm water over the perineum, administering daily enemas, and massaging the abdomen would not assist this client.

4. A nurse assesses a client with a fungal urinary tract infection (UTI). Which assessments should the nurse complete? (Select all that apply.) a. Palpate the kidneys and bladder. b. Assess the medical history and current medical problems. c. Perform a bladder scan to assess post-void residual. d. Inquire about recent travel to foreign countries. e. Obtain a current list of medications.

ANS: B, E Clients who are severely immunocompromised or who have diabetes mellitus are more prone to fungal UTIs. The nurse should assess for these factors by asking about medical history, current medical problems, and the current medication list. A physical examination and a post-void residual may be needed, but not until further information is obtained indicating that these examinations are necessary. Travel to foreign countries probably would not be important because, even if exposed, the client needs some degree of compromised immunity to develop a fungal UTI.

3. A nurse teaches clients about the difference between urge incontinence and stress incontinence. Which statements should the nurse include in this education? (Select all that apply.) a. Urge incontinence involves a post-void residual volume less than 50 mL. b. Stress incontinence occurs due to weak pelvic floor muscles. c. Stress incontinence usually occurs in people with dementia. d. Urge incontinence can be managed by increasing fluid intake. e. Urge incontinence occurs due to abnormal bladder contractions.

ANS: B, E Clients who suffer from stress incontinence have weak pelvic floor muscles or urethral sphincter and cannot tighten their urethra sufficiently to overcome the increased detrusor pressure. Stress incontinence is common after childbirth, when the pelvic muscles are stretched and weakened from pregnancy and delivery. Urge incontinence occurs in people who cannot suppress the contraction signal from the detrusor muscle. Abnormal detrusor contractions may be a result of neurologic abnormalities including dementia, or may occur with no known abnormality. Post-void residual is associated with reflex incontinence, not with urge incontinence or stress incontinence. Management of urge incontinence includes decreasing fluid intake, especially in the evening hours.

7. A client is admitted to the nursing unit after having a tympanoplasty. What activities does the nurse delegate to the unlicensed assistive personnel (UAP)? (Select all that apply.) a. Administer prescribed antibiotics. b. Keep the head of the clients bed flat. c. Remind the client to lie on the operative side. d. Remove the iodoform gauze in 8 hours. e. Take and record postoperative vital signs.

ANS: B, E The UAP can keep the head of the clients bed flat and take/record vital signs. The nurse administers medications. The client should lie flat with the head turned so the operative side is up. The nurse or surgeon removes the gauze packing.

3. A nurse is caring for a client after a stroke. What actions may the nurse delegate to the unlicensed assistive personnel (UAP)? (Select all that apply.) a. Assess neurologic status with the Glasgow Coma Scale. b. Check and document oxygen saturation every 1 to 2 hours. c. Cluster client care to allow periods of uninterrupted rest. d. Elevate the head of the bed to 45 degrees to prevent aspiration. e. Position the client supine with the head in a neutral midline position.

ANS: B, E The UAP can take and document vital signs, including oxygen saturation, and keep the clients head in a neutral, midline position with correct direction from the nurse. The nurse assesses the Glasgow Coma Scale score. The nursing staff should not cluster care because this can cause an increase in the intracranial pressure. The head of the bed should be minimally elevated, up to 30 degrees.

5. A hospitalized client has Mnires disease. What menu selections demonstrate good knowledge of the recommended diet for this disorder? (Select all that apply.) a. Chinese stir fry with vegetables b. Broiled chicken breast c. Chocolate espresso cookies d. Deli turkey sandwich and chips e. Green herbal tea with meals

ANS: B, E The diet recommendations for Mnires disease include low-sodium, caffeine-free foods and fluids distributed evenly throughout the day. Plenty of water is also needed. The broiled chicken breast and herbal tea are the best selections. The stir fry is high in sodium and possibly monosodium glutamate (MSG, also not recommended). The cookies have caffeine, and the sandwich and chips are high in sodium.

5. A nurse assesses a client who is experiencing an absence seizure. For which clinical manifestations should the nurse assess? (Select all that apply.) a. Intermittent rigidity b. Lip smacking c. Sudden loss of muscle tone d. Brief jerking of the extremities e. Picking at clothing f. Patting of the hand on the leg

ANS: B, E, F Automatisms are characteristic of absence seizures. These behaviors consist of lip smacking, picking at clothing, and patting. Rigidity of muscles is associated with the tonic phase of a seizure, and jerking of the extremities is associated with the clonic phase of a seizure. Loss of muscle tone occurs with atonic seizures.

3. An older client's serum calcium level is 8.7 mg/dL (2.18 mmol/L). What possible etiology(ies) does the nurse consider for this result? (Select all that apply.) a. Good dietary intake of calcium and vitamin D b. Normal age-related decrease in serum calcium c. Possible occurrence of osteoporosis or osteopenia d. Potential for metastatic cancer or Paget disease e. Recent bone fracture in a healing stage

ANS: B,C This slightly low calcium level could be an age-related decrease in serum calcium or could indicate a metabolic bone disease, such as osteoporosis or osteopenia. A good dietary intake would be expected to produce normal values. Metastatic cancer, Paget disease, or healing bone fractures will elevate calcium.

2. A nurse is learning about different surgical procedures and their classifications. Which examples below does this include? (Select all that apply.) a. Rhinoplasty: curative b. Liver biopsy: diagnostic c. Arthroscopy: preventative. Ileostomy: palliative. Total shoulder replacement: reconstructive d. Body contouring: cosmetic

ANS: B,C,D A cosmetic procedure is designed to improve the client's appearance or self-confidence; a body contouring procedure is an example. A diagnostic procedure is performed to determine the origin and cause of a disorder by taking a tissue sample with the intention of diagnosing (and staging, if applicable) a condition, such as a liver biopsy. A preventative procedure is performed with the intention that a specific condition will not occur. An example of this is a prophylactic bilateral mastectomy in a woman who carries the BRCA 1 or BRCA 2 gene to prevent the development of breast cancer. A palliative procedure is designed to improve quality of life; an example is an ileostomy. A reconstructive operation improves functional ability is an abnormal or damaged structure. A total shoulder replacement would be an example. A curative operation is performed to resolve a health problem by repairing or removing the cause; a gallbladder removal is an example.

9. A postanesthesia care unit (PACU) nurse is assessing a postoperative client with a nasogastric (NG) tube. What laboratory values would warrant intervention by the nurse? (Select all that apply.) a. Blood glucose: 120 mg/dL (6.7 mmol/L) b. Hemoglobin: 7.8 mg/dL (78 mmol/L) c. pH: 7.68 d. Potassium: 2.9 mEq/L (2.9 mmol/L) e. Sodium: 142 mEq/L (142 mmol/L)

ANS: B,C,D Fluid and electrolyte balance are assessed carefully in the postoperative client because many imbalances can occur. The low hemoglobin may be from blood loss in surgery. The higher pH level indicates alkalosis, possibly from losses through the NG tube. The potassium is very low. The blood glucose is within normal limits for a postsurgical client who has been fasting. The sodium level is normal.

4. A nurse recently hired to the preoperative area learns that certain clients are at higher risk for venous thromboembolism (VTE). Which clients are considered to be at high risk? (Select all that apply.) a. Client with a humerus fracture b. Morbidly obese client c. Client who smokes d. Client with severe heart failure e. Wheelchair-bound client f. 50 years of age or older

ANS: B,C,D,E All surgical clients should be assessed for VTE risk. Those considered to be at higher risk include those who are obese; are over 40; have cancer; have decreased mobility, immobility, or a spinal cord injury; have a history of any thrombotic event, varicose veins, or edema; take oral contraceptives or smoke; have decreased cardiac output; have a hip fracture; or are having total hip or knee surgery.

4. The nurse is caring for a client with a new diagnosis of chronic kidney disease. Which priority complications would the nurse anticipate? (Select all that apply.) a. Dehydration b. Anemia c. Hypertension d. Dysrhythmias e. Heart failure

ANS: B,C,D,E The client who has CKD has fluid overload and electrolyte imbalances, especially hyperkalemia, that can cause hypertension, heart failure, and dysrhythmias. Anemia results because erythropoietin production by the kidneys is decreased.

3. A hospitalized client has a new diagnosis of Ménière disease. What would the nurse include in health teaching to reduce symptoms for this disorder? (Select all that apply.) a. "Apply heat to the ear for 20 minutes three times a day." b. "Move the head slowly to prevent worsening of the vertigo." c. "Avoid food additives such as monosodium glutamate (MSG)." d. "Quit smoking to increase blood flow to the inner ear." e. "Avoid caffeinated beverages." f. "Avoid standing on chairs, step stools, or ladders."

ANS: B,C,D,E,F Ménière disease is an excess of endolymphatic fluid that distorts the entire inner-canal system causing vertigo, tinnitus, and unilateral hearing loss. Applying heat or irrigating the ear canal will not alleviate symptoms. Moving the head slowly will prevent worsening of the vertigo. The diet recommendations for Ménière disease include avoiding caffeine and certain food additives. Smoking causes constriction of blood vessels and decreased blood flow to the inner ear. Clients should also avoid standing on high surfaces to prevent vertigo and falls.

2. The nurse is teaching a client with mild rheumatoid arthritis (RA) about how to protect synovial joints. Which health teaching will the nurse include? (Select all that apply.) a. "Use small joints rather than larger ones during tasks." b. "Use both hands instead of one with holding objects." c. "When getting out of bed or a chair, use the palms of your hands." d. "Bend your knees instead of your waist and keep your back straight." e. "Do not use multiple pillows under your head to prevent neck flexion." f. "Use a device or rubber grip to open jars or bottle tops." g. "Use long-handled devices such as a hairbrush with an extended handle."

ANS: B,C,D,E,F,G All of these options are part of health teaching for joint protection except that large joints should be used instead of smaller ones.

4. The nurse assesses a client with long-term rheumatoid arthritis (RA) for late signs and symptoms. Which assessment findings will the nurse document as late signs and symptoms of RA? (Select all that apply.) a. Anorexia b. Felty syndrome c. Joint deformity d. Low-grade fever e. Weight loss

ANS: B,C,E Late signs and symptoms of RA include Felty syndrome, joint deformity, weight loss, organ involvement, osteoporosis, extreme fatigue, and anemia, among others. Anorexia and low-grade fever are both seen early in the course of the disease.

1. A new perioperative nurse is receiving orientation to the surgical area and learns about the Surgical Care Improvement Project (SCIP) goals. What major areas do these measures focus on? (Select all that apply.) a. Hemorrhage prevention b. Infection prevention c. Malignant hyperthermia testing d. Stroke recognition e. Thromboembolism prevention f. Correct hair removal

ANS: B,E,F The Surgical Care Improvement Project (SCIP), a set of core compliance measures, was initiated in 2006 to reduce surgical complications. Examples of focus included administration of prophylactic antibiotics to prevent infection, correct hair removal processes, the timing of discontinuation of urinary catheterization after surgery, and venous thromboembolism prophylaxis. These practices are now standard in surgical care.

7. The nurse is caring for a client who has diabetes mellitus type 1 and is experiencing hypoglycemia. Which assessment findings will the nurse expect? (Select all that apply.) a. Warm, dry skin b. Nervousness c. Rapid deep respirations d. Dehydration e. Ketoacidosis f. Blurred vision

ANS: B,F The client who has hypoglycemia is often anxious, nervous, and possibly confused. Due to lack of glucose, vision may be blurred or the client may report diplopia (double vision). Clients who have hyperglycemia from diabetes mellitus type 1 have warm skin, Kussmaul respirations that are rapid and deep, dehydration due to elevated blood glucose, and ketoacidosis.

10. The charge nurse of the medical-surgical unit is making staff assignments. Which staff member should be assigned to a client with chronic kidney disease who is exhibiting a low-grade fever and a pericardial friction rub? a. Registered nurse who just floated from the surgical unit b. Registered nurse who just floated from the dialysis unit c. Registered nurse who was assigned the same client yesterday d. Licensed practical nurse with 5 years experience on this floor

ANS: C The client is exhibiting symptoms of pericarditis, which can occur with chronic kidney disease. Continuity of care is important to assess subtle differences in clients. Therefore, the registered nurse (RN) who was assigned to this client previously should again give care to this client. The float nurses would not be as knowledgeable about the unit and its clients. The licensed practical nurse may not have the education level of the RN to assess for pericarditis.

27. The nurse assesses a clients Glasgow Coma Scale (GCS) score and determines it to be 12 (a 4 in each category). What care should the nurse anticipate for this client? a. Can ambulate independently b. May have trouble swallowing c. Needs frequent re-orientation d. Will need near-total care

ANS: C This client will most likely be confused and need frequent re-orientation. The client may not be able to ambulate at all but should do so independently, not because of mental status. Swallowing is not assessed with the GCS. The client will not need near-total care.

9. A nurse is teaching a community group about noise-induced hearing loss. Which client who does not use ear protection should the nurse refer to an audiologist as the priority? a. Client with an hour car commute on the freeway each day b. Client who rides a motorcycle to work 20 minutes each way c. Client who sat in the back row at a rock concert recently d. Client who is a tree-trimmer and uses a chainsaw 6 to 7 hours a day

ANS: D A chainsaw becomes dangerous to hearing after 2 hours of exposure without hearing protection. This client needs to be referred as the priority. Normal car traffic is safe for more than 8 hours. Motorcycle noise is safe for about 8 hours. The safe exposure time for a front-row rock concert seat is 3 minutes, but this client was in the back, and so had less exposure. In addition, a one-time exposure is less damaging than chronic exposure.

8. The nurse is collaborating with the occupational therapist to assist a client with a complete cervical spinal cord injury to transfer from the bed to the wheelchair. What ambulatory aid would be most appropriate for the client to meet this outcome? a. Rolling walker b. Quad cane c. Adjustable crutches d. Sliding board

ANS: D A client who has a complete cervical spinal cord injury is unable to use any extremity except for parts of the hands and possibly the lower arms. Therefore, the client would be unable to use any of these ambulatory aids except for a sliding board, also known as a slider, which provides a "bridge" between the bed and a chair. The client uses his or her arms in a locked position to support the body while moving slowly across the board.

6. The nurse is teaching about signs and symptoms of cataracts. Which change would the nurse emphasize as possibly indicating beginning cataract formation? a. Diplopia b. Cloudy pupil c. Loss of peripheral vision d. Blurred vision

ANS: D A cloudy pupil is a sign of late cataracts and loss of peripheral vision is more common in clients who have glaucoma. Diplopia occurs with a number of neurologic diseases. Blurred vision is the earliest sign that the lens of the eye is undergoing changes.

3. A client is admitted to the emergency department with a probable traumatic brain injury. Which assessment finding would be the priority for the nurse to report to the primary health care provider? a. Mild temporal headache b. Pupils equal and react to light c.Alert and oriented 3 d. Decreasing level of consciousness

ANS: D A decreasing level of consciousness is the first sign of increasing intracranial pressure, a potentially severe and possibly fatal complication of a traumatic brain injury (TBI). A mild headache would be expected for a client having a TBI. Equal reactive pupils and being alert and oriented are normal assessment findings.

10. The nurse is assessing four clients with musculoskeletal disorders. The nurse should assess the client with which laboratory result first? a. Serum alkaline phosphatase (ALP): 108 units/L b. Serum aspartate aminotransferase (AST): 26 units/L c. Serum calcium: 10.2 mg/dL d. Serum phosphorus: 2 mg/dL

ANS: D A normal serum phosphorus level is 3 to 4.5 mg/dL; a level of 2 mg/dL is low, and this client should be assessed first. The values for serum ALP, AST, and calcium are all within normal ranges.

14. The nurse is caring for a postoperative client who have a regional nerve blockade for a surgical tibial fracture repair this morning. What assessment finding would the nurse expect? a. Client reports nausea and vomiting. b. Client reports tingling in the surgical leg. c. Client responds well to imagery. d. Client reports little to no pain.

ANS: D A regional nerve blockade can last for about 24 hours so the client has little to no pain until it wears off. The blockade is localized and therefore does not cause nausea or vomiting.

14. A client has a long history of hypertension. Which category of medications would the nurse expect to be ordered to avoid chronic kidney disease (CKD)? a. Antibiotic b. Histamine blocker c. Bronchodilator d. Angiotensin-converting enzyme (ACE) inhibitor

ANS: D ACE inhibitors stop the conversion of angiotensin I to the vasoconstrictor angiotensin II. This category of medication also blocks bradykinin and prostaglandin, increases renin, and decreases aldosterone, which promotes vasodilation and perfusion to the kidney. Antibiotics fight infection, histamine blockers decrease inflammation, and bronchodilators increase the size of the bronchi; none of these medications helps slow the progression of CKD in clients with hypertension.

21. A nurse cares for a client with amyotrophic lateral sclerosis (ALS). The client states, I do not want to be placed on a mechanical ventilator. How should the nurse respond? a. You should discuss this with your family and health care provider. b. Why are you afraid of being placed on a breathing machine? c. Using the incentive spirometer each hour will delay the need for a ventilator. d. What would you like to be done if you begin to have difficulty breathing?

ANS: D ALS is an adult-onset upper and lower motor neuron disease characterized by progressive weakness, muscle wasting, and spasticity, eventually leading to paralysis. Once muscles of breathing are involved, the client must indicate in the advance directive what is to be done when breathing is no longer possible without intervention.

5. The nurse assesses a client for factors that place the client at risk for cataracts. Which factor places the client at the highest risk for cataract development? a. Heart disease b. Glaucoma c. Diabetes mellitus d. Advanced age

ANS: D Advanced age is the major risk factor for developing cataracts because the lens loses water and lens fibers become more compact.

15. A nurse is seeing clients in the ophthalmology clinic. Which client would the nurse see first? a. Client with intraocular pressure reading of 24 mm Hg b. Client with a tearing, reddened eye with exudate c. Client whose red reflex is absent on ophthalmologic examination d. Client who has had cataract surgery and has worsening vision

ANS: D After cataract surgery, worsening vision indicates a postoperative infection or other complication. The nurse would see this client first. The intraocular pressure is slightly elevated. An absent red reflex may indicate cataracts. The client who has the tearing eye may have an infection.

12. A client diagnosed with Parkinson disease will be starting ropinirole for symptom control. Which statement by the client indicates a need for further teaching? a. "This drug should help decrease my tremors and help me move better." b. "I need to change positions slowly to prevent dizziness or falls." c. "I should take the drug at the same time each day for the best effect." d. "I know the drug will probably make help me prevent constipation."

ANS: D Although ropinirole is a dopamine agonist and mimics dopamine to promote movement, it does not work to prevent constipation. This class of drugs can cause orthostatic hypotension and should be taken at the same time every day.

4. The nurse interviews an older client with moderate osteoarthritis and her husband. What psychosocial assessment question would the nurse include? a. "Do you feel like hurting yourself or others?" b. "Are you planning to retire due to your disease?" c. "Do you ask your husband for assistance?" d. "Do you experience discomfort during sex?"

ANS: D Although some clients can become depressed and anxious as a result of having OA, suicidal ideation is not common. The nurse should not assume that an older adult will want to retire or that the client will need help from her husband. Many clients avoid sexual intercourse because of joint pain and stiffness.

4. A nurse assesses a clients recent memory. Which client statement confirms that the clients remote memory is intact? a. A young girl wrapped in a shroud fell asleep on a bed of clouds. b. I was born on April 3, 1967, in Johnstown Community Hospital. c. Apple, chair, and pencil are the words you just stated. d. I ate oatmeal with wheat toast and orange juice for breakfast.

ANS: D Asking clients about recent events that can be verified, such as what the client ate for breakfast, assesses the clients recent memory. The clients ability to make up a rhyme tests not memory, but rather a higher level of cognition. Asking clients about certain facts from the past that can be verified assesses remote or long-term memory. Asking the client to repeat words assesses the clients immediate memory.

10. After teaching the wife of a client who has Parkinson disease, the nurse assesses the wifes understanding. Which statement by the clients wife indicates she correctly understands changes associated with this disease? a. His masklike face makes it difficult to communicate, so I will use a white board. b. He should not socialize outside of the house due to uncontrollable drooling. c. This disease is associated with anxiety causing increased perspiration. d. He may have trouble chewing, so I will offer bite-sized portions.

ANS: D Because chewing and swallowing can be problematic, small frequent meals and a supplement are better for meeting the clients nutritional needs. A masklike face and drooling are common in clients with Parkinson disease. The client should be encouraged to continue to socialize and communicate as normally as possible. The wife should understand that the clients masklike face can be misinterpreted and additional time may be needed for the client to communicate with her or others. Excessive perspiration is also common in clients with Parkinson disease and is associated with the autonomic nervous systems response.

8. The nurse is admitting a client who has acute glomerulonephritis caused by beta streptococcus. What drug therapy would the nurse expect to be prescribed for this client? a. Antihypertensives b. Antilipidemics c. Antidepressants d. Antibiotics

ANS: D Beta streptococcus is a bacterium that can cause acute glomerulonephritis, so antibiotic therapy is indicated.

23. A nurse is caring for a client who is scheduled for a dose of cefazolin and vitamins at this time. Hemodialysis for this client is also scheduled in 60 minutes. Which action by the nurse is best? a. Administer cefazolin since the level of the antibiotic must be maintained. b. Hold the vitamins but administer the cefazolin. c. Hold the cefazolin but administer the vitamins. d. Hold all medications since both cefazolin and vitamins are dialyzable.

ANS: D Both the cefazolin and the vitamins should be held until after the hemodialysis is completed because they would otherwise be removed by the dialysis process.

5. The nurse is caring for a client with acromegaly who is starting bromocriptine. What health teaching by the nurse about drug therapy will the nurse include? a. "Take this drug on an empty stomach first thing in the morning." b. "You will be starting on a high dose of the drug to ensure it will work." c. "You might experience an increase in blood pressure in about a week." d. "Seek medical attention immediately if you have chest pain and dizziness."

ANS: D Bromocriptine should be started on a low dose and taken with food. The drug can cause decreased blood pressure, including orthostatic hypotension. Serious effects such as cardiac dysrhythmias, coronary artery spasms, and cerebrospinal leak can occur Therefore, the nurse teaches the client should seek medical attention if he or she experiences chest pain, dizziness, and watery nasal discharge.

2. A nurse cares for a client with autosomal dominant polycystic kidney disease (ADPKD). The client asks, Will my children develop this disease? How should the nurse respond? a. No genetic link is known, so your children are not at increased risk. b. Your sons will develop this disease because it has a sex-linked gene. c. Only if both you and your spouse are carriers of this disease. d. Each of your children has a 50% risk of having ADPKD.

ANS: D Children whose parent has the autosomal dominant form of PKD have a 50% chance of inheriting the gene that causes the disease. ADPKD is transmitted as an autosomal dominant trait and therefore is not gender specific. Both parents do not need to have this disorder.

4. The nurse sees several clients with osteoporosis. For which client would bisphosphonates not be a good option? a. Client with diabetes who has a serum creatinine of 0.8 mg/dL b. Client who recently fell and has vertebral compression fractures c. Hypertensive client who takes calcium channel blockers d. Client with a spinal cord injury who cannot tolerate sitting up

ANS: D Clients on bisphosphonates must be able to sit upright for 30 to 60 minutes after taking them. The client who cannot tolerate sitting up is not a good candidate for this class of drug. Poor renal function also makes clients bad candidates for this drug, but the client with a creatinine of 0.8 mg/dL is within normal range. Diabetes and hypertension are not related unless the client also has renal disease. The client who recently fell and sustained fractures is a good candidate for this drug if the fractures are related to osteoporosis.

27.A nurse teaches a client with diabetes mellitus who is experiencing numbness and reduced sensation. Which statement should the nurse include in this clients teaching to prevent injury? a. Examine your feet using a mirror every day. b. Rotate your insulin injection sites every week. c. Check your blood glucose level before each meal. d. Use a bath thermometer to test the water temperature.

ANS: D Clients with diminished sensory perception can easily experience a burn injury when bathwater is too hot. Instead of checking the temperature of the water by feeling it, they should use a thermometer. Examining the feet daily does not prevent injury, although daily foot examinations are important to find problems so they can be addressed. Rotating insulin and checking blood glucose levels will not prevent injury.

11. A nurse teaches a client who is recovering from a nephrectomy secondary to kidney trauma. Which statement should the nurse include in this clients teaching? a. Since you only have one kidney, a salt and fluid restriction is required. b. Your therapy will include hemodialysis while you recover. c. Medication will be prescribed to control your high blood pressure. d. You need to avoid participating in contact sports like football.

ANS: D Clients with one kidney need to avoid contact sports because the kidneys are easily injured. The client will not be required to restrict salt and fluids, end up on dialysis, or have new hypertension because of the nephrectomy.

7. The nurse is assessing a client diagnosed with trigeminal neuralgia affecting cranial nerve V. What assessment findings will the nurse expect for this client? a. Expressive aphasia b. Ptosis (eyelid drooping) c. Slurred speech d. Severe facial pain

ANS: D Cranial nerve (CN) V is the Trigeminal Nerve which has both a motor and sensory function in the face. When affected by a health problem, the client experiences severely facial pain. Expressive aphasia results from damage to the Broca speech area in the frontal lobe of the brain. Ptosis can result from damage to CN III and slurred speech often occurs from either damage to several cranial nerves or from damage to the motor strip in the frontal lobe of the brain.

6. A trauma nurse cares for several clients with fractures. Which client should the nurse identify as at highest risk for developing deep vein thrombosis? a. An 18-year-old male athlete with a fractured clavicle b. A 36-year old female with type 2 diabetes and fractured ribs c. A 55-year-old woman prescribed aspirin for rheumatoid arthritis d. A 74-year-old man who smokes and has a fractured pelvis

ANS: D Deep vein thrombosis (DVT) as a complication with bone fractures occurs more often when fractures are sustained in the lower extremities and the client has additional risk factors for thrombus formation. Other risk factors include obesity, smoking, oral contraceptives, previous thrombus events, advanced age, venous stasis, and heart disease. The other clients do not have risk factors for DVT.

37.When teaching a client recently diagnosed with type 1 diabetes mellitus, the client states, I will never be able to stick myself with a needle. How should the nurse respond? a. I can give your injections to you while you are here in the hospital. b. Everyone gets used to giving themselves injections. It really does not hurt. c. Your disease will not be managed properly if you refuse to administer the shots. d. Tell me what it is about the injections that are concerning you.

ANS: D Devote as much teaching time as possible to insulin injection and blood glucose monitoring. Clients with newly diagnosed diabetes are often fearful of giving themselves injections. If the client is worried about giving the injections, it is best to try to find out what specifically is causing the concern, so it can be addressed. Giving the injections for the client does not promote self-care ability. Telling the client that others give themselves injections may cause the client to feel bad. Stating that you dont know another way to manage the disease is dismissive of the clients concerns.

7.A nurse assesses clients who are at risk for diabetes mellitus. Which client is at greatest risk? a. A 29-year-old Caucasian b. A 32-year-old African- American c. A 44-year-old Asian d. A 48-year-old American Indian

ANS: D Diabetes is a particular problem among African Americans, Hispanics, and American Indians. The incidence of diabetes increases in all races and ethnic groups with age. Being both an American Indian and middle-aged places this client at highest risk.

3.After teaching a young adult client who is newly diagnosed with type 1 diabetes mellitus, the nurse assesses the clients understanding. Which statement made by the client indicates a correct understanding of the need for eye examinations? a. At my age, I should continue seeing the ophthalmologist as I usually do. b. I will see the eye doctor when I have a vision problem and yearly after age 40. c. My vision will change quickly. I should see the ophthalmologist twice a year. d. Diabetes can cause blindness, so I should see the ophthalmologist yearly.

ANS: D Diabetic retinopathy is a leading cause of blindness in North America. All clients with diabetes, regardless of age, should be examined by an ophthalmologist (rather than an optometrist or optician) at diagnosis and at least yearly thereafter.

19. After teaching a client who is diagnosed with new-onset epilepsy and prescribed phenytoin, the nurse assesses the client's understanding. Which statement by the client indicates a correct understanding of the teaching? a. "To prevent complications, I will drink at least 2 L of water daily." b. "This medication will stop me from getting an aura before a seizure." c. "I will not drive a motor vehicle while taking this medication." d. "Even when my seizures stop, I will continue to take this drug."

ANS: D Discontinuing antiepileptic drugs can lead to the recurrence of seizures or status epilepticus. The client does not need to drink more water and can drive while taking this medication. The drug will not stop an aura before a seizure.

7. After teaching a client who is diagnosed with new-onset status epilepticus and prescribed phenytoin (Dilantin), the nurse assesses the clients understanding. Which statement by the client indicates a correct understanding of the teaching? a. To prevent complications, I will drink at least 2 liters of water daily. b. This medication will stop me from getting an aura before a seizure. c. I will not drive a motor vehicle while taking this medication. d. Even when my seizures stop, I will continue to take this drug.

ANS: D Discontinuing antiepileptic drugs can lead to the recurrence of seizures or status epilepticus. The client does not need to drink more water and can drive while taking this medication. The medication will not stop an aura before a seizure.

8. The primary health care provider prescribes donepezil for a client diagnosed with early-stage Alzheimer disease. What teaching about this drug will the nurse provide for the client's family caregiver? a. "Monitor the client's temperature because the drug can cause a low grade fever." b. "Observe the client for nausea and vomiting to determine drug tolerance." c. "Donepezil will prevent the client's dementia from progressing as usual." d. "Report any client dizziness or falls because the drug can cause bradycardia."

ANS: D Donepezil is a cholinesterase inhibitor that may temporarily slow cognitive decline for some clients but does not alter the course of the disease. The family caregiver would want to monitor the client's heart rate and report any incidence of dizziness or falls because the drug can cause bradycardia. It does not typically cause fever or nausea/vomiting.

11. A nurse is irrigating a clients ear when the client becomes nauseated. What action by the nurse is most appropriate for client comfort? a. Have the client tilt the head back. b. Re-position the client on the other side. c. Slow the rate of the irrigation. d. Stop the irrigation immediately.

ANS: D During ear irrigation, if the client becomes nauseated, stop the procedure. The other options are not helpful.

11. A nurse plans care for a client with Parkinson disease. Which intervention should the nurse include in this clients plan of care? a. Ambulate the client in the hallway twice a day. b. Ensure a fluid intake of at least 3 liters per day. c. Teach the client pursed-lip breathing techniques. d. Keep the head of the bed at 30 degrees or greater.

ANS: D Elevation of the head of the bed will help prevent aspiration. The other options will not prevent aspiration, which is the greatest respiratory complication of Parkinson disease, nor do these interventions address any of the complications of Parkinson disease. Ambulation in the hallway is usually implemented to prevent venous thrombosis. Increased fluid intake flushes out toxins from the clients blood. Pursed-lip breathing increases exhalation of carbon dioxide.

6. The nurse is taking a history from an older client who reports having frequent falls. Which dietary habit could be contributing to the client's problem? a. Consumes high-protein foods. b. Eats few concentrated sweets. c. Limits fatty or greasy foods. d. Avoids dairy products.

ANS: D Falls can occur when older adults have inadequate calcium and Vitamin D because they are at risk for osteopenia and osteoporosis. Dairy products have a high concentration of both calcium and Vitamin D and this client avoids those foods. High-protein foods are recommended to help prevent osteopenia and sweets and fatty/greasy foods have no impact on bone health.

19. A nurse teaches a client with functional urinary incontinence. Which statement should the nurse include in this clients teaching? a. You must clean around your catheter daily with soap and water. b. Wash the vaginal weights with a 10% bleach solution after each use. c. Operations to repair your bladder are available, and you can consider these. d. Buy slacks with elastic waistbands that are easy to pull down.

ANS: D Functional urinary incontinence occurs as the result of problems not related to the clients bladder, such as trouble ambulating or difficulty accessing the toilet. One goal is that the client will be able to manage his or her clothing independently. Elastic waistband slacks that are easy to pull down can help the client get on the toilet in time to void. The other instructions do not relate to functional urinary incontinence.

39.A nurse reviews the medication list of a client recovering from a computed tomography (CT) scan with IV contrast to rule out small bowel obstruction. Which medication should alert the nurse to contact the provider and withhold the prescribed dose? a. Pioglitazone (Actos) b. Glimepiride (Amaryl) c. Glipizide (Glucotrol) d. Metformin (Glucophage)

ANS: D Glucophage should not be administered when the kidneys are attempting to excrete IV contrast from the body. This combination would place the client at high risk for kidney failure. The nurse should hold the metformin dose and contact the provider. The other medications are safe to administer after receiving IV contrast.

1. A nurse reviews the urinalysis of a client and notes the presence of glucose. Which action should the nurse take? a. Document findings and continue to monitor the client. b. Contact the provider and recommend a 24-hour urine test. c. Review the clients recent dietary selections. d. Perform a capillary artery glucose assessment.

ANS: D Glucose normally is not found in the urine. The normal renal threshold for glucose is about 220 mg/dL, which means that a person whose blood glucose is less than 220 mg/dL will not have glucose in the urine. A positive finding for glucose on urinalysis indicates high blood sugar. The most appropriate action would be to perform a capillary artery glucose assessment. The client needs further evaluation for this abnormal result; therefore, documenting and continuing to monitor is not appropriate. Requesting a 24-hour urine test or reviewing the clients dietary selections will not assist the nurse to make a clinical decision related to this abnormality.

11. A client has arrived in the inpatient postoperative unit. What action by the inpatient nurse takes priority? a. Assessing fluid and blood output b. Checking the surgical dressings c. Ensuring the client is warm d. Participating in hand-off report

ANS: D Hand-offs are a critical time in client care, and poor communication during this time can lead to serious errors. The inpatient nurse and postanesthesia care nurse participate in hand-off report as the priority. Assessing fluid losses and dressings can be done together as part of the report. Ensuring the client is warm is a lower priority.

16. A nurse teaches a client who is prescribed an insulin pump. Which statement should the nurse include in this clients discharge education? a. Test your urine daily for ketones. b. Use only buffered insulin in your pump. c. Store the insulin in the freezer until you need it. d. Change the needle every 3 days.

ANS: D Having the same needle remain in place through the skin for longer than 3 days drastically increases the risk for infection in or through the delivery system. Having an insulin pump does not require the client to test for ketones in the urine. Insulin should not be frozen. Insulin is not buffered.

22. A client is assessed by the nurse after a hemodialysis session. The nurse notes bleeding from the clients nose and around the intravenous catheter. What action by the nurse is the priority? a. Hold pressure over the clients nose for 10 minutes. b. Take the clients pulse, blood pressure, and temperature. c. Assess for a bruit or thrill over the arteriovenous fistula. d. Prepare protamine sulfate for administration.

ANS: D Heparin is used with hemodialysis treatments. The bleeding alerts the nurse that too much anticoagulant is in the clients system and protamine sulfate should be administered. Pressure, taking vital signs, and assessing for a bruit or thrill are not as important as medication administration.

2.A nurse reviews laboratory results for a client with diabetes mellitus who presents with polyuria, lethargy, and a blood glucose of 560 mg/dL. Which laboratory result should the nurse correlate with the clients polyuria? a. Serum sodium: 163 mEq/L b. Serum creatinine: 1.6 mg/dL c. Presence of urine ketone bodies d. Serum osmolarity: 375 mOsm/kg

ANS: D Hyperglycemia causes hyperosmolarity of extracellular fluid. This leads to polyuria from an osmotic diuresis. The clients serum osmolarity is high. The clients sodium would be expected to be high owing to dehydration. Serum creatinine and urine ketone bodies are not related to the polyuria.

11. A nurse cares for a client who is recovering from a parathyroidectomy. When taking the clients blood pressure, the nurse notes that the clients hand has gone into flexion contractions. Which laboratory result does the nurse correlate with this condition? a. Serum potassium: 2.9 mEq/L b. Serum magnesium: 1.7 mEq/L c. Serum sodium: 122 mEq/L d. Serum calcium: 6.9 mg/dL

ANS: D Hypocalcemia destabilizes excitable membranes and can lead to muscle twitches, spasms, and tetany. This effect of hypocalcemia is enhanced in the presence of tissue hypoxia. The flexion contractions (Trousseaus sign) that occur during blood pressure measurement are indicative of hypocalcemia, not the other electrolyte imbalances, which include hypokalemia, hyponatremia, and hypomagnesemia.

7. A nurse assesses a client who is prescribed levothyroxine (Synthroid) for hypothyroidism. Which assessment finding should alert the nurse that the medication therapy is effective? a. Thirst is recognized and fluid intake is appropriate. b. Weight has been the same for 3 weeks. c. Total white blood cell count is 6000 cells/mm3. d. Heart rate is 70 beats/min and regular.

ANS: D Hypothyroidism decreases body functioning and can result in effects such as bradycardia, confusion, and constipation. If a clients heart rate is bradycardic while on thyroid hormone replacement, this is an indicator that the replacement may not be adequate. Conversely, a heart rate above 100 beats/min may indicate that the client is receiving too much of the thyroid hormone. Thirst, fluid intake, weight, and white blood cell count do not represent a therapeutic response to this medication.

10. A nurse teaches a client with a lower motor neuron lesion who wants to achieve bladder control. Which statement should the nurse include in this clients teaching? a. Stroke the inner aspect of your thigh to initiate voiding. b. Use a clean technique for intermittent catheterization. c. Implement digital anal stimulation when your bladder is full. d. Tighten your abdominal muscles to stimulate urine flow.

ANS: D In clients with lower motor neuron problems such as spinal cord injury, performing a Valsalva maneuver or tightening the abdominal muscles are interventions that can initiate voiding. Stroking the inner aspect of the thigh may initiate voiding in a client who has an upper motor neuron problem. Intermittent catheterization and digital anal stimulation do not initiate voiding or bladder control.

8. A nurse plans care for a client with overflow incontinence. Which intervention should the nurse include in this clients plan of care to assist with elimination? a. Stroke the medial aspect of the thigh. b. Use intermittent catheterization. c. Provide digital anal stimulation. d. Use the Valsalva maneuver.

ANS: D In clients with overflow incontinence, the voiding reflex arc is not intact. Mechanical pressure, such as that achieved through the Valsalva maneuver (holding the breath and bearing down as if to defecate), can initiate voiding. Stroking the medial aspect of the thigh or providing digital anal stimulation requires the reflex arc to be intact to initiate elimination. Due to the high risk for infection, intermittent catheterization should only be implemented when other interventions are not successful.

32.A nurse reviews the laboratory results of a client who is receiving intravenous insulin. Which should alert the nurse to intervene immediately? a. Serum chloride level of 98 mmol/L b. Serum calcium level of 8.8 mg/dL c. Serum sodium level of 132 mmol/L d. Serum potassium level of 2.5 mmol/L

ANS: D Insulin activates the sodium-potassium ATPase pump, increasing the movement of potassium from the extracellular fluid into the intracellular fluid, resulting in hypokalemia. In hyperglycemia, hypokalemia can also result from excessive urine loss of potassium. The chloride level is normal. The calcium and sodium levels are slightly low, but this would not be related to hyperglycemia and insulin administration.

12. A nurse cares for a client who is experiencing deteriorating neurologic functions. The client states, I am worried I will not be able to care for my young children. How should the nurse respond? a. Caring for your children is a priority. You may not want to ask for help, but you have to. b. Our community has resources that may help you with some household tasks so you have energy to care for your children. c. You seem distressed. Would you like to talk to a psychologist about adjusting to your changing status? d. Give me more information about what worries you, so we can see if we can do something to make adjustments.

ANS: D Investigate specific concerns about situational or role changes before providing additional information. The nurse should not tell the client what is or is not a priority for him or her. Although community resources may be available, they may not be appropriate for the client. Consulting a psychologist would not be appropriate without obtaining further information from the client related to current concerns.

10. A nurse obtains a focused health history for a client who is scheduled for magnetic resonance imaging (MRI). Which condition should alert the nurse to contact the provider and cancel the procedure? a. Creatine phosphokinase (CPK) of 100 IU/L b. Atrioventricular graft c. Blood urea nitrogen (BUN) of 50 mg/dL d. Internal insulin pump

ANS: D Metal devices such as internal pumps, pacemakers, and prostheses interfere with the accuracy of the image and can become displaced by the magnetic force generated by an MRI procedure. An atrioventricular graft does not contain any metal. CPK and BUN levels have no impact on an MRI procedure.

19. A nurse prepares a client for prescribed magnetic resonance imaging (MRI). Which action should the nurse implement prior to the test? a. Implement nothing by mouth (NPO) status for 8 hours. b. Withhold all daily medications until after the examination. c. Administer morphine sulfate to prevent claustrophobia during the test. d. Place the client in a gown that has cloth ties instead of metal snaps.

ANS: D Metal objects are a hazard because of the magnetic field used in the MRI procedure. Morphine sulfate is not administered to prevent claustrophobia; lorazepam (Ativan) or diazepam (Valium) may be used instead. The client does not need to be NPO, and daily medications do not need to be withheld prior to MRI.

14. A nurse cares for a client who presents with an acute exacerbation of multiple sclerosis (MS). Which prescribed medication should the nurse prepare to administer? a. Baclofen (Lioresal) b. Interferon beta-1b (Betaseron) c. Dantrolene sodium (Dantrium) d. Methylprednisolone (Medrol)

ANS: D Methylprednisolone is the drug of choice for acute exacerbations of the disease. The other drugs are not used to treat acute exacerbations of MS. Interferon beta-1b is used to treat and control MS, decrease specific symptoms, and slow the progression of the disease. Baclofen and dantrolene sodium are prescribed to lessen muscle spasticity associated with MS.

22.A nurse teaches a client with type 1 diabetes mellitus. Which statement should the nurse include in this clients teaching to decrease the clients insulin needs? a. Limit your fluid intake to 2 liters a day. b. Animal organ meat is high in insulin. c. Limit your carbohydrate intake to 80 grams a day. d. Walk at a moderate pace for 1 mile daily.

ANS: D Moderate exercise such as walking helps regulate blood glucose levels on a daily basis and results in lowered insulin requirements for clients with type 1 diabetes mellitus. Restricting fluids and eating organ meats will not reduce insulin needs. People with diabetes need at least 130 grams of carbohydrates each day.

10.After teaching a client with type 2 diabetes mellitus who is prescribed nateglinide (Starlix), the nurse assesses the clients understanding. Which statement made by the client indicates a correct understanding of the prescribed therapy? a. Ill take this medicine during each of my meals. b. I must take this medicine in the morning when I wake. c. I will take this medicine before I go to bed. d. I will take this medicine immediately before I eat.

ANS: D Nateglinide is an insulin secretagogue that is designed to increase meal-related insulin secretion. It should be taken immediately before each meal. The medication should not be taken without eating as it will decrease the clients blood glucose levels. The medication should be taken before meals instead of during meals.

7. A nurse cares for a client with a urine specific gravity of 1.040. Which action should the nurse take? a. Obtain a urine culture and sensitivity. b. Place the client on restricted fluids. c. Assess the clients creatinine level. d. Increase the clients fluid intake.

ANS: D Normal specific gravity for urine is 1.005 to 1.030. A high specific gravity can occur with dehydration, decreased kidney blood flow (often because of dehydration), and the presence of antidiuretic hormone. Increasing the clients fluid intake would be a beneficial intervention. Assessing the creatinine or obtaining a urine culture would not provide data necessary for the nurse to make a clinical decision.

3. A nurse reviews the urinalysis results of a client and notes a urine osmolality of 1200 mOsm/L. Which action should the nurse take? a. Contact the provider and recommend a low-sodium diet. b. Prepare to administer an intravenous diuretic. c. Obtain a suction device and implement seizure precautions. d. Encourage the client to drink more fluids.

ANS: D Normal urine osmolality ranges from 300 to 900 mOsm/L. This clients urine is more concentrated, indicating dehydration. The nurse should encourage the client to drink more water. Dehydration can be associated with elevated serum sodium levels. Although a low-sodium diet may be appropriate for this client, this diet change will not have a significant impact on urine osmolality. A diuretic would increase urine output and decrease urine osmolality further. Low serum sodium levels, not elevated serum levels, place the client at risk for seizure activity. These options would further contribute to the clients dehydration or elevate the osmolality.

12. A nurse cares for a client who had a wrist cast applied 3 days ago. The client states, The cast is loose enough to slide off. How should the nurse respond? a. Keep your arm above the level of your heart. b. As your muscles atrophy, the cast is expected to loosen. c. I will wrap a bandage around the cast to prevent it from slipping. d. You need a new cast now that the swelling is decreased.

ANS: D Often the surrounding soft tissues may be swollen considerably when the cast is initially applied. After the swelling has resolved, if the cast is loose enough to permit two or more fingers between the cast and the clients skin, the cast needs to be replaced. Elevating the arm will not solve the problem, and the clients muscles should not atrophy while in a cast for 6 weeks or less. An elastic bandage will not prevent slippage of the cast.

3. A nurse obtains the health history of a client with a fractured femur. Which factor identified in the clients history should the nurse recognize as an aspect that may impede healing of the fracture? a. Sedentary lifestyle b. A 30pack-year smoking history c. Prescribed oral contraceptives d. Pagets disease

ANS: D Pagets disease and bone cancer can cause pathologic fractures such as a fractured femur that do not achieve total healing. The other factors do not impede healing but may cause other health risks.

3. The nurse assesses a client who is scheduled to have a laboratory test to determine if the client's adrenal glands are hypoactive. What type of testing would the client likely have? a. Catecholamine testing b. Suppression testing c. Bone marrow testing d. Provocative testing

ANS: D Provocative testing is done to determine if an endocrine gland is capable of producing its normal level of hormone(s), especially when a client is suspected of having a hypoactive endocrine gland.

23. A phone triage nurse speaks with a client who has an arm cast. The client states, My arm feels really tight and puffy. How should the nurse respond? a. Elevate your arm on two pillows and get ice to apply to the cast. b. Continue to take ibuprofen (Motrin) until the swelling subsides. c. This is normal. A new cast will often feel a little tight for the first few days. d. Please come to the clinic today to have your arm checked by the provider.

ANS: D Puffy fingers and a feeling of tightness from the cast may indicate the development of compartment syndrome. The client should come to the clinic that day to be evaluated by the provider because delay of treatment can cause permanent damage to the extremity. Ice and ibuprofen are acceptable actions, but checking the cast is the priority because it ensures client safety. The nurse should not reassure the client that this is normal.

7. The nurse is caring for a young client who has been diagnosed with osteopenia. Which risk factor in the client's history most likely contributed to the bone loss? a. Osteoarthritis b. Hypothyroidism c. Addison disease d. Rheumatoid arthritis

ANS: D Rheumatoid arthritis often occurs in young female adults and can lead to osteoporosis as a common complication. Cushing disease (rather than Addison disease) and hyperthyroidism (rather than hypothyroidism) are also risk factors. Osteoarthritis is a joint disease.

8. A client is going on a cruise but has had motion sickness in the past. What suggestion does the nurse make to this client? a. Avoid alcohol on the cruise ship. b. Change positions slowly on the ship. c. Change your travel plans. d. Try scopolamine (Transderm Scop).

ANS: D Scopolamine can successfully treat the vertigo and dizziness associated with motion sickness. Avoiding alcohol and changing positions slowly are not effective. Telling the client to change travel plans is not a caring suggestion.

12. The nurse assesses a client with rheumatoid arthritis (RA) and Sjögren syndrome. What assessment would be most important for this client? a. Abdominal assessment b. Oxygen saturation c. Breath sounds d. Visual acuity

ANS: D Sjögren syndrome may be seen in clients with RA and manifests with dryness of the eyes, mouth, and vagina in females. Visual disturbances can occur. The other assessments are not related to Sjögren syndrome.

2. A preoperative nurse is assessing a client prior to surgery. Which information would be most important for the nurse to relay to the surgical team? a. Allergy to bee and wasp stings b. History of lactose intolerance c. No previous experience with surgery d. Use of multiple herbs and supplements

ANS: D Some herbs and supplements can interact with medications, so this information needs to be reported as the priority. An allergy to bee and wasp stings should not affect the client during surgery. Lactose intolerance should also not affect the client during surgery but will need to be noted before a postoperative diet is ordered. Lack of experience with surgery may increase anxiety and may require higher teaching needs, but is not the priority over client safety.

4. A nurse cares for a client after a pituitary gland stimulation test using insulin. The clients post-stimulation laboratory results indicate elevated levels of growth hormone (GH) and adrenocorticotropic hormone (ACTH). How should the nurse interpret these results? a. Pituitary hypofunction b. Pituitary hyperfunction c. Pituitary-induced diabetes mellitus d. Normal pituitary response to insulin

ANS: D Some tests for pituitary function involve administering agents that are known to stimulate the secretion of specific pituitary hormones and then measuring the response. Such tests are termed stimulation tests. The stimulation test for GH or ACTH assessment involves injecting the client with regular insulin (0.05 to 1 unit/kg of body weight) and checking circulating levels of GH and ACTH. The presence of insulin in clients with normal pituitary function causes increased release of GH and ACTH.

11. A nurse cares for a client who has kidney stones from gout ricemia. Which medication does the nurse anticipate administering? a. Phenazopyridine b. Doxycyline c. Tolterodine d. Allopurinol

ANS: D Stones caused by hyperuricmia caused by gout or other reason respond to allopurinol. Phenazopyridine is given to clients with urinary tract infections. Doxycycline is an antibiotic. Tolterodine is an anticholinergic with smooth muscle-relaxant properties.

11. A nurse cares for a client who has kidney stones from secondary hyperoxaluria. Which medication should the nurse anticipate administering? a. Phenazopyridine (Pyridium) b. Propantheline (Pro-Banthine) c. Tolterodine (Detrol LA) d. Allopurinol (Zyloprim)

ANS: D Stones caused by secondary hyperoxaluria respond to allopurinol (Zyloprim). Phenazopyridine is given to clients with urinary tract infections. Propantheline is an anticholinergic. Tolterodine is an anticholinergic with smooth muscle relaxant properties.

2. A nurse assesses a client who is recovering from a total thyroidectomy and notes the development of stridor. Which action should the nurse take first? a. Reassure the client that the voice change is temporary. b. Document the finding and assess the client hourly. c. Place the client in high-Fowlers position and apply oxygen. d. Contact the provider and prepare for intubation.

ANS: D Stridor on exhalation is a hallmark of respiratory distress, usually caused by obstruction resulting from edema. One emergency measure is to remove the surgical clips to relieve the pressure. This might be a physician function. The nurse should prepare to assist with emergency intubation or tracheostomy while notifying the provider or the Rapid Response Team. Stridor is an emergency situation; therefore, reassuring the client, documenting, and reassessing in an hour do not address the urgency of the situation. Oxygen should be applied, but this action will not keep the airway open.

2. A nurse assesses a client who is recovering from a subtotal thyroidectomy and observes the development of stridor. What is the priority action for the nurse to take? a. Apply oxygen via nasal cannula at 2 L/min. b. Document the finding and assess the client hourly. c. Place the client in high-Fowler position in the bed. d. Contact the Rapid Response Team and prepare for intubation.

ANS: D Stridor on exhalation is a hallmark of respiratory distress, usually caused by obstruction resulting from edema. The nurse should prepare to assist with emergency intubation or tracheostomy while notifying the Rapid Response Team. Stridor is an emergency situation; therefore, reassuring the client, documenting, and reassessing in an hour do not address the urgency of the situation. Oxygen should be applied, but this action will not keep the airway open.

7. A client has just had a central line catheter placed that is specific for hemodialysis. What is the most appropriate action by the nurse? a. Use the catheter for the next laboratory blood draw. b. Monitor the central venous pressure through this line. c. Access the line for the next intravenous medication. d. Place a heparin or heparin/saline dwell after hemodialysis.

ANS: D The central line should have a heparin or heparin/saline dwell after hemodialysis treatment. The central line catheter used for dialysis should not be used for blood sampling, monitoring central venous pressures, or giving drugs or fluids.

21. The postoperative nurse is caring for a client who reports feeling "something popped" after vomiting. What action by the nurse is best? a. Administer an antiemetic medication. b. Call the primary health care provider. c. Instruct client to avoid coughing. d. Gather sterile nonadherent dressings.

ANS: D The client may have a wound dehiscence. The nurse would gather needed supplies and assess the wound under the dressing. If the incision has dehisced, the nurse would cover it with a sterile nonadherent dressing or saline-moistened gauze dressing then call the primary health care provider. The client may need an antiemetic, but this is not the most important action at this time.

35. The nurse is caring for a newly admitted older adult who has a blood glucose of 300 mg/dL (16.7 mmol/L), a urine output of 185 mL in the past 8 hours, and a blood urea nitrogen (BUN) of 44 mg/dL (15.7 mmol/L). What diabetic complication does the nurse suspect? a. Diabetic ketoacidosis (DKA) b. Severe hypoglycemia c. Chronic kidney disease (CKD) d. Hyperglycemic-hyperosmolar state (HHS)

ANS: D The client most likely has diabetes mellitus type 2 and has a high blood glucose causing increased blood osmolarity and dehydration, as evidenced by an insufficient urinary output and increased BUN. Older adults are at the greatest risk for dehydration due to age-related physiologic changes.

8. A client who has had cold symptoms for a week visits the local urgent care center with report of left ear discomfort, dizziness, and decreased hearing. What additional assessment findings would the nurse expect? a. High fever b. Nausea and vomiting c. Elevated blood pressure d. Purulent ear drainage

ANS: D The client presents with symptoms that indicate possible serous otitis or otitis media. In either case, the client would not have a high fever or blood pressure. Nausea and vomiting are not common with either diagnosis, but purulent ear drainage is likely to occur if the tympanic eardrum perforates. The client'GsRdeAcDreEasSeLdAheBa.rinCgOcMould indicate that perforation already occurred.

5. A nurse prepares to palpate a clients thyroid gland. Which action should the nurse take when performing this assessment? a. Stand in front of the client instead of behind the client. b. Ask the client to swallow after palpating the thyroid. c. Palpate the right lobe with the nurses left hand. d. Place the client in a sitting position with the chin tucked down.

ANS: D The client should be in a sitting position with the chin tucked down as the examiner stands behind the client. The nurse feels for the thyroid isthmus while the client swallows and turns the head to the right, and the nurse palpates the right lobe with the right hand. The technique is repeated in the opposite fashion for the left lobe.

13. The nurse is caring for a client with adrenal insufficiency. What priority physical assessment would the nurse perform? a. Respiratory assessment b. Skin assessment c. Neurologic assessment d. Cardiac assessment

ANS: D The client who has adrenal insufficiency has hyperkalemia that can cause cardiac dysrhythmias. Therefore, the nurse would monitor the client's cardiovascular status through frequent assessments.

6. The nurse teaches a client who has stress incontinence methods to regain more urinary continence. Which health teaching is the most important for the nurse to include for this client? a. What type of incontinence pads to use? b. What types of liquids to drink and when? c. Need to perform intermittent catheterizations. d. How to do Kegel exercises to strengthen muscles?

ANS: D The client who has stress incontinence needs to strengthen the muscles of the pelvic floor using Kegel exercises. Catheterizations would not help with incontinence. Incontinence pads may need to be used by this client but that is not the most important thing to teach, and it does not help the client regain more control over his or her bladder.

9. A client is being treated for diabetes insipidus (DI) with synthetic vasopressin (desmopressin). What is the priority health teaching that the nurse provides regarding drug therapy? a. The need to check the client's urinary specific gravity. b. The need to take blood pressure at least twice a day. c. The need to monitor blood glucose every day. d. The need to weigh every day and report weight gain.

ANS: D The client with DI who takes lifelong hormone replacement will need to report significant weight gain to monitor for water toxicity. Water toxicity causes headache, vomiting, and acute confusion.

21. After teaching a client with a fractured humerus, the nurse assesses the clients understanding. Which dietary choice demonstrates that the client correctly understands the nutrition needed to assist in healing the fracture? a. Baked fish with orange juice and a vitamin D supplement b. Bacon, lettuce, and tomato sandwich with a vitamin B supplement c. Vegetable lasagna with a green salad and a vitamin A supplement d. Roast beef with low-fat milk and a vitamin C supplement

ANS: D The client with a healing fracture needs supplements of vitamins B and C and a high-protein, high-calorie diet. Milk for calcium supplementation and vitamin C supplementation are appropriate. Meat would increase protein in the diet that is necessary for bone healing. Fish, a sandwich, and vegetable lasagna would provide less protein.

17. A nurse cares for a client who is scheduled for the surgical creation of an ileal conduit. The client states, I am anxious about having an ileal conduit. What is it like to have this drainage tube? How should the nurse respond? a. I will ask the provider to prescribe you an antianxiety medication. b. Would you like to discuss the procedure with your doctor once more? c. I think it would be nice to not have to worry about finding a bathroom. d. Would you like to speak with someone who has an ileal conduit?

ANS: D The goal for the client who is scheduled to undergo a procedure such as an ileal conduit is to have a positive self-image and a positive attitude about his or her body. Discussing the procedure candidly with someone who has undergone the same procedure will foster such feelings, especially when the current client has an opportunity to ask questions and voice concerns to someone with first-hand knowledge. Medications for anxiety will not promote a positive self-image and a positive attitude, nor will discussing the procedure once more with the physician or hearing the nurses opinion.

3. An older client who fell at home is admitted to the emergency department and reports pain in her left groin and behind her left knee. What action would the nurse anticipate? a. Administer IV push morphine. b. Prepare for application of a leg cast. c. Begin oxygen at 6 L/min via mask. d. Obtain a left hip x-ray.

ANS: D The location of the client's pain indicates a possible fractured hip and therefore an x-ray of the hip is needed. A leg cast is not appropriate and oxygen may not be needed. Medication to make the client more comfortable would likely be needed after a diagnosis is determined.

10. A nurse cares for a client who is recovering after a nephrostomy tube was placed 6 hours ago. The nurse notes drainage in the tube has decreased from 40 mL/hr to 12 mL over the last hour. Which action should the nurse take? a. Document the finding in the clients record. b. Evaluate the tube as working in the hand-off report. c. Clamp the tube in preparation for removing it. d. Assess the clients abdomen and vital signs.

ANS: D The nephrostomy tube should continue to have a consistent amount of drainage. If the drainage slows or stops, it may be obstructed. The nurse must notify the provider, but first should carefully assess the clients abdomen for pain and distention and check vital signs so that this information can be reported as well. The other interventions are not appropriate.

4. The nurse is performing a neurovascular assessment for an older client who has an extremity fracture. How many seconds would the nurse expect for a capillary refill in it is within normal range? a. 20 seconds b. 15 seconds c. 10 seconds d. 5 seconds

ANS: D The normal capillary refill is usually 3 seconds, but for older adults, the refill usually takes up to 5 seconds due to vascular changes associated with aging.

19. A client is admitted with a large draining wound on the leg. What action does the nurse take first? a. Administer ordered antibiotics. b. Insert an intravenous line. c. Give pain medications if needed. d. Obtain cultures of the leg wound.

ANS: D The nurse first obtains wound cultures prior to administering broad-spectrum antibiotics. The nurse would need to start the IV prior to giving the antibiotics as they will most likely be parenteral. Pain should be treated but that is not the priority.

8. After teaching a client newly diagnosed with epilepsy, the nurse assesses the clients understanding. Which statement by the client indicates a need for additional teaching? a. I will wear my medical alert bracelet at all times. b. While taking my epilepsy medications, I will not drink any alcoholic beverages. c. I will tell my doctor about my prescription and over-the-counter medications. d. If I am nauseated, I will not take my epilepsy medication.

ANS: D The nurse must emphasize that antiepileptic drugs must be taken even if the client is nauseous. Discontinuing the medication can predispose the client to seizure activity and status epilepticus. The client should not drink alcohol while taking seizure medications. The client should wear a medical alert bracelet and should make the doctor aware of all medications to prevent complications of polypharmacy.

26. A client with chronic kidney disease states, I feel chained to the hemodialysis machine. What is the nurses best response to the clients statement? a. That feeling will gradually go away as you get used to the treatment. b. You probably need to see a psychiatrist to see if you are depressed. c. Do you need help from social services to discuss financial aid? d. Tell me more about your feelings regarding hemodialysis treatment.

ANS: D The nurse needs to explore the clients feelings in order to help the client cope and enter a phase of acceptance or resignation. It is common for clients to be discouraged because of the dependency of the treatment, especially during the first year. Referrals to a mental health provider or social services are possibilities, but only after exploring the clients feelings first. Telling the client his or her feelings will go away is dismissive of the clients concerns.

10. A nurse is caring for a client who is recovering from an above-the-knee amputation. The client reports pain in the limb that was removed. How should the nurse respond? a. The pain you are feeling does not actually exist. b. This type of pain is common and will eventually go away. c. Would you like to learn how to use imagery to minimize your pain? d. How would you describe the pain that you are feeling?

ANS: D The nurse should ask the client to rate the pain on a scale of 0 to 10 and describe how the pain feels. Although phantom limb pain is common, the nurse should not minimize the pain that the client is experiencing by stating that it does not exist or will eventually go away. Antiepileptic drugs and antispasmodics are used to treat neurologic pain and muscle spasms after amputation. Although imagery may assist the client, the nurse must assess the clients pain before determining the best action.

5. A client with acute kidney injury has a blood pressure of 76/55 mm Hg. The health care provider ordered 1000 mL of normal saline to be infused over 1 hour to maintain perfusion. The client is starting to develop shortness of breath. What is the nurses priority action? a. Calculate the mean arterial pressure (MAP). b. Ask for insertion of a pulmonary artery catheter. c. Take the clients pulse. d. Slow down the normal saline infusion.

ANS: D The nurse should assess that the client could bNeUdRevSeINloGpiTnBg.CflOuiMd overload and respiratory distress and slow down the normal saline infusion. The calculation of the MAP also reflects perfusion. The insertion of a pulmonary artery catheter would evaluate the clients hemodynamic status, but this should not be the initial action by the nurse. Vital signs are also important after adjusting the intravenous infusion.

7. A nurse assesses a client who is recovering from a radical nephrectomy for renal cell carcinoma. The nurse notes that the clients blood pressure has decreased from 134/90 to 100/56 mm Hg and urine output is 20 mL for this past hour. Which action should the nurse take? a. Position the client to lay on the surgical incision. b. Measure the specific gravity of the clients urine. c. Administer intravenous pain medications. d. Assess the rate and quality of the clients pulse.

ANS: D The nurse should first fully assess the client for signs of volume depletion and shock, and then notify the provider. The radical nature of the surgery and the proximity of the surgery to the adrenal gland put the client at risk for hemorrhage and adrenal insufficiency. Hypotension is a clinical manifestation associated with both hemorrhage and adrenal insufficiency. Hypotension is particularly dangerous for the remaining kidney, which must receive adequate perfusion to function effectively. Re-positioning the client, measuring specific gravity, and administering pain medication would not provide data necessary to make an appropriate clinical decision, nor are they appropriate interventions at this time.

17. A nurse cares for a client who is having trouble voiding. The client states, I cannot urinate in public places. How should the nurse respond? a. I will turn on the faucet in the bathroom to help stimulate your urination. b. I can recommend a prescription for a diuretic to improve your urine output. c. Ill move you to a room with a private bathroom to increase your comfort. d. I will close the curtain to provide you with as much privacy as possible.

ANS: D The nurse should provide privacy to clients who may be uncomfortable or have issues related to elimination or the urogenital area. Turning on the faucet and administering a diuretic will not address the clients concern. Although moving the client to a private room with a private bathroom would be nice, this is not realistic. The nurse needs to provide as much privacy as possible within the clients current room.

12. A client is taking timolol eyedrops. The nurse assesses the client's pulse at 48 beats/min. What action by the nurse is the priority? a. Ask the client about excessive salivation. b. Take the client's blood pressure and temperature. c. Give the drops using punctal occlusion. d. Hold the eyedrops and notify the primary health care provider.

ANS: D The nurse would hold the eyedrops and notify the primary health care provider because beta blockers can slow the heart rate. Excessive salivation can occur with cholinergic agonists. Taking the blood pressure and temperature are not necessary. If the drops are given, the nurse uses punctal occlusion to avoid systemic absorption.

15. A client is scheduled to have a total hip arthroplasty. What preoperative teaching by the nurse is most important? a. Teach the need to discontinue all medications for 5 days before surgery. b. Teach the patient about foods high in protein, Vitamin C, and iron. c. Explain to the client the possible need for blood transfusions postoperatively. d. Remind the client to have all dental procedures completed at least 2 weeks prior to surgery.

ANS: D The nurse would include teaching about dental procedures to avoid infection after new joint has been inserted. Planned procedures would be completed at least 2 weeks before surgery and the client will need to tell any future primary health care providers about having a total joint arthroplasty. Only home medications prescribed that increase the risk for bleeding or clotting need to be discontinued 5 to 10 days before surgery. Clients need to be aware that any postoperative anemia may need to be treated with a blood transfusion, but it is not the most important. Diets high in protein, Vitamin C, and iron help with tissue repair, but are not the most important.

13. A nurse assesses a client with a pelvic fracture. Which assessment finding should the nurse identify as a complication of this injury? a. Hypertension b. Constipation c. Infection d. Hematuria

ANS: D The pelvis is very vascular and close to major organs. Injury to the pelvis can cause integral damage that may manifest as blood in the urine (hematuria) or stool. The nurse should also assess for signs of hemorrhage and hypovolemic shock, which include hypotension and tachycardia. Constipation and infection are not complications of a pelvic fracture.

19. A nurse cares for a client who has been diagnosed with the Huntington gene but has no symptoms. The client asks for options related to family planning. What is the nurses best response? a. Most clients with the Huntington gene do not pass on Huntington disease to their children. b. I understand that they can diagnose this disease in embryos. Therefore, you could select a healthy embryo from your fertilized eggs for implantation to avoid passing on Huntington disease. c. The need for family planning is limited because one of the hallmarks of Huntington disease is infertility. d. Tell me more specifically what information you need about family planning so that I can direct you to the right information or health care provider.

ANS: D The presence of the Huntington gene means that the trait will be passed on to all offspring of the affected person. Understanding options for contraception and conception (e.g., surrogacy options) and implications for children may require the expertise of a genetic counselor or a reproductive specialist. The other statements are not accurate.

20. The nurse administers epoetin alfa to a client who has chronic kidney disease (CKD). Which laboratory test value would the nurse monitor to determine this drug's effectiveness? a. Potassium b. Sodium c. Renin d. Hemoglobin

ANS: D The purpose of giving epoetin alfa to a client with CKD is to manage anemia by stimulating the bone marrow to produce more red blood cells. Therefore, monitoring the client's hemoglobin, hematocrit, and red blood cell count would indicate if the drug was effective.

20. A nurse cares for a client experiencing diabetic ketoacidosis who presents with Kussmaul respirations. What action would the nurse take? a. Administration of oxygen via facemask b. Intravenous administration of 10% glucose c. Implementation of seizure precautions d. Administration of intravenous insulin

ANS: D The rapid, deep respiratory efforts of Kussmaul respirations are the body's attempt to reduce the acids produced by using fat rather than glucose for fuel. Only the administration of insulin will reduce this type of respiration by assisting glucose to move into cells and to be used for fuel instead of fat. The patient who is in ketoacidosis may not experience any respiratory impairment and therefore does not need additional oxygen. Giving the patient glucose would be contraindicated. The patient does not require seizure precautions.

20.A nurse cares for a client experiencing diabetic ketoacidosis who presents with Kussmaul respirations. Which action should the nurse take? a. Administration of oxygen via face mask b. Intravenous administration of 10% glucose c. Implementation of seizure precautions d. Administration of intravenous insulin

ANS: D The rapid, deep respiratory efforts of Kussmaul respirations are the bodys attempt to reduce the acids produced by using fat rather than glucose for fuel. Only the administration of insulin will reduce this type of respiration by assisting glucose to move into cells and to be used for fuel instead of fat. The client who is in ketoacidosis may not experience any respiratory impairment and therefore does not need additional oxygen. Giving the client glucose would be contraindicated. The client does not require seizure precautions.

18. A client in the intensive care unit is scheduled for a lumbar puncture (LP) today. On assessment, the nurse finds the client breathing irregularly with one pupil fixed and dilated. What action by the nurse is best? a. Ensure that informed consent is on the chart. b. Document these findings in the clients record. c. Give the prescribed preprocedure sedation. d. Notify the provider of the findings immediately.

ANS: D This client is exhibiting signs of increased intracranial pressure. The nurse should notify the provider immediately because performing the LP now could lead to herniation. Informed consent is needed for an LP, but this is not the priority. Documentation should be thorough, but again this is not the priority. The preprocedure sedation (or other preprocedure medications) should not be given as the LP will most likely be canceled.

27. A nurse plans care for a client who is prescribed skeletal traction. Which intervention should the nurse include in this plan of care to decrease the clients risk for infection? a. Wash the traction lines and sockets once a day. b. Release traction tension for 30 minutes twice a day. c. Do not place the traction weights on the floor. d. Schedule for pin care to be provided every shift.

ANS: D To decrease the risk for infection in a client with skeletal traction of external fixation, the nurse should provide routine pin care and assess manifestations of infection at the pin sites every shift. The traction lines and sockets are external and do not come in contact with the clients skin; these do not need to be washed. Although traction weights should not be removed or released for any period of time without a prescription, or placed on the floor, this does not decrease the risk for infection.

7. A nurse delegates care of a client in traction to an unlicensed assistive personnel (UAP). Which statement should the nurse include when delegating hygiene care for this client? a. Remove the traction when re-positioning the client. b. Inspect the clients skin when performing a bed bath. c. Provide pin care by using alcohol wipes to clean the sites. d. Ensure that the weights remain freely hanging at all times.

ANS: D Traction weights should be freely hanging at all times. They should not be lifted manually or allowed to rest on the floor. The client should remain in traction during hygiene activities. The nurse should assess the clients skin and provide pin and wound care for a client who is in traction; this should not be delegated to the UAP.

20. A client has an ingrown toenail. About what self-management measure does the nurse teach the client? a. Long-term antibiotic use b. Shoe padding c. Toenail trimming d. Warm moist soaks

ANS: D Treatment of an ingrown toenail includes a podiatrist clipping away the ingrown part of the nail, warm moist soaks, and antibiotic ointment if needed. Antibiotics are not used long-term. Padding the shoes will not treat or prevent ingrown toenails. Clients should not attempt to trim ingrown nails themselves.

21.A nurse cares for a client who has type 1 diabetes mellitus. The client asks, Is it okay for me to have an occasional glass of wine? How should the nurse respond? a. Drinking any wine or alcohol will increase your insulin requirements. b. Because of poor kidney function, people with diabetes should avoid alcohol. c. You should not drink alcohol because it will make you hungry and overeat. d. One glass of wine is okay with a meal and is counted as two fat exchanges.

ANS: D Under normal circumstances, blood glucose levels will not be affected by moderate use of alcohol when diabetes is well controlled. Because alcohol can induce hypoglycemia, it should be ingested with or shortly after a meal. One alcoholic beverage is substituted for two fat exchanges when caloric intake is calculated. Kidney function is not impacted by alcohol intake. Alcohol is not associated with increased hunger or overeating.

13. A client continues to have persistent low back pain even after using a number of nonpharmacologic pain management strategies. Which prescribed drug would the nurse anticipate that the client might need to manage the pain? a. Oxycontin b. Gabapentin c. Lorazepam d. Tramadol

ANS: D When nonpharmacologic strategies, including physical therapy, are not effective in managing pain, current standards recommend a mild opioid such as tramadol or serotonin-norepinephrine reuptake inhibitor. Strong opioids such as oxycontin and benzodiazepines such as lorazepam are not considered best practice.

6. A nurse prepares to provide perineal care to a client with meningococcal meningitis. Which personal protective equipment should the nurse wear? (Select all that apply.) a. Particulate respirator b. Isolation gown c. Shoe covers d. Surgical mask e. Gloves

ANS: D, E Meningeal meningitis is spread via saliva and droplets, and Droplet Precautions are necessary. Caregivers should wear a surgical mask when within 6 feet of the client and should continue to use Standard Precautions, including gloves. A particulate respirator, an isolation gown, and shoe covers are not necessary for Droplet Precautions.

16. An emergency nurse cares for a client who is experiencing an acute adrenal crisis. Which action should the nurse take first? a. Obtain intravenous access. b. Administer hydrocortisone succinate (Solu-Cortef). c. Assess blood glucose. d. Administer insulin and dextrose.

ANS: A All actions are appropriate for the client with adrenal crisis. However, therapy is given intravenously, so the priority is to establish IV access. Solu-Cortef is the drug of choice. Blood glucose is monitored hourly and treatment is provided as needed. Insulin and dextrose are used to treat any hyperkalemia.

6. The nurse is teaching a client who is prescribed acetaminophen for control of osteoarthritic joint pain. What statement by the client indicates a need for further teaching? a. "I won't take more than 5000 mg of this drug each day." b. "I'll follow up to get my lab tests done to check my liver." c. "I'll check drugs that I take for acetaminophen in them." d. "I can use topical patches and creams to help relieve pain."

ANS: A All of the choices are correct about acetaminophen except that the maximum daily dosage is 4000 mg. For older adults, 3000 mg are recommended due to slower drug metabolism by the liver.

3. A clients chart indicates anisocoria. For what should the nurse assess? a. Difference in pupil size b. Draining infection c. Recent eye trauma d. Tumor of the eyelid

ANS: A Anisocoria is a noticeable difference in the size of a persons pupils. This is a normal finding in a small percentage of the population. Infection, trauma, and tumors are not related.

11. A home health nurse assesses a client with diabetes who has a new cast on the arm. The nurse notes the clients fingers are pale, cool, and slightly swollen. Which action should the nurse take first? a. Raise the arm above the level of the heart. b. Encourage range of motion. c. Apply heat to the affected hand. d. Bivalve the cast to decrease pressure.

ANS: A Arm casts can impair circulation when the arm is in the dependent position. The nurse should immediately elevate the arm above the level of the heart, ensuring that the hand is above the elbow, and should re-assess the extremity in 15 minutes. If the fingers are warmer and less swollen, the cast is not too tight and adjustments do not need to be made, but a sling should be worn when the client is upright. Encouraging range of motion would not assist the client as much as elevating the arm. Heat would cause increased edema and should not be used. If the cast is confirmed to be too tight, it could be bivalved.

15. A postoperative client has just been admitted to the postanesthesia care unit (PACU). What assessment by the PACU nurse takes priority? a. Airway b. Bleeding c. Breathing d. Cardiac rhythm

ANS: A Assessing the airway always takes priority, followed by breathing and circulation. Bleeding is part of the circulation assessment, as is cardiac rhythm.

21. A client has a brain abscess and is receiving phenytoin (Dilantin). The spouse questions the use of the drug, saying the client does not have a seizure disorder. What response by the nurse is best? a. Increased pressure from the abscess can cause seizures. b. Preventing febrile seizures with an abscess is important. c. Seizures always occur in clients with brain abscesses. d. This drug is used to sedate the client with an abscess.

ANS: A Brain abscesses can lead to seizures as a complication. The nurse should explain this to the spouse. Phenytoin is not used to prevent febrile seizures. Seizures are possible but do not always occur in clients with brain abscesses. This drug is not used for sedation.

8. The nurse is performing an assessment of cranial nerve III. Which testing is appropriate? a. Pupil constriction b. Deep tendon reflexes c. Upper muscle strength d. Speech and language

ANS: A CN III is the oculomotor nerve which controls eye movement, pupil constriction, and eyelid movement.

11. A nurse teaches a client who is scheduled for a positron emission tomography scan of the brain. Which statement should the nurse include in this clients teaching? a. Avoid caffeine-containing substances for 12 hours before the test. b. Drink at least 3 liters of fluid during the first 24 hours after the test. c. Do not take your cardiac medication the morning of the test. d. Remove your dentures and any metal before the test begins.

ANS: A Caffeine-containing liquids and foods are central nervous system stimulants and may alter the test results. No contrast is used; therefore, the client does not need to increase fluid intake. The client should take cardiac medications as prescribed. Metal does not have to be removed; this is done for magnetic resonance imaging.

12. A nurse reviews the allergy list of a client who is scheduled for an intravenous urography. Which client allergy should alert the nurse to urgently contact the health care provider? a. Seafood b. Penicillin c. Bee stings d. Red food dye

ANS: A Clients with seafood allergies often have severe allergic reactions to the standard dyes used during intravenous urography. The other allergies have no impact on the clients safety during an intravenous urography.

17. A nurse delegates care to the unlicensed assistive personnel (UAP). Which statement should the nurse include when delegating care for a client with cranial nerve II impairment? a. Tell the client where food items are on the breakfast tray. b. Place the client in a high-Fowlers position for all meals. c. Make sure the clients food is visually appetizing. d. Assist the client by placing the fork in the left hand.

ANS: A Cranial nerve II, the optic nerve, provides central and peripheral vision. A client who has cranial nerve II impairment will not be able to see, so the UAP should tell the client where different food items are on the meal tray. The other options are not appropriate for a client with cranial nerve II impairment.

9. A client with a stroke has damage to Brocas area. What intervention to promote communication is best for this client? a. Assess whether or not the client can write. b. Communicate using yes-or-no questions. c. Reinforce speech therapy exercises. d. Remind the client not to use neologisms.

ANS: A Damage to Brocas area often leads to expressive aphasia, wherein the client can understand what is said but cannot express thoughts verbally. In some instances the client can write. The nurse should assess to see if that ability is intact. Yes-or-no questions are not good for this type of client because he or she will often answer automatically but incorrectly. Reinforcing speech therapy exercises is good for all clients with communication difficulties. Neologisms are made-up words often used by clients with sensory aphasia.

2. The nurse reads on a clients chart that the client has exophthalmos. What assessment finding is consistent with this diagnosis? a. Bulging eyes b. Drooping eyelids c. Sunken-in eyes d. Yellow sclera

ANS: A Exophthalmos is bulging eyes. Drooping eyelids is ptosis. Sunken-in eyes is enophthalmos. Yellow sclera indicates jaundice.

16. The nurse is teaching a client with chronic kidney disease (CKD) about the sodium restriction needed in the diet to prevent edema and hypertension. Which statement by the client indicates more teaching is needed? a. I am thrilled that I can continue to eat fast food. b. I will cut out bacon with my eggs every morning. c. My cooking style will change by not adding salt. d. I will probably lose weight by cutting out potato chips.

ANS: A Fast food restaurants usually serve food that is high in sodium. This statement indicates that more teaching needs to occur. The other statements show a correct understanding of the teaching.

24. A nurse is providing community screening for risk factors associated with stroke. Which client would the nurse identify as being at highest risk for a stroke? a. A 27-year-old heavy cocaine user b. A 30-year-old who drinks a beer a day c. A 40-year-old who uses seasonal antihistamines d. A 65-year-old who is active and on no medications

ANS: A Heavy drug use, particularly cocaine, is a risk factor for stroke. Heavy alcohol use is also a risk factor, but one beer a day is not considered heavy drinking. Antihistamines may contain phenylpropanolamine, which also increases the risk for stroke, but this client uses them seasonally and there is no information that they are abused or used heavily. The 65-year-old has only age as a risk factor.

42.A nurse prepares to administer insulin to a client at 1800. The clients medication administration record contains the following information: Insulin glargine: 12 units daily at 1800 Regular insulin: 6 units QID at 0600, 1200, 1800, 2400 Based on the clients medication administration record, which action should the nurse take? a. Draw up and inject the insulin glargine first, and then draw up and inject the regular insulin. b. Draw up and inject the insulin glargine first, wait 20 minutes, and then draw up and inject the regular insulin. c. First draw up the dose of regular insulin, then draw up the dose of insulin glargine in the same syringe, mix, and inject the two insulins together. d. First draw up the dose of insulin glargine, then draw up the dose of regular insulin in the same syringe, mix, and inject the two insulins together.

ANS: A Insulin glargine must not be diluted or mixed with any other insulin or solution. Mixing results in an unpredictable alteration in the onset of action and time to peak action. The correct instruction is to draw up and

15.A nurse cares for a client with diabetes mellitus who is visually impaired. The client asks, Can I ask my niece to prefill my syringes and then store them for later use when I need them? How should the nurse respond? a. Yes. Prefilled syringes can be stored for 3 weeks in the refrigerator in a vertical position with the needle pointing up. b. Yes. Syringes can be filled with insulin and stored for a month in a location that is protected from light. c. Insulin reacts with plastic, so prefilled syringes are okay, but you will need to use glass syringes. d. No. Insulin syringes cannot be prefilled and stored for any length of time outside of the container.

ANS: A Insulin is relatively stable when stored in a cool, dry place away from light. When refrigerated, prefilled plastic syringes are stable for up to 3 weeks. They should be stored in the refrigerator in the vertical position with the needle pointing up to prevent suspended insulin particles from clogging the needle.

9. A nurse obtains a focused health history for a client who is suspected of having bacterial meningitis. Which question should the nurse ask? a. Do you live in a crowded residence? b. When was your last tetanus vaccination? c. Have you had any viral infections recently? d. Have you traveled out of the country in the last month?

ANS: A Meningococcal meningitis tends to occur in multiple outbreaks. It is most likely to occur in areas of high- density population, such as college dormitories, prisons, and military barracks. A tetanus vaccination would not place the client at increased risk for meningitis or protect the client from meningitis. A viral infection would not lead to bacterial meningitis but could lead to viral meningitis. Simply knowing if the client traveled out of the country does not provide enough information. The nurse should ask about travel to specific countries in which the disease is common, for example, sub-Saharan Africa.

1. An older client is hospitalized after an operation. When assessing the client for postoperative infection, the nurse places priority on which assessment? a. Change in behavior b. Daily white blood cell count c. Presence of fever and chills d. Tolerance of increasing activity

ANS: A Older people have an age-related decrease in immune system functioning and may not show classic signs of infection such as increased white blood cell count, fever and chills, or obvious localized signs of infection. A change in behavior often signals an infection or onset of other illness in the older client.

8. A nurse teaches a client who has been prescribed a 24-hour urine collection to measure excreted hormones. The client asks, Why do I need to collect urine for 24 hours instead of providing a random specimen? How should the nurse respond? a. This test will assess for a hormone secreted on a circadian rhythm. b. The hormone is diluted in urine; therefore, we need a large volume. c. We are assessing when the hormone is secreted in large amounts. d. To collect the correct hormone, you need to urinate multiple times.

ANS: A Some hormones are secreted in a pulsatile, or circadian, cycle. When testing for these substances, a collection that occurs over 24 hours will most accurately reflect hormone secretion. Dilution of hormones in urine, secretion of hormone amounts, and ability to collect the correct hormone are not reasons to complete a 24-hour urine test.

18. After delegating to an unlicensed assistive personnel (UAP) the task of completing a bladder scan examination for a client, the nurse evaluates the UAPs performance. Which action by the UAP indicates the nurse must provide additional instructions when delegating this task? a. Selecting the female icon for all female clients and male icon for all male clients b. Telling the client, This test measures the amount of urine in your bladder. c. Applying ultrasound gel to the scanning head and removing it when finished d. Taking at least two readings using the aiming icon to place the scanning head

ANS: A The UAP should use the female icon for women who have not had a hysterectomy. This allows the scanner to subtract the volume of the uterus from readings. If a woman has had a hysterectomy, the UAP should choose the male icon. The UAP should explain the procedure to the client, apply gel to the scanning head and clean it after use, and take at least two readings.

1. A client who had a surgical fractured femur repair reports new-onset shortness of breath and increased respirations. What is the nurse's first action? a. Place the client in a high-Fowler position. b. Document the client's oxygen saturation level. c. Start oxygen therapy at 2 L/min via nasal cannula. d. Contact the primary health care provider.

ANS: A The client is experiencing respiratory distress which could be due to pulmonary embolus, fat embolism syndrome, or anxiety. Regardless of the cause, the nurse would place the client in a sitting position first and then perform additional assessment. Oxygen would likely be needed, especially if the client's oxygen saturation was under 95%.

6. A nurse collaborates with an unlicensed assistive personnel (UAP) to provide care for a client who is prescribed a 24-hour urine specimen collection. Which statement should the nurse include when delegating this activity to the UAP? a. Note the time of the clients first void and collect urine for 24 hours. b. Add the preservative to the container at the end of the test. c. Start the collection by saving the first urine of the morning. d. It is okay if one urine sample during the 24 hours is not collected.

ANS: A The collection of a 24-hour urine specimen is often delegated to a UAP. The nurse must ensure that the UAP understands the proper process for collecting the urine. The 24-hour urine collection specimen is started after the clients first urination. The first urine specimen is discarded because there is no way to know how long it has been in the bladder, but the time of the clients first void is noted. The client adds all urine voided after that first discarded specimen during the next 24 hours. When the 24-hour mark is reached, the client voids one last time and adds this specimen to the collection. The preservative, if used, must be added to the container at the beginning of the collection. All urine samples need to be collected for the test results to be accurate.

6. A client has a serum potassium level of 6.5 mmol/L, a serum creatinine level of 2 mg/dL, and a urine output of 350 mL/day. What is the best action by the nurse? a. Place the client on a cardiac monitor immediately. b. Teach the client to limit high-potassium foods. c. Continue to monitor the clients intake and output. d. Ask to have the laboratory redraw the blood specimen.

ANS: A The priority action by the nurse should be to check the cardiac status with a monitor. High potassium levels can lead to dysrhythmias. The other choices are logical nursing interventions for acute kidney injury but not the best immediate action.

3. A male client comes into the emergency department with a serum creatinine of 2.2 mg/dL and a blood urea nitrogen (BUN) of 24 mL/dL. What question should the nurse ask first when taking this clients history? a. Have you been taking any aspirin, ibuprofen, or naproxen recently? b. Do you have anyone in your family with renal failure? c. Have you had a diet that is low in protein recently? d. Has a relative had a kidney transplant lately?

ANS: A There are some medications that are nephrotoxic, such as the nonsteroidal anti-inflammatory drugs ibuprofen, aspirin, and naproxen. This would be a good question to initially ask the client since both the serum creatinine and BUN are elevated, indicating some renal problems. A family history of renal failure and kidney transplantation would not be part of the questioning and could cause anxiety in the client. A diet high in protein could be a factor in an increased BUN.

23. A nurse assesses a client who presents with renal calculi. Which question should the nurse ask? a. Do any of your family members have this problem? b. Do you drink any cranberry juice? c. Do you urinate after sexual intercourse? d. Do you experience burning with urination?

ANS: A There is a strong association between family history and stone formation and recurrence. Nephrolithiasis is associated with many genetic variations; therefore, the nurse should ask whether other family members have also had renal stones. The other questions do not refer to renal calculi but instead are questions that should be asked of a client with a urinary tract infection.

20. A nurse assesses a client with the National Institutes of Health (NIH) Stroke Scale and determines the clients score to be 36. How should the nurse plan care for this client? a. The client will need near-total care. b. The client will need cuing only. c. The client will need safety precautions. d. The client will be discharged home.

ANS: A This client has severe neurologic deficits and will need near-total care. Safety precautions are important but do not give a full picture of the clients dependence. The client will need more than cuing to complete tasks. A home discharge may be possible, but this does not help the nurse plan care for a very dependent client.

14. What information does the nurse teach a women's group about osteoporosis? a. "Primary osteoporosis occurs in postmenopausal women due to lack of estrogen." b. "Men actually have higher rates of the disease but are underdiagnosed." c. "There is no way to prevent or slow osteoporosis after menopause." d. "Women and men have an equal chance of getting osteoporosis."

ANS: A Women are more at risk of developing primary osteoporosis after menopause due to the lack of estrogen. Men have a slower loss of bone after the age of 75. Many treatments are now available for women to slow osteoporosis after menopause.

7. A nurse teaches a client about self-care after experiencing a urinary calculus treated by lithotripsy. Which statements should the nurse include in this clients discharge teaching? (Select all that apply.) a. Finish the prescribed antibiotic even if you are feeling better. b. Drink at least 3 liters of fluid each day. c. The bruising on your back may take several weeks to resolve. d. Report any blood present in your urine. e. It is normal to experience pain and difficulty urinating.

ANS: A, B, C The client should be taught to finish the prescribed antibiotic to ensure that he or she does not get a urinary tract infection. The client should drink at least 3 liters of fluid daily to dilute potential stone-forming crystals, prevent dehydration, and promote urine flow. After lithotripsy, the client should expect bruising that may take several weeks to resolve. The client should also experience blood in the urine for several days. The client should report any pain, fever, chills, or difficulty with urination to the provider as these may signal the beginning of an infection or the formation of another stone.

3. A nurse cares for a client with a fracture injury. Twenty minutes after an opioid pain medication is administered, the client reports pain in the site of the fracture. Which actions should the nurse take? (Select all that apply.) a. Administer additional opioids as prescribed. b. Elevate the extremity on pillows. c. Apply ice to the fracture site. d. Place a heating pad at the site of the injury. e. Keep the extremity in a dependent position.

ANS: A, B, C The client with a new fracture likely has edema; elevating the extremity and applying ice probably will help in decreasing pain. Administration of an additional opioid within the dosage guidelines may be ordered. Heat will increase edema and may increase pain. Dependent positioning will also increase edema.

2. The nursing student studying the eye learns that which cranial nerves control its functions? (Select all that apply.) a. II b. III c. VI d. XII e. X

ANS: A, B, C The cranial nerves involved with eye function include II, III, IV, V, VI, and VII.

9. A client has a small-bore feeding tube (Dobhoff tube) inserted for continuous enteral feedings while recovering from a traumatic brain injury. What actions should the nurse include in the clients care? (Select all that apply.) a. Assess tube placement per agency policy. b. Keep the head of the bed elevated at least 30 degrees. c. Listen to lung sounds at least every 4 hours. d. Run continuous feedings on a feeding pump. e. Use blue dye to determine proper placement.

ANS: A, B, C, D All of these options are important for client safety when continuous enteral feedings are in use. Blue dye is not used because it can cause lung injury if aspirated.

2. A nurse assesses a client with nephrotic syndrome. For which clinical manifestations should the nurse assess? (Select all that apply.) a. Proteinuria b. Hypoalbuminemia c. Dehydration d. Lipiduria e. Dysuria f. Costovertebral angle (CVA) tenderness

ANS: A, B, D Nephrotic syndrome is caused by glomerular damage and is characterized by proteinuria (protein level higher than 3.5 g/24 hr), hypoalbuminemia, edema, and lipiduria. Fluid overload leading to edema and hypertension is common with nephrotic syndrome; dehydration does not occur. Dysuria is present with cystitis. CVA tenderness is present with inflammatory changes in the kidney.

5. A client is admitted with a bone tumor. The nurse finds the client weak and lethargic with decreased deep tendon reflexes. What actions by the nurse are best? (Select all that apply.) a. Assess the daily serum calcium level. b. Consult the provider about a loop diuretic. c. Institute seizure precautions for the client. d. Instruct the client to call for help out of bed. e. Place the client on a 1500-mL fluid restriction.

ANS: A, B, D The client is exhibiting manifestations of possible hypercalcemia. This disorder is treated with increased fluids and loop diuretics. The nurse should assess the calcium level, consult with the provider, and instruct the client to call for help getting out of bed due to possible fractures and weakness. The client does not need seizure precautions or fluid restrictions.

8. A nurse is working with many stroke clients. Which clients would the nurse consider referring to a mental health provider on discharge? (Select all that apply.) a. Client who exhibits extreme emotional lability b. Client with an initial National Institutes of Health (NIH) Stroke Scale score of 38 c. Client with mild forgetfulness and a slight limp d. Client who has a past hospitalization for a suicide attempt e. Client who is unable to walk or eat 3 weeks post-stroke

ANS: A, B, D, E Clients most at risk for post-stroke depression are those with a previous history of depression, severe stroke (NIH Stroke Scale score of 38 is severe), and post-stroke physical or cognitive impairment. The client with mild forgetfulness and a slight limp would be a low priority for this referral.

6. A nurse teaches a client with a history of calcium phosphate urinary stones. Which statements should the nurse include in this clients dietary teaching? (Select all that apply.) a. Limit your intake of food high in animal protein. b. Read food labels to help minimize your sodium intake. c. Avoid spinach, black tea, and rhubarb. d. Drink white wine or beer instead of red wine. e. Reduce your intake of milk and other dairy products.

ANS: A, B, E Clients with calcium phosphate urinary stones should be taught to limit the intake of foods high in animal protein, sodium, and calcium. Clients with calcium oxalate stones should avoid spinach, black tea, and rhubarb. Clients with uric acid stones should avoid red wine.

9. A nurse is caring for a client with meningitis. Which laboratory values should the nurse monitor to identify potential complications of this disorder? (Select all that apply.) a. Sodium level b. Liver enzymes c. Clotting factors d. Cardiac enzymes e. Creatinine level

ANS: A, C Inflammation associated with meningitis can stimulate the hypothalamus and result in excessive production of antidiuretic hormone. The nurse should monitor sodium levels for early identification of syndrome of inappropriate antidiuretic hormone. A systemic inflammatory response (SIR) can also occur with meningitis. A SIR can result in a coagulopathy that leads to disseminated intravascular coagulation. The nurse should monitor clotting factors to identify this complication. The other laboratory values are not specific to complications of meningitis.

7. A nurse assesses a client who experienced a spinal cord injury at the T5 level 12 hours ago. Which manifestations should the nurse correlate with neurogenic shock? (Select all that apply.) a. Heart rate of 34 beats/min b. Blood pressure of 185/65 mm Hg c. Urine output less than 30 mL/hr d. Decreased level of consciousness e. Increased oxygen saturation

ANS: A, C, D Neurogenic shock with acute spinal cord injury manifests with decreased oxygen saturation, symptomatic bradycardia, decreased level of consciousness, decreased urine output, and hypotension.

7. A nurse assesses a client with Cushings disease. Which assessment findings should the nurse correlate with this disorder? (Select all that apply.) a. Moon face b. Weight loss c. Hypotension d. Petechiae e. Muscle atrophy

ANS: A, D, E Clinical manifestations of Cushings disease include moon face, weight gain, hypertension, petechiae, and muscle atrophy.

5. A nursing student studying traumatic brain injuries (TBIs) should recognize which facts about these disorders? (Select all that apply.) a. A client with a moderate trauma may need hospitalization. b. A Glasgow Coma Scale score of 10 indicates a mild brain injury. c. Only open head injuries can cause a severe TBI. d. A client with a Glasgow Coma Scale score of 3 has severe TBI. e. The terms mild TBI and concussion have similar meanings.

ANS: A, D, E Mild TBI is a term used synonymously with the term concussion. A moderate TBI has a Glasgow Coma Scale (GCS) score of 9 to 12, and these clients may need to be hospitalized. Both open and closed head injuries can cause a severe TBI, which is characterized by a GCS score of 3 to 8.

6. The nurse assesses a client who has meningitis. Which sign(s) and symptom(s) would the nurse anticipate? (Select all that apply.) a. Photophobia b. Decreased level of consciousness c. Severe headache d. Fever and chills e. Bradycardia

ANS: A,B,C,D All of the choices except for bradycardia are key features of meningitis. Tachycardia is more likely than bradycardia due to the infectious process and fever.

10. The nurse is taking a history on an older adult. Which factors would the nurse assess as potential risks for low back pain? (Select all that apply.) a. Scoliosis b. Spinal stenosis c. Hypocalcemia d. Osteoporosis e. Osteoarthritis

ANS: A,B,C,D,E All of these factors place the client at risk for low back pain due to changes in spinal alignment, loss of bone, or joint degeneration. Bone loss worsens if serum calcium levels are below normal.

1. The nurse is caring for a client who has possible hypothyroidism. What possible risk factors can cause this health problem? (Select all that apply.) a. Lithium drug therapy b. Thyroid cancer c. Autoimmune thyroid disease d. Iodine deficiency e. Laryngitis f. Pituitary tumors

ANS: A,B,C,D,F All of these factors place a client at risk for hypothyroidism except for laryngitis which is an inflammation of the larynx.

6. A nurse learns older adults are at higher risk for complications after surgery. What reasons for this does the nurse understand? (Select all that apply.) a. Decreased cardiac output b. Decreased oxygenation c. Frequent nocturia d. Mobility alterations e. Inability to adapt to changes f. Slower reaction times

ANS: A,B,C,D,F Older adults have many age-related physiologic changes that put them at higher risk of falling and other complications after surgery. Some of these include decreased cardiac output, decreased oxygenation of tissues, nocturia, mobility alterations, and slower reaction times. They also have a decreased ability to adapt to new surroundings, but that is not the same as being unable to adapt.

2. The nurse is caring for a client who is diagnosed with urinary tract infection (UTI). What common urinary signs and symptoms does the nurse expect? (Select all that apply.) a. Dysuria b. Frequency c. Burning d. Fever e. Chills f. Hematuria

ANS: A,B,C,F Fever and chills may occur in clients who have a UTI if the infection has expanded beyond the bladder into the kidneys. However, these symptoms are not urinary signs and symptoms.

7. A client asks the nurse about what medications may be included for nonopioid multimodal analgesia following a total knee arthroplasty. What medications may be given to the client? (Select all that apply.) a. Gabapentin b. Ketorolac c. Hydrocodone d. Ketamine e. Morphine f. Bupivacaine

ANS: A,B,D,F All of the choices are appropriate to use for nonopioid multimodal analgesia except for the two opioid drugs—hydrocodone and morphine. The nonopioid medications are used to decrease inflammation and pain.

7. The nurse assesses a client who has Parkinson disease. Which signs and symptoms would the nurse recognize as a key feature of this disease? (Select all that apply.) a. Flexed trunk b. Long, extended steps c. Slow movements d. Uncontrolled drooling e. Tachycardia

ANS: A,C,D Key features of Parkinson disease include a flexed trunk, slow and hesitant steps, bradykinesia, and uncontrolled drooling. Tachycardia is not a key feature of this disease.

9. A nurse plans care for an older adult who is admitted to the hospital for pneumonia. The client has no known drug allergies and no significant health history. Which action should the nurse include in this clients plan of care? a. Initiate Airborne Precautions. b. Offer fluids every hour or two. c. Place an indwelling urinary catheter. d. Palpate the clients thyroid gland.

ANS: B A normal age-related endocrine change is decreased antidiuretic hormone (ADH) production. This results in a more diluted urine output, which can lead to dehydration. If no contraindications are known, the nurse should offer (or delegate) the client something to drink at least every 2 hours. A client with simple pneumonia would not require Airborne Precautions. Indwelling urinary catheterization is not necessary for this client and would increase the clients risk for infection. The nurse should plan a toileting schedule and assist the client to the bathroom if needed. Palpating the clients thyroid gland is a part of a comprehensive examination but is not specifically related to this client.

15. A nurse assesses a client after administering prescribed levetiracetam (Keppra). Which laboratory tests should the nurse monitor for potential adverse effects of this medication? a. Serum electrolyte levels b. Kidney function tests c. Complete blood cell count d. Antinuclear antibodies

ANS: B Adverse effects of levetiracetam include coordination problems and renal toxicity. The other laboratory tests are not affected by levetiracetam.

14. After teaching a client who is recovering from a complete thyroidectomy, the nurse assesses the clients understanding. Which statement made by the client indicates a need for additional instruction? a. I may need calcium replacement after surgery. b. After surgery, I wont need to take thyroid medication. c. Ill need to take thyroid hormones for the rest of my life. d. I can receive pain medication if I feel that I need it.

ANS: B After the client undergoes a thyroidectomy, the client must be given thyroid replacement medication for life. He or she may also need calcium if the parathyroid is damaged during surgery, and can receive pain medication postoperatively.

3. The nurse assesses a client with diabetes and osteoarthritis (OA) during a checkup. The nurse notes the client's blood glucose readings have been elevated. What question by the nurse is most appropriate? a. "Are you following the prescribed diabetic diet?" b. "Have you been taking glucosamine supplements?" c. "How much exercise do you really get each week?" d. "You're still taking your diabetic medication, right?"

ANS: B All of the topics are appropriate for a client whose blood glucose readings have been higher than usual. However, since this client also has OA, and glucosamine can increase blood glucose levels, the nurse would ask about its use. The other questions all have an element of nontherapeutic communication in them. Asking how much exercise the client "really" gets is or if the diet is being followed is accusatory. Asking if the client takes his or her medications "right?" is patronizing.

24. A client is having a peritoneal dialysis treatment. The nurse notes an opaque color to the effluent. What is the priority action by the nurse? a. Warm the dialysate solution in a microwave before instillation. b. Take a sample of the effluent and send to the laboratory. c. Flush the tubing with normal saline to maintain patency of the catheter. d. Check the peritoneal catheter for kinking and curling.

ANS: B An opaque or cloudy effluent is the first sign of peritonitis. A sample of the effluent would need to be sent to the laboratory for culture and sensitivity in order to administer the correct antibiotic. Warming the dialysate in a microwave and flushing the tubing are not safe actions by the nurse. Checking the catheter for obstruction is a viable option but will not treat the peritonitis.

8. A client is admitted to the emergency department with a fractured femur resulting from a motor vehicle crash. What the nurse's priority action? a. Keep the client warm and comfortable. b. Assess airway, breathing, and circulation. c. Maintain the client in a supine position. d. Immobilize the injured extremity with a splint

ANS: B As part of the primary survey, the nurse would ensure that the client does not have any life-threatening problem by assessing the ABCs first. If there are not major problems, then the nurse could attend to the injured extremity.

12. A nurse cares for a client who has excessive catecholamine release. Which assessment finding should the nurse correlate with this condition? a. Decreased blood pressure b. Increased pulse c. Decreased respiratory rate d. Increased urine output

ANS: B Catecholamines are responsible for the fight-or-flight stress response. Activation of the sympathetic nervous system can be correlated with tachycardia. Catecholamines do not decrease blood pressure or respiratory rate, nor do they increase urine output.

7. A nurse is caring for a patient who has excessive catecholamine release. Which assessment finding would the nurse correlate with this condition? a. Decreased blood pressure b. Increased pulse c. Decreased respiratory rate d. Increased urine output

ANS: B Catecholamines are responsible for the fight-or-flight stress response. Activation of the sympathetic nervous system can be correlated with tachycardia. Catecholamines do not decrease blood pressure or respiratory rate, nor do they increase urine output.

8. An emergency department nurse assesses a client with kidney trauma and notes that the clients abdomen is tender and distended and blood is visible at the urinary meatus. Which prescription should the nurse consult the provider about before implementation? a. Assessing vital signs every 15 minutes b. Inserting an indwelling urinary catheter c. Administering intravenous fluids at 125 mL/hr d. Typing and crossmatching for blood products

ANS: B Clients with blood at the urinary meatus should not have a urinary catheter inserted via the urethra before additional diagnostic studies are done. The urethra could be torn. The nurse should question the provider about the need for a catheter; if one is needed, the provider can insert a suprapubic catheter. The nurse should monitor the clients vital signs closely, send blood for type and crossmatch in case the client needs blood products, and administer intravenous fluids.

22. A nurse plans care for clients with urinary incontinence. Which client is correctly paired with the appropriate intervention? a. A 29-year-old client after a difficult vaginal delivery Habit training b. A 58-year-old postmenopausal client who is not taking estrogen therapy Electrical stimulation c. A 64-year-old female with Alzheimers-type senile dementia Bladder training d. A 77-year-old female who has difficulty ambulating Exercise therapy

ANS: B Exercise therapy and electrical stimulation are used for clients with stress incontinence related to childbirth or low levels of estrogen after menopause. Exercise therapy increases pelvic wall strength; it does not improve ambulation. Physical therapy and a bedside commode would be appropriate interventions for the client who has difficulty ambulating. Habit training is the type of bladder training that will be most effective with cognitively impaired clients. Bladder training can be used only with a client who is alert, aware, and able to resist the urge to urinate.

9. The nurse is teaching a client about factors that can cause external otitis. Which of these factors would the nurse emphasize as the highest risk? a. Excess cerumen b. Swimming c. Sinus congestion d. Meniere disease

ANS: B External otitis is often called "swimmer's ear" because it is most often caused by swimming in lakes, ponds, and untreated pools.

2. A nurse assesses a client with multiple sclerosis after administering prescribed fingolimod. For which common side effect would the nurse monitor? a. Peripheral edema b. Facial flushing c. Tachycardia d. Fever

ANS: B Fingolimod is an oral immunomodulator that has two common side effects—facial flushing and GI disturbance, such as diarrhea. Peripheral edema, tachycardia, and fever are not common side effects of this drug.

7. A nurse assesses a female client who presents with hirsutism. Which question should the nurse ask when assessing this client? a. How do you plan to pay for your treatments? b. How do you feel about yourself? c. What medications are you prescribed? d. What are you doing to prevent this from happening?

ANS: B Hirsutism, or excessive hair growth on the face and body, can result from endocrine disorders. This may cause a disruption in body image, especially for female clients. The nurse should inquire into the clients body image and self-perception. Asking about the clients financial status or current medications does not address the clients immediate problem. The client is not doing anything to herself to cause the problem, nor can the client prevent it from happening.

1. A nurse cares for a client who is prescribed a drug that blocks a hormones receptor site. Which therapeutic effect should the nurse expect? a. Greater hormone metabolism b. Decreased hormone activity c. Increased hormone activity d. Unchanged hormone response

ANS: B Hormones cause activity in the target tissues by binding with their specific cellular receptor sites, thereby changing the activity of the cell. When receptor sites are occupied by other substances that block hormone binding, the cells response is the same as when the level of the hormone is decreased.

11. A nurse cares for a client with adrenal hyperfunction. The client screams at her husband, bursts into tears, and throws her water pitcher against the wall. She then tells the nurse, I feel like I am going crazy. How should the nurse respond? a. I will ask your doctor to order a psychiatric consult for you. b. You feel this way because of your hormone levels. c. Can I bring you information about support groups? d. I will close the door to your room and restrict visitors.

ANS: B Hypercortisolism can cause the client to show neurotic or psychotic behavior. The client needs to know that these behavior changes do not reflect a true psychiatric disorder and will resolve when therapy results in lower and steadier blood cortisol levels. The client needs to understand this effect and does not need a psychiatrist, support groups, or restricted visitors at this time.

10. A nurse plans care for a client with hyperparathyroidism. Which intervention should the nurse include in this clients plan of care? a. Ask the client to ambulate in the hallway twice a day. b. Use a lift sheet to assist the client with position changes. c. Provide the client with a soft-bristled toothbrush for oral care. d. Instruct the unlicensed assistive personnel to strain the clients urine for stones.

ANS: B Hyperparathyroidism causes increased resorption of calcium from the bones, increasing the risk for pathologic fractures. Using a lift sheet when moving or positioning the client, instead of pulling on the client, reduces the risk of bone injury. Hyperparathyroidism can cause kidney stones, but not every client will need to have urine strained. The priority is preventing injury. Ambulating in the hall and using a soft toothbrush are not specific interventions for this client.

2. A nurse plans care for a client who has a hypoactive response to a test of deep tendon reflexes. Which intervention should the nurse include in this clients plan of care? a. Check bath water temperature with a thermometer. b. Provide the client with assistance when ambulating. c. Place elastic support hose on the clients legs. d. Assess the clients feet for wounds each shift.

ANS: B Hypoactive deep tendon reflexes and loss of vibration sense can impair balance and coordination, predisposing the client to falls. The nurse should plan to provide the client with ambulation assistance to prevent injury. The other interventions do not address the clients problem.

14. A client with rheumatoid arthritis (RA) has an acutely swollen, red, and painful joint. What nonpharmacologic intervention does the nurse recommend? a. Heating pad b. Ice packs c. Splint d. Paraffin dip

ANS: B Ice is best for acute inflammation. Heat often helps with joint stiffness. Splinting helps preserve joint function. A paraffin dip is used to provide warmth to the joint which is more appropriate for chronic pain and stiffness.

9. The nurse is caring for four clients with chronic kidney disease. Which client should the nurse assess first upon initial rounding? a. Woman with a blood pressure of 158/90 mm Hg b. Client with Kussmaul respirations c. Man with skin itching from head to toe d. Client with halitosis and stomatitis

ANS: B Kussmaul respirations indicate a worsening of chronic kidney disease (CKD). The client is increasing the rate and depth of breathing to excrete carbon dioxide through the lungs. Hypertension is common in most clients with CKD, and skin itching increases with calcium-phosphate imbalances, another common finding in CKD. Uremia from CKD causes ammonia to be formed, resulting in the common findings of halitosis and stomatitis.

1. The nurse is planning health teaching for a client starting on levothyroxine. What health teaching about this drug would the nurse include? a. The need to take the drug when the client feels fatigued and weak. b. The need to report chest pain and dyspnea when starting the drug. c. The need to check blood pressure and pulse every day. d. The need to rotate injection sites when giving self the drug.

ANS: B Levothyroxine is a replacement hormone for clients who have hypothyroidism and is taken orally for life. Vital signs do not have to be checked every day, but the client should report any chest pain and dyspnea when first starting the drug.

2. A nurse reviews the health history of a client with an oversecretion of renin. Which disorder should the nurse correlate with this assessment finding? a. Alzheimers disease b. Hypertension c. Diabetes mellitus d. Viral hepatitis

ANS: B Renin is secreted when special cells in the distal convoluted tubule, called the macula densa, sense changes in blood volume and pressure. When the macula densa cells sense that blood volume, blood pressure, or blood sodium levels are low, renin is secreted. Renin then converts angiotensinogen into angiotensin I. This leads to a series of reactions that cause secretion of the hormone aldosterone. This hormone increases kidney reabsorption of sodium and water, increasing blood pressure, blood volume, and blood sodium levels. Inappropriate or excessive renin secretion is a major cause of persistent hypertension. Renin has no impact on Alzheimers disease, diabetes mellitus, or viral hepatitis.

30.A nurse develops a dietary plan for a client with diabetes mellitus and new-onset microalbuminuria. Which component of the clients diet should the nurse decrease? a. Carbohydrates b. Proteins c. Fats d. Total calories

ANS: B Restriction of dietary protein to 0.8 g/kg of body weight per day is recommended for clients with microalbuminuria to delay progression to renal failure. The clients diet does not need to be decreased in carbohydrates, fats, or total calories.

16. The nurse assesses a client with a history of urinary incontinence who presents with extreme dry mouth, constipation, and an inability to void. Which question would the nurse ask first? a. "Are you drinking plenty of water?" b. "What medications are you taking?" c. "Have you tried laxatives or enemas?" d. "Has this type of thing ever happened before?"

ANS: B Some types of incontinence or other health problems are treated with anticholinergic medications. Anticholinergic side effects include dry mouth, constipation, and urinary retention. The nurse needs to assess the client's medication list to determine whether the he or she is taking an anticholinergic medication. The other questions are not as helpful to understanding the current situation.

2. The student nurse is performing a Weber tuning fork test. What technique is most appropriate? a. Holding the vibrating tuning fork 10 to 12 inches from the clients ear b. Placing the vibrating fork in the middle of the clients head c. Starting by placing the vibrating fork on the mastoid process d. Tapping the vibrating tuning fork against the bridge of the nose

ANS: B The Weber tuning fork test includes placing the vibrating tuning fork in the middle of the clients head and asking in which ear the client hears the vibrations louder. The other techniques are incorrect.

2. The nurse is teaching assistant personnel (AP) about care of an older ambulatory adult who has osteopenia. Which statement by the AP indicates understanding of the teaching? a. "I will tell the client to change positions frequently to prevent pressure injury." b. "I will remind the client to take frequent walks to strengthen bones." c. "I will assist the client with activities of daily living as needed." d. "I will apply warm compresses to the joints to relieve pain."

ANS: B The ambulatory client who has osteopenia has experienced bone loss. Therefore, taking walks as a weight-bearing exercise helps to prevent further bone loss. The client does not have joint pain and does not need assistance or position changes because the client is ambulatory and probably independent.

6. When assessing a client who had a traumatic brain injury, the nurse notes that the client is drowsy but easily aroused. What level of consciousness will the nurse document to describe this client's current level of consciousness? a. Alert b. Lethargic c. Stuporous d. Comatose

ANS: B The client is categorized as being lethargic because he or she can be easily aroused even though drowsy. The nurse would carefully monitor the client to determine any decrease in the level of consciousness (LOC).

5. A nurse witnesses a client begin to experience a tonic-clonic seizure and loss of consciousness. Which action should the nurse take? a. Start fluids via a large-bore catheter. b. Turn the clients head to the side. c. Administer IV push diazepam. d. Prepare to intubate the client.

ANS: B The nurse should turn the clients head to the side to prevent aspiration and allow drainage of secretions. Anticonvulsants are administered on a routine basis if a seizure is sustained. If the seizure is sustained (status epilepticus), the client must be intubated and should be administered oxygen, 0.9% sodium chloride, and IV push lorazepam or diazepam.

5. A nurse cares for a client who presents with bradycardia secondary to hypothyroidism. Which medication should the nurse anticipate being prescribed to the client? a. Atropine sulfate b. Levothyroxine sodium (Synthroid) c. Propranolol (Inderal) d. Epinephrine (Adrenalin)

ANS: B The treatment for bradycardia from hypothyroidism is to treat the hypothyroidism using levothyroxine sodium. If the heart rate were so slow that it became an emergency, then atropine or epinephrine might be an option for short-term management. Propranolol is a beta blocker and would be contraindicated for a client with bradycardia.

5. A nurse cares for a client who presents with bradycardia secondary to hypothyroidism. Which medication does the nurse prepare to administer? a. Atropine sulfate b. Levothyroxine c. Propranolol d. Epinephrine

ANS: B The treatment for bradycardia from hypothyroidism is to treat the hypothyroidism using levothyroxine. If the heart rate were so slow that it became an emergency, then atropine or epinephrine might be an option for short-term management. Propranolol is a beta blocker and would be contraindicated for a client with bradycardia.

15. A client with osteoporosis is going home where the client lives alone. What action by the nurse is best? a. Refer the client to Meals on Wheels. b. Arrange a home safety evaluation. c. Ensure that the client has a walker at home. d. Help the client look into assisted living.

ANS: B This client has several risk factors that place him or her at a high risk for falling. The nurse should consult social work or home health care to conduct a home safety evaluation. The other options may or may not be needed based upon the client's condition at discharge.

3. An older clients serum calcium level is 8.7 mg/dL. What possible etiologies does the nurse consider for this result? (Select all that apply.) a. Good dietary intake of calcium and vitamin D b. Normal age-related decrease in serum calcium c. Possible occurrence of osteoporosis or osteomalacia d. Potential for metastatic cancer or Pagets disease e. Recent bone fracture in a healing stage

ANS: B, C This slightly low calcium level could be an age-related decrease in serum calcium or could indicate a metabolic bone disease such as osteoporosis or osteomalacia. A good dietary intake would be expected to produce normal values. Metastatic cancer, Pagets disease, or healing bone fractures will elevate calcium.

4. A nurse assesses a client who is recovering from a nephrostomy. Which assessment findings should alert the nurse to urgently contact the health care provider? (Select all that apply.) a. Clear drainage b. Bloody drainage at site c. Client reports headache d. Foul-smelling drainage e. Urine draining from site

ANS: B, D, E After a nephrostomy, the nurse should assess the client for complications and urgently notify the provider if drainage decreases or stops, drainage is cloudy or foul-smelling, the nephrostomy sites leaks blood or urine, or the client has back pain. Clear drainage is normal. A headache would be an unrelated finding.

6. A nurse assesses an older client. Which assessment findings should the nurse identify as normal changes in the nervous system related to aging? (Select all that apply.) a. Long-term memory loss b. Slower processing time c. Increased sensory perception d. Decreased risk for infection e. Change in sleep patterns

ANS: B, E Normal changes in the nervous system related to aging include recent memory loss, slower processing time, decreased sensory perception, an increased risk for infection, changes in sleep patterns, changes in perception of pain, and altered balance and/or decreased coordination.

7. The nurse is caring for a client who just had a kyphoplasty. What nursing care is needed for the client at this time? (Select all that apply.) a. Place the client in a prone position to prevent pressure on the surgical area. b. Apply an ice pack to the surgical area to help relieve pain. c. Assess the client's pain level to compare it with pain before the procedure. d. Take vital signs, including oxygen saturation, frequently. e. Monitor for complications such as bleeding or shortness of breath. f. Perform frequent neurologic assessments and report major changes.

ANS: B,C,D,E,F All of the choices are correct except that the client should stay in a flat supine position immediately after the procedure.

3. The nurse is teaching a client about preventing intraocular pressure increase after cataract surgery. Which health teaching would the nurse include? (Select all that apply.) a. "Don't lift objects weighing more than 20 lb (9.1 kg)." b. "Avoid blowing your nose or sneezing." c. "Don't bend down from the waist." d. "Don't strain to have a bowel movement." e. "Avoid having sexual intercourse." f. "Don't wear tight shirt or blouse collars."

ANS: B,C,D,E,F All of these precautions can help prevent an increase in intraocular pressure except that the client should not lift anything weighing more than 10 lb (4.5 kg).

7. A nurse working in the preoperative holding area performs which functions to ensure client safety? (Select all that apply.) a. Allow small sips of plain water. b. Check that consent is on the chart. c. Ensure that the client has an armband on. d. Have the client help mark the surgical site. e. Allow the client to use the toilet before giving sedation. f. Assess the client for fall risks.

ANS: B,C,D,E,F Providing for client safety is a priority function of the preoperative nurse. Checking for appropriately completed consent, verifying the client's identity, having the client assist in marking the surgical site if applicable, assessing for fall risk, and allowing the client to use the toilet prior to sedating him or her are just some examples of important safety measures. The preoperative client should be NPO, so water should not be provided unless an oral medication is ordered to be given in pre-op.

2. An 84-year-old client who is usually alert and oriented experiences an acute cognitive decline. Which of the following factors would the nurse anticipate as contributing to this neurologic change? (Select all that apply.) a. Chronic hearing loss b. Infection c. Drug toxicity d. Dementia e. Hypoxia f. Aging

ANS: B,C,E Acute client conditions that occur in older adults often cause acute confusion and associated emotional behaviors. Infection, drug toxicity, and hypoxia are all acute health problems that can contribute to the client's cognitive decline. Aging does not cause changes in cognition. If the client had dementia, he or she would not be alert and oriented. Having a chronic hearing loss is not a change in the client's condition.

10. A nurse on the postoperative nursing unit provides care to reduce the incidence of surgical wound infection. What actions are best to achieve this goal? (Select all that apply.) a. Administering antibiotics for 72 hours b. Disposing of dressings properly c. Leaving draining wounds open to air d. Performing proper hand hygiene e. Removing and replacing wet dressings

ANS: B,D,E Interventions necessary to prevent surgical wound infection include proper disposal of soiled dressings, performing proper hand hygiene, and removing wet dressings as they can be a source of infection. Prophylactic antibiotics may be given to clients at risk for infection, but not all clients need them for 72 hours. Draining wounds would always be covered.

4. A nurse assesses an older client. Which assessment findings would the nurse identify as normal changes in the nervous system related to aging? (Select all that apply.) a. Long-term memory loss b. Slower processing time c. Increased sensory perception d. Decreased risk for infection e. Change in sleep patterns

ANS: B,E Normal changes in the nervous system related to aging include recent memory loss, slower processing time, decreased sensory perception, an increased risk for infection, changes in sleep patterns, changes in perception of pain, and altered balance and/or decreased coordination.

13. A client had a surgical procedure with spinal anesthesia. The client's blood pressure was 122/78 mm Hg 30 minutes ago and is now 138/60 and the client reports nausea. What action by the nurse is best? a. Call the Rapid Response Team. b. Increase the IV fluid rate. c. Notify the primary health care provider. d. Nothing; this is expected.

ANS: C A widening pulse pressure (44 to 78 mm Hg) and nausea may indicate autonomic blockade, a complication of spinal anesthesia causing widespread vasodilation. The nurse would notify the primary health care provider. The Rapid Response Team is not yet warranted; the nurse would not increase the IV rate without a prescription.

8. A client is prescribed celecoxib for joint pain. What statement by the client indicates a need for further teaching? a. "I'll report any signs of bleeding or bruising to my primary health care provider." b. "I'll take this drug only as prescribed by my primary health care provider." c. "I'll be sure to take this drug three times a day only on an empty stomach." d. "I'll monitor the amount of urine that I excrete every day and report any changes."

ANS: C All of the choices are correct for this NSAID except that celecoxib can cause GI distress unless taken with meals or food. The drug should not be taken on an empty stomach and is rarely taken more than twice a day.

1. The nurse assesses an older client. What age-related physiologic changes would the nurse expect? a. Heat intolerance b. Rheumatoid arthritis c. Dehydration d. Increased appetite

ANS: C As people age, the many of the endocrine glands decrease hormone production, including a decrease in antidiuretic hormone production. This change, in addition to less body fluid being present as one ages, can cause dehydration. Older adults usually have cold intolerance and a decrease in appetite. Rheumatoid arthritis is not an age-related change; osteoarthritis causes primarily by aging.

20. A nurse is teaching a client with chronic migraine headaches. Which statement related to complementary therapy should the nurse include in this clients teaching? a. Place a warm compress on your forehead at the onset of the headache. b. Wear dark sunglasses when you are in brightly lit spaces. c. Lie down in a darkened room when you experience a headache. d. Set your alarm to ensure you do not sleep longer than 6 hours at one time.

ANS: C At the onset of a migraine attack, the client may be able to alleviate pain by lying down and darkening the room. He or she may want both eyes covered and a cool cloth on the forehead. If the client falls asleep, he or she should remain undisturbed until awakening. The other options are not recognized therapies for migraines.

22. A nurse delegates care for a client with early-stage Alzheimers disease to an unlicensed assistive personnel (UAP). Which statement should the nurse include when delegating this clients care? a. If she is confused, play along and pretend that everything is okay. b. Remove the clock from her room so that she doesnt get confused. c. Reorient the client to the day, time, and environment with each contact. d. Use validation therapy to recognize and acknowledge the clients concerns.

ANS: C Clients who have early-stage Alzheimers disease should be reoriented frequently to person, place, and time. The UAP should reorient the client and not encourage the clients delusions. The room should have a clock and white board with the current date written on it. Validation therapy is used with late-stage Alzheimers disease.

4. A client with multiple sclerosis is being discharged from rehabilitation. Which statement would the nurse include in the client's discharge teaching? a. "Be sure that you use a wheelchair when you go out in public." b. "Wear an undergarment brief at all times in case of incontinence." c. "Avoid overexertion, stress, and extreme temperature if possible." d. "Avoid having sexual intercourse to conserve energy."

ANS: C Clients who have multiple sclerosis have chronic fatigue and are prone to disease exacerbation (flare-up) is they overexert, are stressed, or are exposed to extreme temperature and humidity. They should not wear briefs unless they have actual problems with continence and should not use a wheelchair if they are able to ambulate with a cane or walker. Maintaining independence and self-esteem is important, so participating in sexual activities is encouraged.

4.A nurse assesses a client who has a 15-year history of diabetes and notes decreased tactile sensation in both feet. Which action should the nurse take first? a. Document the finding in the clients chart. b. Assess tactile sensation in the clients hands. c. Examine the clients feet for signs of injury. d. Notify the health care provider.

ANS: C Diabetic neuropathy is common when the disease is of long duration. The client is at great risk for injury in any area with decreased sensation because he or she is less able to feel injurious events. Feet are common locations for neuropathy and injury, so the nurse should inspect them for any signs of injury. After assessment, the nurse should document findings in the clients chart. Testing sensory perception in the hands may or may not be needed. The health care provider can be notified after assessment and documentation have been completed.

13. A nurse assesses a client with early-onset multiple sclerosis (MS). Which clinical manifestation should the nurse expect to find? a. Hyperresponsive reflexes b. Excessive somnolence c. Nystagmus d. Heat intolerance

ANS: C Early signs and symptoms of MS include changes in motor skills, vision, and sensation. Hyperresponsive reflexes, excessive somnolence, and heat intolerance are later manifestations of MS.

2. A nurse plans care for a client with a growth hormone deficiency. Which action should the nurse include in this clients plan of care? a. Avoid intramuscular medications. b. Place the client in protective isolation. c. Use a lift sheet to re-position the client. d. Assist the client to dangle before rising.

ANS: C In adults, growth hormone is necessary to maintain bone density and strength. Adults with growth hormone deficiency have thin, fragile bones. Avoiding IM medications, using protective isolation, and assisting the client as he or she moves from sitting to standing will not serve as safety measures when the client is deficient in growth hormone.

3. A nurse teaches an 80-year-old client with diminished touch sensation. Which statement should the nurse include in this clients teaching? a. Place soft rugs in your bathroom to decrease pain in your feet. b. Bathe in warm water to increase your circulation. c. Look at the placement of your feet when walking. d. Walk barefoot to decrease pressure ulcers from your shoes.

ANS: C Older clients with decreased sensation are at risk of injury from the inability to sense changes in terrain when walking. To compensate for this loss, the client is instructed to look at the placement of her or his feet when walking. Throw rugs can slip and increase fall risk. Bath water that is too warm places the client at risk for thermal injury. The client should wear sturdy shoes for ambulation.

1. A client is scheduled for a tympanoplasty. What action(s) by the nurse are (is) most appropriate? (Select all that apply.) a. Administer preoperative opioids. b. Assess for allergies to local anesthetics. c. Ensure that informed consent is on the health record. d. Give prescribed antivertigo medications. e. Teach that hearing improves immediately.

ANS: C Preoperatively, the nurse ensures that informed consent is in the health record. Local anesthetics can be used, but general anesthesia is used more often. Antivertigo medications are not used. Hearing will be decreased immediately after the operation until the ear packing is removed

2. The nurse is caring for a client who has chronic pyelonephritis. What assessment finding would the nurse expect? a. Fever b. Flank pain c. Hypertension d. Nausea and vomiting

ANS: C The client who has chronic pyelonephritis has renal damage and therefore has hypertension. The other assessment findings commonly occur in clients with acute pyelonephritis.

45.At 4:45 p.m., a nurse assesses a client with diabetes mellitus who is recovering from an abdominal hysterectomy 2 days ago. The nurse notes that the client is confused and diaphoretic. The nurse reviews the assessment data provided in the chart below: Capillary Blood Glucose Testing (AC/HS) At 0630: 95 At 1130: 70 At 1630: 47 Dietary Intake Breakfast: 10% eaten client states she is not hungry Lunch: 5% eaten client is nauseous; vomits once After reviewing the clients assessment data, which action is appropriate at this time? a. Assess the clients oxygen saturation level and administer oxygen. b. Reorient the client and apply a cool washcloth to the clients forehead. c. Administer dextrose 50% intravenously and reassess the client. d. Provide a glass of orange juice and encourage the client to eat dinner.

ANS: C The clients symptoms are related to hypoglycemia. Since the client has not been tolerating food, the nurse should administer dextrose intravenously. The clients oxygen level could be checked, but based on the information provided, this is not the priority. The client will not be reoriented until the glucose level rises.

10. The nurse is teaching assistive personnel (AP) about fluid restriction for a client who has acute kidney injury (AKI). The client's 24-hour urinary output is 120 mL. How much fluid would the client be allowed to have over the next 24 hours? a. 380 mL b. 500 mL c. 620 mL d. 750 mL

ANS: C The general principle for fluid restriction for clients is that they may have a daily fluid intake of 500 mL plus the amount of their urinary output. In this case, 120 mL urinary output plus 500 mL equals 620 mL fluid allowance.

16. A postoperative client has respiratory depression after receiving morphine for pain. Which medication and dose does the nurse prepare to administer? a. Flumazenil 0.2 to 1 mg b. Flumazenil 2 to 10 mg c. Naloxone 0.4 to 2 mg d. Naloxone 4 to 20 mg

ANS: C The nurse would prepare to administer naloxone, an opioid antagonist, at a dose of between 0.04 and 0.05 mg up to 2 mg, depending on the client's symptoms. Flumazenil is a benzodiazepine antagonist.

25.A nurse assesses a client with diabetes mellitus 3 hours after a surgical procedure and notes the clients breath has a fruity odor. Which action should the nurse take? a. Encourage the client to use an incentive spirometer. b. Increase the clients intravenous fluid flow rate. c. Consult the provider to test for ketoacidosis. d. Perform meticulous pulmonary hygiene care.

ANS: C The stress of surgery increases the action of counterregulatory hormones and suppresses the action of insulin, predisposing the client to ketoacidosis and metabolic acidosis. One manifestation of ketoacidosis is a fruity odor to the breath. Documentation should occur after all assessments have been completed. Using an incentive spirometer, increasing IV fluids, and performing pulmonary hygiene will not address this clients problem.

9. A nurse is caring for four clients. After the hand-off report, which client does the nurse see first? a. Client with osteoporosis and a white blood cell count of 27,000/mm3 b. Client with osteoporosis and a bone fracture who requests pain medication c. Post-microvascular bone transfer client whose distal leg is cool and pale d. Client with suspected bone tumor who just returned from having a spinal CT

ANS: C This client is the priority because the assessment findings indicate a critical lack of perfusion. A high white blood cell count is an expected finding for the client with osteoporosis. The client requesting pain medication should be seen second. The client who just returned from a CT scan is stable and needs no specific post procedure care.

7. After a total knee arthroplasty, a client is on the postoperative nursing unit with a continuous femoral nerve blockade. On assessment, the nurse notes the skin of both legs is pale pink, warm, and dry, but the client is unable to dorsiflex or plantar flex the surgical foot. What action would the nurse take next? a. Document the findings and monitor as prescribed. b. Increase the frequency of monitoring the client. c. Notify the surgeon or anesthesia provider immediately. d. Palpate the client's bladder or perform a bladder scan.

ANS: C With the femoral nerve block, the client would still be able to dorsiflex and plantarflex the affected surgical foot. Since this client has an abnormal finding, the nurse would notify either the surgeon or the anesthesia provider immediately. Documentation is the last priority. Increasing the frequency of assessment may be appropriate, but first the nurse must notify the appropriate provider. Palpating the bladder is not related.

14. A nurse reviews the laboratory test values for a client with a new diagnosis of diabetes mellitus type 2. Which A1C value would the nurse expect? a. 5.0% b. 5.7% c. 6.2% d. 7.4%

ANS: D A client is diagnosed with diabetes if the client's A1C is 6.5% or greater. All listed values are below that level except for 7.4%.

3. A nurse assesses a client who is recovering from a diskectomy 6 hours ago. Which assessment finding should the nurse address first? a. Sleepy but arouses to voice b. Dry and cracked oral mucosa c. Pain present in lower back d. Bladder palpated above pubis

ANS: D A distended bladder may indicate damage to the sacral spinal nerves. The other findings require the nurse to provide care but are not the priority or a complication of the procedure.

14. The nurse is caring for four clients with traumatic brain injuries. Which client should the nurse assess first? a. Client with cerebral perfusion pressure of 72 mm Hg b. Client who has a Glasgow Coma Scale score of 12 c. Client with a PaCO2 of 36 mm Hg who is on a ventilator d. Client who has a temperature of 102 F (38.9 C)

ANS: D A fever is a poor prognostic indicator in clients with brain injuries. The nurse should see this client first. A Glasgow Coma Scale score of 12, a PaCO2 of 36, and cerebral perfusion pressure of 72 mm Hg are all desired outcomes.

9. The nurse reviews the function of thyroid gland hormones. What is the primary function of calcitonin? a. Sodium and potassium balance b. Magnesium balance c. Norepinephrine balance d. Calcium and phosphorus balance

ANS: D Calcitonin is the primary body hormone that is secreted from the thyroid gland and is responsible for maintaining calcium and phosphorus balance.

4. A nurse assesses a client on the medical-surgical unit. Which statement made by the client should alert the nurse to the possibility of hypothyroidism? a. My sister has thyroid problems. b. I seem to feel the heat more than other people. c. Food just doesnt taste good without a lot of salt. d. I am always tired, even with 12 hours of sleep.

ANS: D Clients with hypothyroidism usually feel tired or weak despite getting many hours of sleep. Thyroid problems are not inherited. Heat intolerance is indicative of hyperthyroidism. Loss of taste is not a manifestation of hypothyroidism.

17.After teaching a client who has diabetes mellitus and proliferative retinopathy, nephropathy, and peripheral neuropathy, the nurse assesses the clients understanding. Which statement made by the client indicates a correct understanding of the teaching? a. I have so many complications; exercising is not recommended. b. I will exercise more frequently because I have so many complications. c. I used to run for exercise; I will start training for a marathon. d. I should look into swimming or water aerobics to get my exercise.

ANS: D Exercise is not contraindicated for this client, although modifications based on existing pathology are necessary to prevent further injury. Swimming or water aerobics will give the client exercise without the worry of having the correct shoes or developing a foot injury. The client should not exercise too vigorously.

17. A nurse assesses a client with a neurologic disorder. Which assessment finding should the nurse identify as a late manifestation of amyotrophic lateral sclerosis (ALS)? a. Dysarthria b. Dysphagia c. Muscle weakness d. Impairment of respiratory muscles

ANS: D In ALS, progressive muscle atrophy occurs until a flaccid quadriplegia develops. Eventually, the respiratory muscles are involved, which leads to respiratory compromise. Dysarthria, dysphagia, and muscle weakness are early clinical manifestations of ALS.

1. The nurse is caring for an older client who has kyphosis and a widened gait. For which health problems is the client at risk? a. Osteoporosis b. Contracture c. Osteopenia d. Falls

ANS: D Kyphosis is caused by bone loss and causes the client to bend forward which changes the center of gravity leading to problems with balance. Older adults who have balance issues are at risk for falls.

15. The nurse is teaching an older adult how to prevent buildup of ear wax. Which statement by the nurse is most appropriate? a. "Visit your primary health GcaRreApDrEovSidLeAr Bea.chCmOMonth for wax removal." b. "Drink plenty of water and other liquids to prevent hardening of the ear wax." c. "Irrigate each ear once a month to remove wax and prevent was buildup." d. "Put one drop of mineral oil in each ear once a week at bedtime."

ANS: D Mineral oil provides lubrication to soften cerumen so that it flows out of the ears to prevent buildup. It is a safer method than irrigating the ears. If needed, the client would need to go to a primary health care provider for removal of impaction. Drinking water helps prevent hardening of wax but does not necessarily prevent wax buildup.

38.A nurse assesses a client with diabetes mellitus who self-administers subcutaneous insulin. The nurse notes a spongy, swelling area at the site the client uses most frequently for insulin injection. Which action should the nurse take? a. Apply ice to the site to reduce inflammation. b. Consult the provider for a new administration route. c. Assess the client for other signs of cellulitis. d. Instruct the client to rotate sites for insulin injection.

ANS: D The clients tissue has been damaged from continuous use of the same site. The client should be educated to rotate sites. The damaged tissue is not caused by cellulitis or any type infection, and applying ice may cause more damage to the tissue. Insulin can only be administered subcutaneously and intravenously. It would not be appropriate or practical to change the administration route.

8. An emergency room nurse initiates care for a client with a cervical spinal cord injury who arrives via emergency medical services. Which action should the nurse take first? a. Assess level of consciousness. b. Obtain vital signs. c. Administer oxygen therapy. d. Evaluate respiratory status.

ANS: D The first priority for a client with a spinal cord injury is assessment of respiratory status and airway patency. Clients with cervical spine injuries are particularly prone to respiratory compromise and may even require intubation. The other assessments should be performed after airway and breathing are assessed.

14. A nurse assesses clients on the medical-surgical unit. Which client is at greatest risk for bladder cancer? a. A 25-year-old female with a history of sexually transmitted diseases b. A 42-year-old male who has worked in a lumber yard for 10 years c. A 55-year-old female who has had numerous episodes of bacterial cystitis d. An 86-year-old male with a 50pack-year cigarette smoking history

ANS: D The greatest risk factor for bladder cancer is a long history of tobacco use. The other factors would not necessarily contribute to the development of this specific type of cancer.

10. A nursing student is instructed to remove a clients ear packing and instill ear drops. What action by the student requires intervention by the registered nurse? a. Assessing the eardrum with an otoscope b. Inserting a cotton ball in the ear after the drops c. Warming the eardrops in water for 5 minutes d. Washing the hands and removing the packing

ANS: D The student should wash his or her hands, don gloves, and then remove the packing. The other actions are correct.

4. A client with a stroke is being evaluated for fibrinolytic therapy. What information from the client or family is most important for the nurse to obtain? a. Loss of bladder control b. Other medical conditions c. Progression of symptoms d. Time of symptom onset

ANS: D The time limit for initiating fibrinolytic therapy for a stroke is 3 to 4.5 hours, so the exact time of symptom onset is the most important information for this client. The other information is not as critical.

15. A nurse cares for a client in skeletal traction. The nurse notes that the skin around the clients pin sites is swollen, red, and crusty with dried drainage. Which action should the nurse take next? a. Request a prescription to decrease the traction weight. b. Apply an antibiotic ointment and a clean dressing. c. Cleanse the area, scrubbing off the crusty areas. d. Obtain a prescription to culture the drainage.

ANS: D These clinical manifestations indicate inflammation and possible infection. Infected pin sites can lead to osteomyelitis and should be treated immediately. The nurse should obtain a culture and assess vital signs. The provider should be notified. By decreasing the traction weight, applying a new dressing, or cleansing the area, the infection cannot be significantly treated.

3. A nurse receives a report on a client who had a left-sided stroke and has homonymous hemianopsia. What action by the nurse is most appropriate for this client? a. Assess for bladder retention and/or incontinence. b. Listen to the clients lungs after eating or drinking. c. Prop the clients right side up when sitting in a chair. d. Rotate the clients meal tray when the client stops eating.

ANS: D This condition is blindness on the same side of both eyes. The client must turn his or her head to see the entire visual field. The client may not see all the food on the tray, so the nurse rotates it so uneaten food is now within the visual field. This condition is not related to bladder function, difficulty swallowing, or lack of trunk control.


संबंधित स्टडी सेट्स

Quiz 2 (Nazi Berlin & Divided Berlin)

View Set

Autism spectrum disorder (ASD) (Sherpath)

View Set

Life-Limited to the Payment of Funeral and Burial Expense

View Set

ECO-2050 HW Assignment 12, Chapter 11

View Set